Está en la página 1de 182

Cmo elaborar preguntas para evaluaciones escritas en el rea de ciencias bsicas y clnicas

Tercera edicin (Revisada)

National Board of Medical Examiners 3750 Market Street Philadelphia, PA 19104

El National Board of Medical Examiners (NBME) no dispone de copias impresas de esta publicacin. Se puede obtener copias adicionales descargando este manual desde el sitio web del NBME. El NBME confiere permiso para duplicar y distribuir este manual siempre que (1) los derechos reservados y el permiso para publicar el manual aparezcan en cada copia, (2) se use este manual solamente para propsitos cientficos, educativos y no comerciales, y (3) no se modifique el documento. El NBME se reserva cualquier otro derecho no explcitamente mencionado aqu.

Copyright 1996, 1998 National Board of Medical Examiners (NBME). Copyright 2001, 2002 National Board of Medical Examiners (NBME). Copyright 2006 National Board of Medical Examiners (NBME). Reservados todos los derechos. Impreso en Estados Unidos.

Cmo elaborar preguntas para evaluaciones escritas en el rea de ciencias bsicas y clnicas
Tercera edicin (Revisada)

Autores colaboradores Susan M. Case, PhD y David B. Swanson, PhD* National Board of Medical Examiners 3750 Market Street Philadelphia, PA 19104
*La Dra. Case era funcionaria ejecutiva de evaluaciones de la National Board of Medical Examiners cuando se publicaron las primeras tres ediciones de este manual; actualmente se desempea como Directora de Evaluaciones en la National Conference of Bar Examiners. El Dr. Swanson es vicepresidente adjunto del rea de Servicios Profesionales en la National Board of Medical Examiners.

Reconocimiento
La National Board of Medical Examiners (NBME) quiere agradecer al Dr. Alberto Galofr de la Pontificia Universidad Catlica de Chile por sus esfuerzos en la elaboracin de esta edicin en espaol de Cmo elaborar preguntas para evaluaciones escritas en el rea de ciencias bsicas y clnicas. La NBME tambin quiere extender un agradecimiento especial a todos los colaboradores que entusiastamente ayudaron al Dr. Galofr, as como a la Pontificia Universidad Catlica de Chile por su apoyo a este proyecto.

ndice
Seccin I Temas relacionados con el formato y la estructura de las preguntas de una evaluacin . . . . . . . . . . . . . . . . . . . . . . .
Pgina 7

Captulo 1. Introduccin . . . . . . . . . . . . . . . . . . . . . . . . . . . . . . . . . . . . . . . . . . . . . . . . . . . . . . . . . . . . . . . . . . . . . . . . . . . . 9 La evaluacin: un componente importante de la enseanza . . . . . . . . . . . . . . . . . . . . . . . . . . . . . . . . . . . . . . . . . . . . . . . 9 Temas relacionados con el muestreo . . . . . . . . . . . . . . . . . . . . . . . . . . . . . . . . . . . . . . . . . . . . . . . . . . . . . . . . . . . . . . . . 10 Importancia de las consideraciones psicomtricas . . . . . . . . . . . . . . . . . . . . . . . . . . . . . . . . . . . . . . . . . . . . . . . . . . . . . . 11 Captulo 2. Formatos de tems de opcin mltiple . . . . . . . . . . . . . . . . . . . . . . . . . . . . . . . . . . . . . . . . . . . . . . . . . . . . . . Preguntas de verdadero/falso versus las de seleccin de la mejor respuesta . . . . . . . . . . . . . . . . . . . . . . . . . . . . . . . . . La familia de preguntas verdadero/falso . . . . . . . . . . . . . . . . . . . . . . . . . . . . . . . . . . . . . . . . . . . . . . . . . . . . . . . . . . . . . La familia de tems de seleccin de la mejor respuesta . . . . . . . . . . . . . . . . . . . . . . . . . . . . . . . . . . . . . . . . . . . . . . . . . . Lo esencial de los formatos de los tems . . . . . . . . . . . . . . . . . . . . . . . . . . . . . . . . . . . . . . . . . . . . . . . . . . . . . . . . . . . . . Captulo 3. Defectos tcnicos en los tems . . . . . . . . . . . . . . . . . . . . . . . . . . . . . . . . . . . . . . . . . . . . . . . . . . . . . . . . . . . . . Temas relacionados con la estrategia para rendir exmenes . . . . . . . . . . . . . . . . . . . . . . . . . . . . . . . . . . . . . . . . . . . . . . Temas relacionados con dificultades irrelevantes . . . . . . . . . . . . . . . . . . . . . . . . . . . . . . . . . . . . . . . . . . . . . . . . . . . . . . Resumen de los defectos tcnicos en los tems . . . . . . . . . . . . . . . . . . . . . . . . . . . . . . . . . . . . . . . . . . . . . . . . . . . . . . . . Uso de trminos imprecisos en las preguntas de una evaluacin . . . . . . . . . . . . . . . . . . . . . . . . . . . . . . . . . . . . . . . . . . 13 13 14 16 18 19 19 22 26 27 31

Seccin II Cmo redactar preguntas de seleccin de la mejor respuesta en el rea de ciencias bsicas y clnicas . . . . . . . . . .

Reglas bsicas para los tems de seleccin de la mejor respuesta . . . . . . . . . . . . . . . . . . . . . . . . . . . . . . . . . . . . . . . . . . 33 Captulo 4. Contenido de los tems: evaluacin de la aplicacin de conocimientos sobre ciencias bsicas . . . . . . . . . . 35 Contenido de los tems para las ciencias bsicas . . . . . . . . . . . . . . . . . . . . . . . . . . . . . . . . . . . . . . . . . . . . . . . . . . . . . . . 35 Plantillas de tems . . . . . . . . . . . . . . . . . . . . . . . . . . . . . . . . . . . . . . . . . . . . . . . . . . . . . . . . . . . . . . . . . . . . . . . . . . . . . . 38 Plantillas adicionales . . . . . . . . . . . . . . . . . . . . . . . . . . . . . . . . . . . . . . . . . . . . . . . . . . . . . . . . . . . . . . . . . . . . . . . . . . . . . 39 Tipos de preguntas; Ejemplos de preguntas introductorias y listas de opciones . . . . . . . . . . . . . . . . . . . . . . . . . . . . . . . 40 Redaccin de las opciones: cmo modificar la dificultad de un tem . . . . . . . . . . . . . . . . . . . . . . . . . . . . . . . . . . . . . . . 41 La forma de un tem . . . . . . . . . . . . . . . . . . . . . . . . . . . . . . . . . . . . . . . . . . . . . . . . . . . . . . . . . . . . . . . . . . . . . . . . . . . . . 42 Aprendizaje basado en problemas y el uso de un grupo de preguntas en base a un caso . . . . . . . . . . . . . . . . . . . . . . . . 43 Ejemplos de tems para las ciencias bsicas . . . . . . . . . . . . . . . . . . . . . . . . . . . . . . . . . . . . . . . . . . . . . . . . . . . . . . . . . . . 47 Captulo 5. Contenido de los tems: evaluacin de la aplicacin de conocimientos sobre ciencias clnicas . . . . . . . . . . 51 Mtodos de evaluacin . . . . . . . . . . . . . . . . . . . . . . . . . . . . . . . . . . . . . . . . . . . . . . . . . . . . . . . . . . . . . . . . . . . . . . . . . . . 51

Temas generales con respecto a lo que se debe evaluar . . . . . . . . . . . . . . . . . . . . . . . . . . . . . . . . . . . . . . . . . . . . . . . . . Evaluacin de la retencin de hechos aislados o de la aplicacin del conocimiento . . . . . . . . . . . . . . . . . . . . . . . . . . . Cmo redactar tems de seleccin de la mejor respuesta . . . . . . . . . . . . . . . . . . . . . . . . . . . . . . . . . . . . . . . . . . . . . . . . Puntos precisos sobre los enunciados de los tems . . . . . . . . . . . . . . . . . . . . . . . . . . . . . . . . . . . . . . . . . . . . . . . . . . . . . Verbosidad, adornos y pistas falsas: mejoran el tem? . . . . . . . . . . . . . . . . . . . . . . . . . . . . . . . . . . . . . . . . . . . . . . . . . . Cmo redactar tems relacionados con las tareas del mdico . . . . . . . . . . . . . . . . . . . . . . . . . . . . . . . . . . . . . . . . . . . . . Cmo redactar tems sobre temas difciles . . . . . . . . . . . . . . . . . . . . . . . . . . . . . . . . . . . . . . . . . . . . . . . . . . . . . . . . . . .

52 53 56 57 58 61 66 69 71 72 74 75 76 77 81 82 90 93 94 96 97 99

Seccin III Items de emparejamiento ampliado

................................................................ Captulo 6. Items de emparejamiento ampliado (tipo R) . . . . . . . . . . . . . . . . . . . . . . . . . . . . . . . . . . . . . . . . . . . . . . . . . . Cmo evitar errores al redactar tems de emparejamiento ampliado para su evaluacin . . . . . . . . . . . . . . . . . . . . . . . . Ejemplos de oraciones introductorias y temas para las listas de opciones . . . . . . . . . . . . . . . . . . . . . . . . . . . . . . . . . . . Aspectos adicionales sobre las opciones para los conjuntos R . . . . . . . . . . . . . . . . . . . . . . . . . . . . . . . . . . . . . . . . . . . . Cmo redactar los enunciados de los tems . . . . . . . . . . . . . . . . . . . . . . . . . . . . . . . . . . . . . . . . . . . . . . . . . . . . . . . . . . . Ejemplos de enunciados buenos y malos que usan la misma lista de opciones . . . . . . . . . . . . . . . . . . . . . . . . . . . . . . . Resumen de los pasos para redactar tems de emparejamiento ampliado . . . . . . . . . . . . . . . . . . . . . . . . . . . . . . . . . . . . Ejemplos de conjuntos de emparejamiento ampliado . . . . . . . . . . . . . . . . . . . . . . . . . . . . . . . . . . . . . . . . . . . . . . . . . . . Pasos para organizar un grupo que redacte tems tipo R de contenido clnico . . . . . . . . . . . . . . . . . . . . . . . . . . . . . . . . Formulario para redactar los conjuntos tipo R . . . . . . . . . . . . . . . . . . . . . . . . . . . . . . . . . . . . . . . . . . . . . . . . . . . . . . . . Cdigo SPSSX de muestra para determinar el puntaje de los exmenes de opcin mltiple que incluyen tems de emparejamiento ampliado . . . . . . . . . . . . . . . . . . . . . . . . . . . . . . . . . . . . . . . . . . . . . . . . . . . . . . . . . . . . . . . Comparacin de los tems del formato de cinco opciones con los de emparejamiento ampliado . . . . . . . . . . . . . . . . . Desde los A hasta los R y de vuelta . . . . . . . . . . . . . . . . . . . . . . . . . . . . . . . . . . . . . . . . . . . . . . . . . . . . . . . . . . . . . . . . . Captulo 7. Items de seleccin de N respuestas: una ampliacin del formato de emparejamiento ampliado . . . . . . . . . .

Seccin IV Temas complementarios

. . . . . . . . . . . . . . . . . . . . . . . . . . . . . . . . . . . . . . . . . . . . . . . . . . . . . . . . . . . . . . . . . . . . . . . . . 105 Captulo 8. Interpretacin de los resultados del anlisis de los tems . . . . . . . . . . . . . . . . . . . . . . . . . . . . . . . . . . . . . . 107 Captulo 9. Cmo establecer un estndar de aprobado/reprobado . . . . . . . . . . . . . . . . . . . . . . . . . . . . . . . . . . . . . . . . . 111 Definiciones y principios bsicos . . . . . . . . . . . . . . . . . . . . . . . . . . . . . . . . . . . . . . . . . . . . . . . . . . . . . . . . . . . . . . . . . . 111 Dos mtodos de determinacin de estndares en base a juicios sobre los tems . . . . . . . . . . . . . . . . . . . . . . . . . . . . . 112 Estndares de compromiso relativo/absoluto: el mtodo de Hofstee . . . . . . . . . . . . . . . . . . . . . . . . . . . . . . . . . . . . . . 114 Captulo 10. Reflexiones varias sobre temas relacionados con la evaluacin . . . . . . . . . . . . . . . . . . . . . . . . . . . . . . . . . 115 Apndice A. El cementerio de formatos de los tems de la NBME . . . . . . . . . . . . . . . . . . . . . . . . . . . . . . . . . . . . . . . . . . 117 Apndice B. Ejemplos de plantillas, enunciados, preguntas introductorias y listas de opciones para la redaccin de tems en el rea de ciencias bsicas y clnicas . . . . . . . . . . . . . . . . . . . . . . . . . . . . . . . . . . . . . . . . . . . . . . . . . . . . . .129

Prlogo de la tercera edicin


Este manual fue redactado con el objetivo de ayudar a que los profesores universitarios mejoren la calidad de las preguntas de opcin mltiple que redactan para sus exmenes. El manual proporciona un panorama general de los formatos de tems, y se concentra en los formatos tradicionales de emparejamiento y de seleccin de la mejor respuesta. Revisa los temas relacionados con los defectos tcnicos y el contenido de los tems. Tambin brinda informacin bsica para ayudar a que los profesores revisen los indicadores estadsticos de la calidad de los tems despus de administrar una evaluacin. Tambin se proporciona un panorama general sobre las tcnicas de determinacin de estndares. No se tratan en profundidad los temas relacionados con la programacin de exmenes. Nos hemos concentrado casi exclusivamente en el nivel correspondiente a las preguntas y hemos dejado para otra publicacin todo lo relacionado con la planificacin del nivel del examen. Creemos que este manual ser de gran utilidad, principalmente para aquellos profesores universitarios que ensean a alumnos de medicina en cursos de ciencias bsicas y prctica clnica. Los ejemplos se concentran en educacin mdica para estudiantes universitarios; sin embargo, el enfoque general para la redaccin de tems puede ser til para evaluar a los candidatos en otros niveles. Este manual refleja las lecciones que hemos aprendido durante la elaboracin de preguntas y exmenes en los ltimos 20 aos. Durante este perodo, hemos revisado (literalmente) decenas de miles de preguntas de opcin mltiple y hemos llevado a cabo talleres sobre la redaccin de preguntas dirigidos a miles de encargados de la redaccin de los exmenes de USMLE, NBME y de las juntas de especialidades mdicas, al igual que para profesores en ms de 60 facultades de medicina que elaboran preguntas para sus propias evaluaciones. Cada participante en los talleres nos ha ayudado a formarnos una opinin con respecto a cmo redactar preguntas de mejor calidad para las evaluaciones y, con el transcurso del tiempo, hemos podido expresar (segn creemos) las razones y el por qu. Esperamos que este manual ayude a comunicar estos pensamientos.

Dra. Susan M. Case, PhD Dr. David B. Swanson, PhD Enero de 1998

Seccin I Temas relacionados con el formato y la estructura de las preguntas de una evaluacin
Esta seccin analiza los temas estructurales que son relevantes para la elaboracin de preguntas de alta calidad para los exmenes. La siguiente seccin revisar los temas relacionados con el contenido de las preguntas.

Captulo 1 Introduccin
La evaluacin: un componente importante de la enseanza
La evaluacin es un componente esencial de la enseanza. Si se la utiliza adecuadamente, puede ayudar a lograr objetivos curriculares importantes. El impacto de las decisiones que se refieren a cmo y cundo evaluar los conocimientos y el rendimiento de los alumnos no puede sobreestimarse. Un objetivo fundamental de los exmenes es indicar lo que usted considera importante. Los exmenes son un poderoso motivador y los alumnos aprendern lo que creen que usted considera valioso. La evaluacin tambin ayuda a llenar brechas en la enseanza ya que estimula a los estudiantes para que amplen sus conocimientos por sus propios medios y participen en las oportunidades educativas que estn disponibles. Este resultado de los exmenes es especialmente importante en las rotaciones clnicas donde el plan de estudios puede variar entre un estudiante y otro, como consecuencia de factores tales como el entorno clnico y el flujo aleatorio de pacientes. El resultado tambin puede ser importante en algunos escenarios de aprendizaje de ciencias bsicas (por ejemplo, aprendizaje basado en problemas) en los que las experiencias educativas pueden ser diferentes entre los estudiantes. Ya que los exmenes tienen tanta influencia en el aprendizaje de un alumno, es fundamental que usted elabore evaluaciones que favorezcan sus objetivos educativos. Si se les presenta un examen que evala las habilidades clnicas prcticas, los alumnos saldrn de la biblioteca para entrar a la clnica en donde podrn buscar ayuda para el desarrollo de sus habilidades en exmenes fsicos. Por el contrario, si se les presenta un examen que evala solamente lo que recuerdan de hechos aislados, esto los llevar a memorizar los libros de revisin del curso. Este manual se concentra en cmo redactar preguntas de opcin mltiple de alta calidad que evalen la capacidad para interpretar datos y tomar decisiones. Consideramos que stos son componentes fundamentales de las habilidades clnicas. El camino que los alumnos recorren para lograr la maestra o hasta la excelencia, ser menos escabroso si reciben comentarios continuos sobre sus avances.

Captulo 1. Introduccin

Objetivos de la evaluacin
Indicar a sus alumnos qu material es importante Incentivar a los alumnos a estudiar Identificar las reas deficientes que requieren de ms estudio Determinar las notas finales o tomar decisiones de promocin Detectar los puntos dbiles del curso/programa de estudios

Qu se debe evaluar?
El contenido del examen debe coincidir con los objetivos del curso o de las rotaciones clnicas Los temas fundamentales deben tener ms peso que los temas menos importantes El tiempo de evaluacin dedicado a cada tema debe reflejar la importancia relativa del tema. La muestra de preguntas debe representar los objetivos del aprendizaje

Temas relacionados con el muestreo


El objetivo de toda evaluacin es extraer conclusiones con respecto a las aptitudes de los alumnos: es decir, conclusiones que se extienden ms all de los problemas particulares (o, de manera equivalente, los casos o preguntas de la evaluacin) que se incluyen en el examen hasta alcanzar el dominio ms amplio a partir del cual se realiza el muestreo de los casos (o preguntas). Todos estamos de acuerdo en que la evaluacin requiere tiempo. Tambin est claro que si se aumenta el tiempo que se dedica a una actividad, se debe disminuir el tiempo para el resto de las actividades. Ya sea que decida sobre un plan integral de evaluacin o sobre lo que debe incluir en un examen en particular, bsicamente se enfrenta con un problema de muestreo. Con las preguntas de opcin mltiple, primero necesita decidir lo que desea incluir en el examen. La atencin que se presta al evaluar un aspecto debe reflejar su importancia relativa. Es necesario que explore temas y tambin habilidades (por ejemplo, determinar el diagnstico, decidir sobre el siguiente paso en el tratamiento); no se pueden abarcar todos los temas en las preguntas. El rendimiento observado en la muestra proporciona una base para calcular los logros en el dominio ms amplio que es el que realmente interesa. La naturaleza de la muestra determina el grado de reproducibilidad del clculo de la capacidad verdadera (si es confiable, generalizable) y de exactitud (si es vlido). Si la muestra no es representativa del campo ms

10

amplio de inters (por ejemplo, incluye solamente contenido relacionado con temas cardiovasculares en un examen de competencia en la prctica mdica general), los resultados del examen estarn sesgados y no proporcionarn una base slida para calcular el logro en el campo de inters. Si la muestra es demasiado pequea, los resultados del examen no sern lo suficientemente precisos (reproducibles, confiables) como para garantizar que reflejen la competencia verdadera. Con un examen de opcin mltiple, casi siempre existe un evaluador (normalmente un equipo de computacin) y una serie de preguntas o grupos de preguntas; escoger una muestra implica la seleccin de un subgrupo de preguntas para incluir en el examen. Con otros mtodos de evaluacin (por ejemplo, exmenes orales que se basan en casos de pacientes, exmenes estandarizados de pacientes, exmenes de ensayos), la confeccin de una muestra es mucho ms complicada. Cualquier mtodo que no pueda evaluarse de manera mecnica requiere la consideracin de una segunda dimensin en la muestra: la dimensin del evaluador. En estos exmenes, lo que interesa es el rendimiento en un amplio espectro de casos, y se prefiere que el puntaje sea independiente de quin sea el evaluador. Por lo tanto, es necesario confeccionar la muestra en dos dimensiones: una que considere las preguntas o casos y la otra, a los jueces o evaluadores. Es necesario abarcar un amplio espectro de casos ya que el rendimiento en un caso no predice con precisin el rendimiento en otros casos. Tambin es necesario que la muestra tenga en cuenta a diferentes tipos de evaluadores para minimizar los efectos de la rigurosidad o la indulgencia de un evaluador, as como otros temas como el efecto de aureola que provocan problemas en la coherencia del puntaje de distintos evaluadores. Con muestras amplias, los altibajos en el rendimiento y los altibajos por las diferencias en los evaluadores tienden a nivelarse. Si bien este manual se concentra en las preguntas de opcin mltiple, consideramos que es generalmente apropiado usar distintos mtodos de evaluacin. Ninguno podr abarcar todas las habilidades que son de inters. Tambin se debe tener en cuenta que el mtodo utilizado para la evaluacin no afecta directamente la calidad del examen ni determina el componente de la competencia que se mide en el examen.

Importancia de las consideraciones psicomtricas


El grado de importancia de las caractersticas psicomtricas de un mtodo de evaluacin est determinado por el objetivo del examen y por las decisiones que se tomarn sobre la base de los resultados. En el caso de exmenes en los cuales hay mucho en juego (aquellos que se usan para decisiones sobre promocin o graduacin, incluso para determinar la nota en el curso), los resultados deben ser razonablemente reproducibles (precisos, confiables) y exactos (vlidos). Para los exmenes de menor relevancia, las caractersticas psicomtricas son menos relevantes y la consideracin principal debe ser la de dirigir el aprendizaje del alumno. Como se mencion anteriormente, para generar un puntaje reproducible, es necesario elaborar una muestra con un contenido amplio (es decir, normalmente una docena o ms de casos, 100 o ms preguntas de opcin multiple o preguntas de respuesta breve).

Captulo 1. Introduccin

11

Los trabajos que se indican a continuacin contienen ms informacin sobre temas relacionados con la evaluacin en general.
Swanson DB. (1987) A measurement framework for performance-based tests. En Hart I, Harden R (Eds.), Further Developments in Assessing Clinical Competence. Montreal: Can-Heal Publications, 13-4. Swanson DB, Norcini JJ, Grosso L. (1987) Assessment of clinical competence: written and computer-based simulations. Assessment and Evaluation in Higher Education, 12(3), 220-246. Friedman C, de Bliek R, Greer D, Mennin S, Norman G, Sheps C, Swanson DB, Woodward C. (1990) Charting the winds of change: recommendations for evaluating innovative medical curricula. Academic Medicine, 65, 8-14. Swanson DB, Case SM, van der Vleuten CP. (1991:1997/2nd edition) Strategies for student assessment. In Boud, D. & Feletti, G (Eds.), The Challenge of Problem-Based Learning. Londres, Kogan Page Ltd., 269-282. Newble DI, Dauphinee D, Woolliscroft JO, MacDonald M, Mulholland H, Page G, Swanson DB, Thomson A, van der Vleuten CP. (1994) Guidelines for assessing clinical competence. Teaching and Learning in Medicine, 6:3, 213-220. Swanson DB, Norman G, Linn R. (1995) Performance-based assessment: Lessons from the health professions. Educational Researcher, 24:5, pp511,35. Versin abreviada reimpresa en Pedagogue: Perspectives on Health Sciences Education, 6 (Verano 1996), McMaster University Program for Educational Development, 1-7. Swanson DB, Case SM (1997) Assessment in basic science instruction: Directions for practice and research. Advances in Health Sciences Education: Theory & Practice, 2: 71-84. Case SM, (1997) Assessment of truths we hold as self-evident and their implications. En Scherpbier AJJA, van der Vleuten CPM, Rethans JJ, van der Steeg AFW (Eds.), Advances in Medical Education. Dordrecht, Holanda: Kluwer Academic Publishers, 2-6. Swanson DB, Clauser BE, Case SM. (1999) Clinical skills assessment with standardized patients in high-stakes tests: A framework for thinking about score precision, equating, and security. Advances in Health Sciences Education, 4: 67-106.

Los siguientes trabajos analizan ms detalladamente los temas relacionados con el formato de las preguntas.
Case SM, Downing SM. (1989) Performance of various multiple-choice item types on medical specialty examinations: types A, B, C, K, and X. Proceedings of the 28th Annual Conference on Research in Medical Education, 167-172. Swanson DB, Case SM. (1992) Trends in written assessment: a strangely biased perspective. En Harden R, Hart I, Mulholland H (Eds.), Approaches to the Assessment of Clinical Competence: Part 1. Page Brothers, Norwich, Inglaterra, 38-53. Case SM, Swanson DB. (1993) Extended-matching items: a practical alternative to free-response questions. Teaching and Learning in Medicine, 5(2), 107-115. Swanson DB, Case SM. (1995) Variation in item difficulty and discrimination by item format on Part I (basic sciences) and Part II (clinical sciences) of U.S. licensing examinations. En Rothman A, Cohen R (Eds.), Proceedings of the Sixth Ottawa Conference on Medical Education. University of Toronto Bookstore Custom Publishing, 285-287. Norman G, Swanson DB, Case SM. (1996) Conceptual and methodological issues in studies comparing assessment formats. Teaching and Learning in Medicine, 8(4):208-216.

12

Captulo 2 Formatos de tems de opcin mltiple


Para que una pregunta de un examen sea aceptable, debe cumplir con dos criterios bsicos. Primero, debe incluir contenido relevante. Esta es una condicin esencial que se analizar en detalle ms adelante en este manual. Obviamente, el contenido del tem es de vital importancia, pero el concentrarse en el contenido relevante no es suficiente para garantizar que la pregunta sea adecuada. Los tems que intentan evaluar temas de importancia fundamental no pueden hacerlo a menos que estn correctamente estructurados. Evitar defectos que beneficien a candidatos que saben rendir exmenes y evitar dificultades irrelevantes son requisitos previos que se deben cumplir para que las preguntas del examen generen puntajes vlidos.

Preguntas de verdadero/falso versus las de seleccin de la mejor respuesta


El universo de las preguntas de opcin mltiple puede dividirse en dos familias de preguntas: aquellas que requieren que el alumno indique todas las respuestas que son correctas (verdadero/falso) y aquellas que requieren que el alumno indique una sola respuesta (la mejor). Cada familia est representada por varios formatos especficos, como se indica a continuacin: Los formatos de tems de verdadero/falso requieren que el alumno seleccione todas las opciones que sean verdaderas. C (A/B/Ambas/Ninguna) K (preguntas complejas de verdadero/falso) X (preguntas simples de verdadero/falso) Simulaciones, como por ejemplo, Resolucin de problemas del paciente Los formatos de tems de seleccin de la mejor respuesta requieren que los alumnos seleccionen una sola respuesta, la que sea mejor. A (4 o ms opciones, preguntas nicas o conjuntos) B (preguntas de emparejamiento con 4 5 opciones en conjuntos de 2 a 5 preguntas) R (preguntas de emparejamiento ampliado en conjuntos de 2 a 20 preguntas) Las letras usadas para identificar los formatos de tems no tienen significado propio. Han sido asignadas en forma ms o menos secuencial a los nuevos formatos de tems a medida que se han desarrollado (ver Apndice A).

Captulo 2. Formatos de tems de opcin mltiple

13

La familia de preguntas de verdadero/falso


Las familias de preguntas de verdadero/falso y las de seleccin de la mejor respuesta implican diferentes tareas para el alumno. Los tems de verdadero/falso exigen que el alumno seleccione todas las opciones que sean verdaderas. Para estas preguntas, el alumno debe decidir cul es el lmite de corte, es decir hasta qu punto una respuesta debe ser verdadera para ser considerada como tal. Si bien esta tarea requiere de una consideracin adicional (ms de la que se necesita para seleccionar la mejor respuesta), es probable que esta consideracin adicional no est relacionada con la experiencia o el conocimiento clnico. Con bastante frecuencia, los alumnos tienen que adivinar lo que el redactor pensaba porque las opciones no son totalmente falsas ni totalmente verdaderas. El siguiente es un ejemplo de un tem aceptable de verdadero/falso desde una perspectiva estructural.* Observe que el enunciado es claro y que las opciones son totalmente verdaderas o falsas sin ambigedad. Las opciones se pueden diagramar de la siguiente manera. Cules de las siguientes son enfermedades recesivas ligadas al cromosoma X? 1. Hemofilia A (hemofilia clsica) 2. Fibrosis qustica 3. Distrofia muscular de Duchenne 4. Enfermedad de Tay-Sachs 2 4 Opciones totalmente incorrectas 1 3 Opciones totalmente correctas

Las afirmaciones correctas sobre la fibrosis qustica (FQ) incluyen: 1. La incidencia de la FQ es de 1:2000. 2. Los nios que padecen FQ generalmente mueren en la adolescencia. 3. Los hombres que padecen FQ son estriles. 4. La FQ es una enfermedad autosmica recesiva.

Este tem de verdadero/falso est mal construido. Las opciones 1, 2 y 3 no pueden considerarse como absolutamente verdaderas ni falsas; un grupo de expertos no podra ponerse de acuerdo sobre las respuestas. Si se considera la opcin 1, observe que la incidencia no es exactamente 1:2000; los expertos solicitaran ms informacin: Se refiere a Estados Unidos? Se presenta en todos los grupos tnicos? Si se modifica la redaccin a aproximadamente 1:2000, no se soluciona el problema ya que no se especifica la banda. Se presentan problemas similares con las opciones 2 y 3, mientras que la opcin 4 es clara.

* De acuerdo con la tradicin, en el caso de tems de verdadero/falso, las opciones se identifican con nmeros, mientras que en el caso de los tems de seleccin de la mejor respuesta, las opciones se identifican con letras.

14

Aunque es una broma (del segundo autor), este tem de verdadero/falso ejemplifica un problema comn: el enunciado no es claro. Segn su perspectiva, las opciones 1, 2 y 3 podran ser verdaderas; desde otro punto de vista, las opciones 1, 2 y 3 podran ser falsas y la 4 verdadera.

Se llega al corazn de un hombre a travs de 1. la aorta 2. las arterias pulmonares 3. las venas pulmonares 4. el estmago

En este ejemplo de verdadero/falso, existen trminos poco precisos en las opciones que proporcionan pistas para los alumnos que saben rendir exmenes. Por ejemplo, el trmino puede en las opciones 1, 2 y 3 le da indicios al alumno que sabe rendir de que esas opciones son verdaderas. Es ms difcil adivinar en la opcin 4: qu significa normalmente? Los estudios de investigacin han demostrado que estos trminos de frecuencia poco precisos no tienen una definicin compartida. Los expertos no estaran de acuerdo sobre si la cuarta opcin es verdadera o falsa.

En la evaluacin clnica del dolor crnico, 1. la actitud personal del mdico con respecto al dolor puede afectar su criterio mdico. 2. las emociones desagradables pueden convertirse en quejas de dolor fsico. 3. el dolor puede tener un significado simblico. 4. el aspecto de la cara o la postura del cuerpo dan normalmente una indicacin de la gravedad del dolor.

Los defectos de esta pregunta son ms sutiles. La dificultad es que el alumno debe realizar suposiciones sobre la gravedad de la enfermedad, la edad del paciente y sobre si la enfermedad ha sido o no tratada. Las diferentes suposiciones llevan a diferentes respuestas, incluso entre los expertos.

En los nios, la comunicacin interventricular est asociada a 1. soplo sistlico 2. hipertensin pulmonar 3. tetraloga de Fallot 4. cianosis

Observe que en cada ejemplo de un tem mal construido, el enunciado no es claro, las opciones contienen trminos poco precisos o las opciones son parcialmente correctas. En cada caso, un grupo de expertos tendra dificultades para lograr un consenso sobre cul es la respuesta correcta.

Captulo 2. Formatos de tems de opcin mltiple

15

Como los alumnos deben elegir todas las opciones que son verdaderas, los tems de verdadero/falso deben cumplir con las siguientes reglas: Los enunciados deben ser claros y sin ambigedades. Se deben evitar las frases como est asociado a; es til para; es importante y palabras que proporcionan pistas, tales como puede o podra ser; as como tambin los trminos poco precisos, como normalmente o con frecuencia. Las opciones deben ser absolutamente verdaderas o falsas; no se admiten trminos medios. Evite las frases y palabras que se indicaron en el primer punto anteriormente.

La familia de tems de seleccin de la mejor respuesta


Contrariamente a las preguntas de verdadero/falso, las de seleccin de la mejor respuesta (tipo A) indican de forma expresa el nmero de opciones que se deben seleccionar. El tipo A es el formato de opcin mltiple ms usado. Constan de un enunciado (por ejemplo, la presentacin de un caso clnico) y una pregunta introductoria seguida de una serie de opciones que tienen normalmente una respuesta correcta y cuatro distractores. La pregunta a continuacin describe una situacin (en este caso, un paciente) y pide al alumno que indique la causa ms probable del problema. Enunciado: Un hombre de 32 aos presenta debilidad progresiva en sus extremidades desde hace 4 das. Goza de buena salud, excepto por una infeccin en las vas respiratorias superiores hace 10 aos. Su temperatura corporal es de 37,8 C (100 F), la presin arterial es de 130/80 mm Hg, su pulso es de 94/min y la frecuencia respiratoria es de 42/min y superficial. Tiene debilidad simtrica en ambos lados del rostro y en los msculos distales y proximales de las extremidades. La sensacin est intacta. No se pueden provocar reacciones en los reflejos tendinosos profundos; las respuestas plantares son flexoras. Pregunta introductoria: Cul de las siguientes opciones sera el diagnstico ms probable? Opciones: A. Encefalomielitis diseminada aguda B. Sndrome de Guillain-Barr C. Miastenia grave D. Poliomielitis E. Polimiositis

16

Observe que las opciones incorrectas no lo son totalmente. Las opciones se pueden diagramar de la siguiente manera: D Menos correcta C A E B Ms correcta

Aunque las respuestas incorrectas no son totalmente incorrectas, son menos correctas que la respuesta de la clave. Se pide al alumno que seleccione el diagnstico ms probable; todos los expertos estaran de acuerdo en que el diagnstico ms probable es B; tambin acordaran que los otros diagnsticos son algo probables, pero menos probables que B. Siempre que las opciones puedan disponerse en una sola secuencia, en este caso de diagnstico ms probable a diagnstico menos probable, los distractores en las preguntas de seleccin de la mejor respuesta no tienen que ser totalmente incorrectos. Esta pregunta presenta defectos en su construccin. Despus de leer el enunciado, el alumno tiene solamente una idea general sobre el tema de la pregunta. En un intento por determinar la mejor respuesta, el alumno tiene que decidir si se presenta con ms frecuencia en mujeres es ms o menos verdadero que en muy pocas ocasiones se asocia con dolor agudo en una articulacin. Esta es una comparacin de manzanas con naranjas. Para establecer el orden de la veracidad relativa de las opciones, stas deben diferir en una sola dimensin o de lo contrario, todas las opciones deben ser absolutamente un 100% verdaderas o falsas.

Cul de las siguientes opciones es correcta sobre la seudogota? A. Se presenta con ms frecuencia en mujeres. B. En muy pocas ocasiones se asocia con dolor agudo en una articulacin. C. Puede estar asociada a un hallazgo de condrocalcinosis. D. Es claramente hereditaria en la mayora de los casos. E. Responde satisfactoriamente al tratamiento con alopurinol.

Sexo A

D Herencia

Asociaciones Falso Verdadero

Aqu se presenta un posible diagrama de estas opciones. Las opciones son heterogneas y tratan sobre hechos variados; no pueden ordenarse desde la menos hasta la ms correcta en una sola dimensin. A pesar de que esta pregunta aparentemente evala el conocimiento de varios puntos diferentes, su error inherente excluye esta posibilidad. La pregunta no es clara; no se puede dar una respuesta sin ver las opciones.

Captulo 2. Formatos de tems de opcin mltiple

17

A diferencia de las opciones que figuran en la pregunta sobre seudogota, las opciones que se refieren al sndrome de GuillainBarr son homogneas (por ejemplo, todas se refieren al diagnstico); los alumnos que conocen este tema pueden ordenar en forma secuencial las opciones en una sola dimensin. Las preguntas de seleccin de la mejor respuesta que estn elaboradas adecuadamente cumplen con la regla de ocultar las opciones. Las preguntas podran administrarse en forma de preguntas para escribir la respuesta. El enunciado incluye la pregunta completa.

Lo esencial de los formatos de los tems


Le recomendamos que no utilice las preguntas de verdadero/falso. Si bien muchos redactores consideran que las preguntas de verdadero/falso son ms fciles de redactar que las de seleccin de la mejor respuesta, nosotros creemos que son ms problemticas. El redactor de las preguntas tena una idea en particular en su mente cuando escribi la pregunta; no obstante, una revisin detallada revela comnmente las dificultades sutiles que no eran evidentes para el autor de la pregunta. A menudo, la diferencia entre verdadero y falso no es clara y es frecuente que los revisores posteriores alteren la clave de respuestas. En consecuencia, los revisores vuelven a redactar o eliminan las preguntas de verdadero/falso con ms frecuencia que las preguntas redactadas en otro formato. Algunas ambigedades se pueden aclarar, pero otras no. Existe una razn final que es ms convincente que todas las que se mencionaron anteriormente. Encontramos que para evitar la ambigedad, nos vemos llevados a evaluar lo que el alumno recuerda sobre un hecho aislado y esto es lo que tratamos de evitar por todos los medios. Las preguntas de seleccin de la mejor respuesta son las que mejor evalan la aplicacin de conocimientos, la integracin, la sntesis y el juicio. En consecuencia, la NBME ha dejado de usar totalmente los formatos de verdadero/falso en sus exmenes. Tambin recomendamos que no se utilicen las preguntas negativas de tipo A. Las que causan ms inconvenientes son aquellas que tienen la forma de: Cada una de las siguientes opciones es correcta EXCEPTO o Cul de las siguientes opciones NO es correcta? Estas tienen el mismo problema que las preguntas de verdadero/falso: si las opciones no se pueden ordenar en una sola secuencia, el alumno es incapaz de determinar cul es la respuesta menos o ms correcta. Por otra parte, en algunos exmenes ocasionalmente usamos preguntas de tipo A negativas con un enfoque correcto junto con opciones de una sola palabra, en gran parte como un sustituto (deficiente) de las preguntas que les indican a los alumnos que seleccionen ms de una respuesta. Ms adelante, se analiza en este manual un mejor formato para este fin el formato Elija N en el que los alumnos deben seleccionar N respuestas. El Apndice A ejemplifica una variedad de formatos de preguntas que ya no se usan en los exmenes de la NBME.

18

Captulo 3 Defectos tcnicos en los tems


Esta seccin describe dos tipos de defectos tcnicos en los tems: estrategia para rendir exmenes y dificultad irrelevante. Los defectos relacionados con la estrategia para rendir exmenes hacen posible que algunos alumnos respondan correctamente a la pregunta con facilidad, solamente con sus aptitudes para rendir exmenes. Estos defectos comnmente se presentan en tems que no tienen un buen enfoque o que no cumplen con la regla de ocultar las opciones. Los defectos relacionados con dificultades irrelevantes hacen que la pregunta sea difcil por razones que no estn relacionadas con la caracterstica central que se evala. El objetivo de esta seccin es detectar los defectos comunes y alentarlo para que los elimine de sus preguntas de manera que pueda proporcionar un campo de juego uniforme para los alumnos que tienen estrategias para rendir exmenes y para los que no las tienen. La probabilidad de responder correctamente a una pregunta debe estar relacionada con la experiencia del alumno en el tema que se evala y no con su experiencia en estrategias para rendir exmenes.

Temas relacionados con la estrategia para rendir exmenes


Pistas gramaticales: uno o ms distractores no tienen correspondencia gramatical con el enunciado Como el redactor del tem tiende a prestar ms atencin a la respuesta correcta que a los distractores, los errores gramaticales son ms comunes en las respuestas falsas. En este ejemplo, los alumnos que saben rendir exmenes eliminaran las opciones A y C porque no respetan gramatical o lgicamente el enunciado. Los alumnos que tienen estrategias para rendir exmenes deben elegir solamente entre B, D y E. Un hombre de 60 aos es trasladado al departamento de urgencias por la polica, que lo encontr en estado inconsciente en la acera. Luego de determinar que la va area est abierta, el primer paso en el tratamiento debera ser la administracin intravenosa de A. examen del lquido cefalorraqudeo B. glucosa con vitamina B1 (tiamina) C. tomografa computarizada de la cabeza D. fenitona E. diazepam

Captulo 3. Defectos tcnicos en los tems

19

Pistas lgicas: un subgrupo de las opciones son en conjunto exhaustivas En este tem, las opciones A, B y C incluyen todas las posibilidades. El alumno con estrategias para rendir exmenes sabe que A, B o C deben ser correctas, mientras que los alumnos que carecen de estas estrategias dedican tiempo a considerar las opciones D y E. A menudo, los redactores de tems agregan D y E solamente porque desean enumerar cinco opciones. En estas situaciones, es probable que el redactor no haya prestado mucha atencin a los mritos de las opciones D y E. Algunas veces, son parcialmente correctas y confusas porque no se pueden ordenar en forma secuencial en la misma dimensin que las opciones A, B y C. Este defecto se observa comnmente con opciones tales como aumenta, disminuye y permanece igual.

El delito A. se distribuye igualmente entre las clases sociales B. est sobrerrepresentado entre los pobres C. est sobrerrepresentado entre la clase media y rica D. es fundamentalmente un indicio de desajuste psicosexual E. est llegando a una meseta de tolerancia en la nacin

Trminos absolutos: son aquellos como siempre o nunca que se usan en las opciones En este tem, las opciones A, B y E contienen trminos que son menos absolutos que los de las opciones C y D. El alumno con estrategias para rendir exmenes eliminar las opciones C y D porque tienen menos posibilidades de ser verdaderas que otras expresadas de manera menos absoluta. Observe que este defecto no se presentara si el enunciado estuviera bien definido y las opciones fueran cortas; solamente surge cuando los verbos estn incluidos en las opciones en lugar de estar en la pregunta introductoria.

En pacientes que padecen demencia avanzada, tipo Alzheimer, el defecto de memoria A. puede ser tratado correctamente con fosfatidilcolina (lecitina) B. podra ser una secuela de enfermedad de Parkinson precoz C. nunca se observa en pacientes con ovillos neurofibrilares en la autopsia D. nunca es severo E. posiblemente compromete al sistema colinrgico

20

Respuesta correcta extensa: la respuesta correcta es ms extensa, ms especfica o ms completa que las otras opciones En este tem, la opcin C es ms extensa que las otras opciones; tambin es la nica opcin doble. Los redactores de tems tienden a prestar ms atencin a la respuesta correcta que a los distractores. Como son maestros, redactan respuestas correctas extensas que incluyen material instructivo adicional, informacin entre parntesis, advertencias, etc. Algunas veces, se llega a casos bastante extremos: la respuesta correcta tiene un prrafo de extensin mientras que los distractores son de una sola palabra.

La ganancia secundaria A. es sinnimo de simulacin B. es un problema frecuente en los trastornos obsesivos compulsivos C. es una complicacin de una serie de enfermedades y tiende a prolongar muchas de ellas D. nunca se observa en casos de dao cerebral orgnico

Repeticiones de palabras: se incluye una palabra o frase en el enunciado y en la respuesta correcta Este tem utiliza la palabra irreal en el enunciado y desrealizacin es la respuesta correcta. Algunas veces, una palabra se repite solamente con sentido metafrico, por ejemplo, un enunciado que menciona un dolor seo, y la respuesta correcta comienza con osteo-.

Un hombre de 58 aos de edad, con antecedentes de alcoholismo e internaciones psiquitricas previas presenta confusin y agitacin. Comenta que siente que el mundo es algo irreal. El sntoma se llama A. despersonalizacin B. descarrilamiento C. desrealizacin D. dficit focal de memoria E. ansiedad seal

Captulo 3. Defectos tcnicos en los tems

21

Estrategia de convergencia: la respuesta correcta incluye la mayor cantidad de elementos en comn con las otras opciones Este defecto es menos evidente que los otros, pero aparece con frecuencia y vale la pena tenerlo en cuenta. El defecto se observa en distintas formas. La premisa subyacente es que la respuesta correcta es la opcin que tiene ms puntos en comn con las otras opciones; no es probable que sea un valor extremo. Por ejemplo, en las opciones numricas, la respuesta correcta es con ms frecuencia el nmero del medio y no un valor extremo. En las opciones dobles, es ms probable que la respuesta correcta sea la opcin que tiene la mayor cantidad de elementos en comn con las otras opciones incorrectas. Por ejemplo, si las opciones son lpiz y lapicera, lpiz y marcador, lpiz y crayn, lapicera y rotulador, es probable que la respuesta correcta sea lpiz y lapicera (es decir, simplemente al contar, lpiz aparece 3 veces en las opciones; lapicera aparece dos veces; los otros elementos aparecen solamente una vez). Aunque puede parecer ridculo, este defecto ocurre porque los redactores de tems comienzan con la respuesta correcta y escriben variantes de la respuesta correcta como distractores. Por lo tanto, es muy probable que la respuesta correcta tenga elementos en comn con el resto de las Los anestsicos locales son ms eficaces en opciones. Es ms probable que las respuestas incorrectas sean A. la forma aninica y actan desde el interior valores extremos ya que el redactor tiene dificultad para de la membrana nerviosa generar opciones incorrectas viables. En este ejemplo, el B. la forma catinica y actan desde el interior alumno que cuenta con estrategias para rendir exmenes de la membrana nerviosa eliminara la opcin forma aninica por considerarla poco C. la forma catinica y actan desde el exterior probable ya que forma aninica figura solamente una vez; de la membrana nerviosa ese alumno tambin excluira en el exterior de la membrana D. la forma sin carga y actan desde el interior nerviosa porque la palabra exterior figura con menos frede la membrana nerviosa cuencia que interior. Por lo tanto, el alumno debera decidir E. la forma sin carga y actan desde el exterior entre las opciones B y D. Ya que tres de las cinco opciones de la membrana nerviosa hablan sobre la carga, la estrategia del alumno que sabe rendir exmenes lo llevara a elegir la opcin B.

Temas relacionados con dificultades irrelevantes


Las opciones son extensas, complicadas o dobles Este tem muestra un defecto comn. El enunciado presenta un contenido irrelevante, pero lo ms significante es que las opciones son demasiado extensas y complicadas. Si se intenta decidir entre estas opciones, es necesario leerlas varias veces debido al nmero de elementos en cada opcin. Esto puede llevar a que el tem, en lugar de medir el conocimiento del contenido, mida la velocidad de lectura. Observe que este defecto se relaciona solamente con las opciones. Hay muchas pre-

22

guntas elaboradas correctamente que incluyen una extensa descripcin del enunciado. Las decisiones sobre la extensin del enunciado deben tomarse segn el objetivo del tem. Si el objetivo es evaluar si el alumno puede o no interpretar y resumir la informacin para determinar, por ejemplo, el diagnstico ms probable, entonces es adecuado que el enunciado incluya una descripcin bastante completa de la situacin. Los comits de revisin cientfica de pares en las HMO pueden iniciar acciones contra las credenciales de un mdico a fin de cuidar la atencin de los participantes de la HMO. Existe un requisito asociado para garantizar que el mdico reciba el proceso debido durante el desarrollo de estas actividades. El proceso debido debe incluir cul de las siguientes opciones? A. Una notificacin, un tribunal imparcial, un consejo, la oportunidad de escuchar y confrontar la evidencia en su contra. B. La notificacin correspondiente, un tribunal con facultades para tomar decisiones, la posibilidad de confrontar testigos en su contra y la oportunidad de presentar evidencia en su defensa. C. Una notificacin razonable y oportuna, un tribunal imparcial con facultad para tomar decisiones, la oportunidad de escuchar la evidencia en su contra y de confrontar a los testigos y la capacidad de presentar evidencia en su defensa.

Los datos numricos no se expresan de manera uniforme Cuando se usan opciones numricas, las opciones deben estar incluidas en orden numrico y en un solo formato (es decir, como trminos individuales o como intervalos). La confusin se presenta cuando los formatos estn mezclados y cuando las opciones se enumeran en un orden ilgico o con un formato desigual. En este ejemplo, las opciones A, B y C se expresan como intervalos, mientras que las opciones D y E son porcentajes especficos. Todas las opciones deben expresarse como intervalos o como porcentajes especficos; no se recomienda mezclarlos. Adems, el intervalo para la opcin C incluye las opciones D y E, lo que prcticamente lleva a descartar estas opciones como respuestas correctas.

Luego de un segundo episodio infeccioso, cul es la probabilidad de que una mujer sea estril? A. Menos del 20% B. De un 20 a un 30% C. Ms de 50% D. 90% E. 75%

Captulo 3. Defectos tcnicos en los tems

23

Los trminos que indican frecuencia en las opciones son poco precisos (por ejemplo, pocas veces, normalmente) Los estudios de investigacin han demostrado que los trminos que indican frecuencia y que son poco precisos no son definidos ni interpretados de manera uniforme ni siquiera por los expertos. En la pgina 29 se incluye un anlisis ms completo de esta investigacin.

La obesidad severa en los primeros aos de la adolescencia A. normalmente responde de manera dramtica al rgimen alimenticio B. a menudo est relacionada con trastornos endocrinos C. tiene un 75% de posibilidades de desaparecer espontneamente D. tiene un mal pronstico E. normalmente responde a la farmacoterapia y a la psicoterapia intensiva

La redaccin de las opciones no es paralela; las opciones siguen un orden ilgico Este tem muestra un defecto comn en el sentido de que las opciones son extensas y la redaccin usada dificulta y hace ms lenta la determinacin de cul es la ms correcta. Generalmente, este defecto se puede corregir con una revisin atenta y cuidadosa. En este tem en particular, se puede cambiar la pregunta introductoria a Por cul de las siguientes razones no se puede extraer una conclusin a partir de los resultados presentados? Luego, se pueden corregir las opciones (por ejemplo, A. No se realiz un seguimiento de los nios que no se vacunaron; B. El nmero de casos era demasiado bajo; C. Slo participaron nios varones en el estudio, y se puede escribir una opcin nueva para D).

En un estudio de vacunas, se administr a 200 varones de 2 aos de edad una vacuna contra una enfermedad determinada y luego fueron controlados durante cinco aos para determinar la presencia de la enfermedad. De este grupo, el 85% nunca contrajo la enfermedad. Cul de las siguientes afirmaciones sobre estos resultados es correcta? A. No se puede extraer una conclusin ya que no se realiz el seguimiento de los nios que no se vacunaron B. El nmero de casos (es decir, 30 casos en un perodo de cinco aos) es demasiado reducido para obtener conclusiones estadsticamente significativas C. No se pueden extraer conclusiones porque slo participaron nios varones en el estudio D. El porcentaje de eficacia de la vacuna (%) se calcula como 85-15/100

24

Ninguna de las anteriores se utiliza como una opcin de respuesta La frase Ninguna de las anteriores es problemtica en Qu ciudad est ms cerca de Nueva York? aquellas preguntas que implican un discernimiento y donde las opciones no son absolutamente verdaderas o falsas. Si se A. Boston pretende que la respuesta correcta sea una de las otras B. Chicago opciones enumeradas, los alumnos que han estudiado se C. Dallas enfrentarn con un dilema porque debern decidir entre una D. Los ngeles opcin perfecta muy detallada y la que usted pens como E. Ninguna de las anteriores opcin correcta. A menudo, los alumnos podrn imaginar una Si los alumnos seleccionan la opcin E, no se puede opcin ms correcta que la que usted pens. El uso de saber si piensan en Filadelfia o Londres. ninguna de las anteriores transforma esencialmente al tem en uno de verdadero/falso; cada opcin tiene que ser evaluada como ms o menos verdadera que el universo de opciones no enumeradas. Generalmente ser posible solucionar dichas preguntas mediante el reemplazo de ninguna de las anteriores por otra opcin que signifique aproximadamente lo mismo pero que sea ms especfica. Por ejemplo, en un tem que solicita al alumno que especifique la farmacoterapia ms adecuada, si se reemplaza ninguna de las anteriores por en este momento no se deben administrar medicamentos, se eliminar la ambigedad de ninguna de las anteriores. Los enunciados son engaosos o innecesariamente complicados Algunas veces, los redactores de tems pueden trabajar con una pregunta fcil y transformarla en otra tan complicada que solamente el ms resuelto de los alumnos la leer. ste es un ejemplo de ese tipo de tems. El sistema de notacin en I: a V: es complejo; el indicar el orden en nmeros romanos despus de atravesar esa notacin es irrelevante e innecesariamente difcil.

Ordene a los padres/madres de los siguientes nios con sndrome de Down segn el riesgo de recidiva, de mayor riesgo a menor riesgo. Suponga que la edad de la madre en todos los casos es de 22 aos de edad y que los embarazos posteriores tuvieron lugar dentro de un perodo de 5 aos. Los cariotipos de las hijas son: I: 46, XX, -14, +T (14q21q) pat II: 46, XX, -14, +T (14q21q) de novo III: 46, XX, -14, +T (14q21q) mat IV: 46, XX, -21, +T (14q21q) pat V: 47, XX, -21, +T (21q21q) (sin cariotipo de padre/madre) A. B. C. D. E. III, IV, I, V, II IV, III, V, I, II III, I, IV, V, II IV, III, I, V, II III, IV, I, II, V

Captulo 3. Defectos tcnicos en los tems

25

Resumen de los defectos tcnicos en los tems


Temas relacionados con la estrategia para rendir exmenes
Pistas gramaticales: uno o ms distractores no tienen correspondencia gramatical con el enunciado Pistas lgicas: un subgrupo de las opciones son en conjunto exhaustivas Trminos absolutos: se usan trminos como siempre o nunca en algunas opciones Respuesta correcta extensa: la respuesta correcta es ms extensa, ms especfica o ms completa que las otras opciones Repeticiones de palabras: se incluye una palabra o frase en el enunciado y en la respuesta correcta Estrategia de convergencia: la respuesta correcta incluye la mayor cantidad de elementos en comn con las otras opciones

Temas relacionados con dificultades irrelevantes


Las opciones son extensas, complicadas o dobles Los datos numricos no se expresan de manera uniforme Los trminos en las opciones son poco precisos (por ejemplo, pocas veces, normalmente) La redaccin en las opciones no es paralela Las opciones estn en un orden que no es lgico Ninguna de las anteriores se usa como una opcin Los enunciados son engaosos o innecesariamente complicados La respuesta a una pregunta depende de la respuesta a otra pregunta relacionada

Pautas generales para la elaboracin de un tem


Asegrese de que el tem pueda ser respondida sin mirar a las opciones o bien de que las opciones sean 100% verdaderas o falsas. Incluya en el enunciado la mayor cantidad de datos posible; el enunciado debe ser extenso y las opciones cortas. Evite incluir informacin innecesaria. Evite redactar tems engaosos y excesivamente complicados. Redacte opciones que sean gramaticalmente coherentes y lgicamente compatibles con el enunciado; enumrelas en orden alfabtico o lgico. Redacte distractores que sean verosmiles y de la misma longitud que la respuesta. Evite usar trminos absolutos, tales como siempre, nunca y todo en las opciones; tambin evite usar trminos poco precisos, como normalmente y con frecuencia. Evite las preguntas redactadas en forma de oracin negativa (por ejemplo, aquellas que incluyen excepto o no en la pregunta introductoria). Si es necesario expresar el enunciado mediante una oracin negativa, use solamente opciones breves (preferentemente de una sola palabra). Y lo que es ms importante: concntrese en conceptos importantes; no pierda tiempo en evaluar datos triviales.

26

Uso de trminos imprecisos en las preguntas de una evaluacin


Si bien nosotros usamos trminos imprecisos en nuestro lenguaje cotidiano y en la escritura, estos trminos causan confusin cuando son usados en las preguntas de una evaluacin. En un estudio llevado a cabo en la NBME, 60 integrantes de ocho comits de evaluacin que redactaron preguntas para distintos exmenes de especialidades mdicas revisaron una lista de los trminos usados en preguntas de opcin multiple para expresar algunos conceptos relacionados con la frecuencia e indicaron el porcentaje de tiempo que cada trmino reflejaba. Los resultados (que se muestran a continuacin) indicaron que los trminos no tienen una definicin operativa que sea por lo general compartida, incluso entre los mismos redactores de tems. El valor medio, ms o menos una desviacin estndar, exceda el 50 por ciento en ms de la mitad de las frases. Por ejemplo, en promedio, los redactores de tems consideraban que el trmino con frecuencia indicaba el 70% del tiempo; la mitad de ellos crean que se refera a entre un 45% y un 75% del tiempo; las respuestas reales variaron entre un 20% y un 80%. Resulta de particular inters observar que los valores de con frecuencia casi se superpusieron con los valores de pocas veces. La consecuencia de estos resultados en la interpretacin de las preguntas de examen vara segn el formato de la pregunta. Los trminos poco precisos crean muchos ms inconvenientes graves en los diferentes tipos de tems de verdadero/falso (tems de tipo K, C y X) que en los de seleccin de la mejor respuesta (tipo A y R). Por ejemplo, los trminos imprecisos ocasionan inconvenientes importantes en los tems de verdadero/falso, como es el caso de este ejemplo:

Las afirmaciones verdaderas sobre la seudogota incluyen: 1. Se presenta comnmente en mujeres. 2. A menudo se asocia con dolor agudo. 3. Normalmente es hereditaria. 4. Con frecuencia hay un aumento en los niveles de calcio srico.

En los tems de verdadero/falso, el alumno tiene que discernir si cada opcin es verdadera o falsa. Cuando las opciones no son absolutamente verdaderas o falsas, el alumno confa en su definicin personal de los trminos ambiguos o sus presunciones sobre el significado que estos trminos tienen para el redactor. Otra posibilidad es que las respuestas del alumno reflejen un estilo de respuesta personal (la tendencia a responder ya sea verdadero o falso cuando no se conoce la respuesta correcta). Estos factores de estilo en la respuesta pueden afectar ms la respuesta correcta o incorrecta por parte del alumno que la influencia que tiene el conocimiento real sobre el tema de estudio, y pueden ser parte de la razn por la cual las preguntas de verdadero/falso tienden a producir resultados mediocres.

Captulo 3. Defectos tcnicos en los tems

27

El volver a formular las opciones mediante la especificacin de los nmeros exactos no soluciona el problema. Por ejemplo, la afirmacin la incidencia entre las mujeres es de 1:2000 no sera una modificacin adecuada de la opcin 1 en el ejemplo que se muestra. La incidencia no es exactamente de 1:2000 y como no se especifica una banda, los alumnos definirn sus propias bandas, ya sea de forma amplia o reducida, supuestamente, segn sus estilos de respuesta personales. En las preguntas de verdadero/falso, la forma adecuada para las opciones numricas es generar una comparacin (por ejemplo, la incidencia es mayor que en la osteoartritis) o especificar un intervalo (por ejemplo, la incidencia es de 1:1000 a 1:2000). El tema que se plantea anteriormente con los tems de verdadero/falso no es tan problemtico con los tems de seleccin de la mejor respuesta que estn bien elaborados (es decir, aquellos que establecen una pregunta clara y tienen opciones homogneas). Por ejemplo, la siguiente pregunta incluye un trmino poco preciso en el enunciado; sin embargo, como la tarea consiste en seleccionar la mejor opcin como respuesta, la pregunta es relativamente poco ambigua. Cul de los siguientes valores de laboratorio normalmente aumenta en pacientes que padecen seudogota? Los problemas surgen con los tems de seleccin de la mejor respuesta que tienen un trmino poco preciso en las opciones, como en este ejemplo. La nica forma de hacer que este tem sea ms ambiguo sera usar como quinta opcin ninguna de las anteriores.

Los pacientes con seudogota tienen dolor: A. con frecuencia B. normalmente C. a menudo D. comnmente

28

Porcentaje de equivalencia

i oc As o ad

Diagrama de caja de distribucin de datos de las respuestas relacionadas con trminos que indican frecuencia. Los resultados se basan en las respuestas de 60 integrantes de ocho comits de redaccin de tems. La lnea horizontal en cada caja indica la respuesta mediana; las cajas incluyen los intervalos del 50% de las respuestas. Las lneas verticales se extienden hasta los valores ms altos y ms bajos indicados. Por ejemplo, la respuesta mediana para con frecuencia indic un 70% del tiempo; la mitad de las personas consideraban que se refera a entre un 45% y un 75% del tiempo; las respuestas reales variaron entre un 20% y un 80%, casi en superposicin con pocas veces. Extrado de: Case SM. (1994) The use of imprecise terms in examination questions: How frequent is frequently? Academic Medicine,
69(suppl):S4-S6.

o ud en m A te en m n om te C en m le ab ob Pr e bl si rra po cu o Es e ia qu enc cu fre on C te en m al su U rte pa or ay po m em La el ti d

C i as a nc nu

s ca Po

s ce ve

n co

Captulo 3. Defectos tcnicos en los tems

29

Seccin II Cmo redactar preguntas de seleccin de la mejor respuesta en el rea de ciencias bsicas y clnicas
Los captulos anteriores analizaron temas tcnicos relacionados con la elaboracin de preguntas de opcin mltiple. La Seccin II se concentra en el contenido de la pregunta.

Reglas bsicas para los tems de seleccin de la mejor respuesta


Cada tem debera centrarse en un concepto importante, que normalmente es un problema clnico comn o potencialmente grave. No dedique tiempo del examen a preguntas que evalen el conocimiento de datos triviales. Concntrese en los problemas que se encontraran en la vida real. Evite el uso de preguntas complicadas, triviales o excesivamente complejas. Cada tem debera evaluar la aplicacin de conocimientos y no la retencin de un hecho aislado. El enunciado del tem puede ser relativamente extenso; las opciones deberan ser breves. La vieta clnica proporciona una buena base para la pregunta. Para el rea de las ciencias clnicas, cada una debera comenzar con la presentacin de un problema de un paciente. Luego, debera continuar con la historia clnica (que incluya la duracin de los signos y sntomas), los datos del examen fsico, resultados de los estudios de diagnstico, tratamiento inicial, hallazgos posteriores, etc. Las vietas pueden incluir solamente un subgrupo de esta informacin pero se debera presentar la informacin en el orden especificado. Para el rea de las ciencias bsicas, la vieta del paciente puede ser muy breve; las vietas de laboratorio tambin son adecuadas. El enunciado del tem debe generar una pregunta clara y debe ser posible llegar a la respuesta con las opciones ocultadas. Para determinar si la pregunta est centrada en un tema en particular, cubra las opciones y observe si la pregunta es clara y si los alumnos pueden intentar responderla solamente a partir de la informacin en el enunciado. Si no es posible, vuelva a elaborar el contenido del enunciado o de las opciones. Todos los distractores (es decir, opciones incorrectas) deben ser homogneos. Deben estar dentro de la misma categora que la respuesta correcta (por ejemplo, diagnsticos, anlisis, tratamientos, pronsticos, alternativas de disposicin). Vuelva a redactar los distractores que no sean de la misma categora. Evite el uso de opciones dobles (por ejemplo, hacer W y X; hacer Y debido a Z) a menos que la respuesta correcta y todos los distractores tengan doble opcin. Vuelva a escribir las opciones dobles para que se concentren en un nico punto. Todos los distractores deben ser convincentes, gramaticalmente correctas, lgicamente compatibles y de la misma extensin (relativa) que la respuesta correcta. Ordene las opciones de manera lgica (por ejemplo, numricamente) o en orden alfabtico. Evite los defectos tcnicos que proporcionan un beneficio especial a los alumnos que saben rendir o que presentan una dificultad irrelevante.

NO redacte preguntas del tipo Cul de las siguientes afirmaciones es correcta? o Las opciones a continuacin son correctas EXCEPTO. Estas preguntas no se concentran en un tema en particular y tienen opciones heterogneas. Someta a cada pregunta a las cinco pruebas contempladas en las reglas anteriores. Si una pregunta pasa con xito los cinco puntos, significa que probablemente est correctamente redactada y que se refiere a un tema adecuado.

33

Tambin puede consultar:


Swanson DB, Case SM. Assessment in basic science instruction: directions for practice and research. Advances in Health Sciences Education: Theory and Practice. 1997; 2:71-84.

34

Captulo 4 Contenido de los tems: evaluacin de la aplicacin de conocimientos sobre ciencias bsicas
Contenido de los tems para las ciencias bsicas
Tradicionalmente, los tems se clasifican segn los procesos cognitivos necesarios para responder a la pregunta (por ejemplo, retencin, interpretacin, resolucin de problemas, memoria, comprensin o razonamiento). Los tems que evalan la retencin estn pensadas para evaluar el conocimiento del alumno sobre hechos aislados. Los tems de interpretacin requieren que el alumno revise algn tipo de informacin (con frecuencia en forma de grfico o tabla) y que llegue a una conclusin (por ejemplo, un diagnstico). Los tems de resolucin de problemas presentan una situacin en la que los alumnos deben decidir cmo actuar (por ejemplo, el siguiente paso en el tratamiento de un paciente). Los tems de interpretacin o de resolucin de problemas implican habilidades de nivel superior en lugar de simplemente usar la memoria de retencin de informacin sobre hechos. Lamentablemente, los procesos cognitivos necesarios para responder a una pregunta son generalmente difciles de determinar porque dependen tanto de la formacin del alumno como del contenido del tem. Por ejemplo, es posible que un tem sobre el flujo sanguneo en un paciente con comunicacin interventricular requiera que un cardilogo peditrico o fisilogo cardiovascular simplemente recuerden lo que ya saben con poco razonamiento, mientras que un tpico alumno inicial quizs tenga que razonar la respuesta a partir de principios bsicos de hemodinmica. Los procesos cognitivos involucrados en la respuesta a una pregunta varan segn el alumno, por lo cual este enfoque taxonmico resulta difcil de usar. Un mtodo ms objetivo y simple basa la clasificacin del tem en la tarea del alumno. Si un tem requiere que el alumno llegue a una conclusin, haga una prediccin o seleccione un conjunto de medidas que se deben tomar, se debe clasificar como un tem de aplicacin de conocimientos. Si un tem solamente evala la memoria de retencin de hechos aislados (sin requerir su aplicacin), se debe clasificar como un tem de retencin. Todos los tems deberan exigir la aplicacin de conocimientos de modo de permitir la evaluacin de la base de informacin del alumno y de la capacidad de usar esa informacin.

Captulo 4. Contenido de los tems: evaluacin de la aplicacin de conocimientos sobre ciencias bsicas

35

El siguiente par de enunciados ejemplifica la diferencia entre una pregunta que evala la retencin de un hecho aislado y una pregunta que evala la aplicacin de conocimientos. Enunciado de una pregunta de retencin sobre las ciencias bsicas: Qu regin recibe el suministro sanguneo a travs de la arteria cerebelosa inferior posterior? Enunciado de una pregunta de aplicacin de conocimientos sobre las ciencias bsicas: Un hombre de 62 aos de edad presenta ataxia de las extremidades del lado izquierdo, sndrome de Horner, nistagmo y prdida de las sensaciones de dolor y temperatura faciales. Qu arteria es ms probable que est obstruida?

Es comn usar las vietas clnicas como enunciados de tems con el fin de evaluar la aplicacin de conocimientos sobre las ciencias bsicas para interpretar situaciones clnicas. Por ejemplo, en lugar de pedir a los alumnos que identifiquen los msculos inervados por un par craneal, proporcione un conjunto de hallazgos fsicos y pida a los alumnos que identifiquen el lugar ms probable de la lesin. En vez de pedir una descripcin de la alcalosis o acidosis respiratorias, se pueden suministrar valores de los gases en sangre arterial (y otros hallazgos, si fuera necesario) y pedir a los alumnos que identifiquen la explicacin fisiopatolgica ms probable. Asegrese de que los alumnos puedan responder a la pregunta sobre la base de la comprensin que tengan de las ciencias bsicas; no debera ser necesario tener experiencia en la atencin mdica de pacientes. Las vietas de laboratorio tambin pueden ser tiles para preparar tems que evalan la aplicacin de conocimientos. Estos tems presentan experimentos de laboratorio y requieren que los alumnos usen su comprensin de los principios cientficos bsicos para predecir o explicar los resultados. Las vietas pueden describir experimentos clsicos en el rea de las ciencias bsicas o pueden involucrar situaciones hipotticas o menos conocidas. Dichos tems cambian efectivamente el objetivo de la evaluacin del conocimiento de hechos aislados hacia el uso de los principios de las ciencias bsicas para resolver problemas. El uso de las vietas de laboratorio y del paciente en la evaluacin de la aplicacin de conocimientos tiene varios beneficios. Primero, la validez aparente del examen aumenta considerablemente al usar tems de resolucin de problemas. Segundo, es ms probable que los tems se concentren en la informacin importante en lugar de la trivial. Tercero, ayuda a identificar a los alumnos que han memorizado una cantidad importante de informacin objetiva pero que no tienen capacidad para usar esa informacin de manera eficaz.

36

Pautas relacionadas con el contenido de tems sobre ciencias bsicas


Evale la aplicacin de conocimientos con el uso de vietas clnicas y experimentales Concentre los tems en conceptos y principios fundamentales que representen informacin esencial (sin acceso a las referencias) que todos los alumnos deben comprender Evale material que sea relevante para el aprendizaje en las prcticas clnicas, educacin mdica de postgrado y educacin posterior Evite los tems que solamente requieren de la retencin de hechos aislados Evite los temas esotricos o interesantes pero que no son fundamentales

Estos dos tems fueron redactados para evaluar el mismo tema. Le recomendamos que las preguntas no se escriban como el primer ejemplo sino como el segundo. La porfiria aguda intermitente es el resultado de un defecto en la va biosinttica A. del colgeno B. del corticosteroide C. del cido graso D. de la glucosa *E. del hem F. de la tiroxina (T4) Un hombre sano de 33 aos de edad tiene debilidad leve y episodios ocasionales de dolor abdominal severo y continuo con algunos calambres pero sin diarrea. Una ta y un primo han tenido episodios similares. Durante una crisis, su abdomen se distiende y disminuyen los ruidos intestinales. El examen neurolgico muestra debilidad leve en la parte superior de los brazos. Estos datos sugieren que existe una anomala en la va biosinttica A. del colgeno B. del corticosteroide C. del cido graso D. de la glucosa *E. del hem F. de la tiroxina (T4)

Captulo 4. Contenido de los tems: evaluacin de la aplicacin de conocimientos sobre ciencias bsicas

37

Plantillas de tems
La estructura general de un tem puede representarse por medio de una plantilla. Normalmente, se pueden elaborar muchos tems usando la misma plantilla. Por ejemplo, la siguiente plantilla podra usarse para elaborar una serie de preguntas relacionadas con la anatoma gruesa: (Descripcin del paciente) no puede (incapacidad funcional). Cul de las/los siguientes es ms probable que haya sufrido una lesin? Esta es una pregunta que podra haberse escrito con esta plantilla: Un hombre de 65 aos de edad tiene dificultades para levantarse cuando est sentado y para enderezar su tronco, pero no tiene inconvenientes para flexionar sus piernas. Cul de los siguientes msculos es ms probable que haya sufrido una lesin? *A. Glteo mayor B. Glteo menor C. Posterior del muslo D. Iliopsoas E. Obturador interno Numerosas preguntas sobre el rea de las ciencias bsicas se pueden presentar dentro del contexto de una vieta de paciente. sta puede incluir algunos o todos los componentes que figuran a continuacin: Edad, sexo (por ejemplo, hombre de 45 aos de edad) Lugar de atencin mdica (por ejemplo, llega al departamento de urgencias) Motivo de presentacin (por ejemplo, debido a un dolor de cabeza) Duracin (por ejemplo, constante desde hace 2 das) Historia clnica del paciente (incluir antecedentes familiares?) Determinaciones del examen fsico +/- Resultados de los estudios de diagnstico +/- Tratamiento inicial, hallazgos posteriores, etc.

38

Plantillas adicionales
(descripcin del paciente) tiene un (tipo de lesin y ubicacin). Cul de las siguientes estructuras es ms probable que est afectada? (descripcin del paciente) tiene (datos de la historia clnica) y toma (medicamentos). Cul de los siguientes medicamentos es ms probable que sea la causa de su (un hallazgo de laboratorio, examen fsico o historia clnica)? (descripcin del paciente) presenta (hallazgos anormales). Cul de los datos [complementarios] sugiere/sugerira un diagnstico de (enfermedad 1) en lugar de (enfermedad 2)? (descripcin del paciente) tiene (signos y sntomas). Estas observaciones sugieren que la enfermedad es el resultado de la (ausencia o presencia) de cul de las/los siguientes (enzimas, mecanismos)? (descripcin del paciente) sigue un (rgimen diettico especfico). Cul de las siguientes enfermedades es ms probable que se presente? (descripcin del paciente) presenta (sntomas, signos o una enfermedad especfica) y est bajo tratamiento con (medicamento o tipo de frmaco). Cul de las/los siguientes (funciones, procesos) se ve inhibida/o por la accin de este medicamento? (descripcin del paciente) presenta (hallazgos anormales). Cul de los siguientes (resultados positivos de laboratorio) se anticipara? (perodo de tiempo) despus de (evento, tal como un viaje o una comida con ciertos alimentos), (descripcin de paciente o grupo) se enferm y presenta (signos y sntomas). Cul de los siguientes (organismos, agentes) es ms probable que se encuentre en el anlisis del (alimento)? Luego de (procedimiento), (descripcin del paciente) desarrolla (signos y sntomas). Los hallazgos de laboratorio muestran (hallazgos). Cul de las siguientes opciones sera la causa ms probable? (descripcin del paciente) muere a causa de (enfermedad). Cul de los siguientes hallazgos sera el ms probable en una autopsia? Un paciente presenta (signos y sntomas). Cul de las siguientes opciones sera la explicacin ms probable de estos (hallazgos)? (descripcin del paciente) tiene (signos y sntomas). La exposicin a cul de los siguientes (agentes txicos) es la causa ms probable? Cul de los siguientes mecanismos es el ms probable en el efecto teraputico de esta (clase de frmaco) en pacientes que padecen de (enfermedad)? Un paciente tiene (hallazgos anormales) pero (hallazgos normales). Cul de las siguientes opciones sera el diagnstico ms probable? Si desea ms ejemplos, consulte el Apndice B.

Captulo 4. Contenido de los tems: evaluacin de la aplicacin de conocimientos sobre ciencias bsicas

39

Tipos de preguntas
Averige cul es mi medicamento Averige cul es mi exposicin txica Averige cul es mi dieta Averige cul es mi estado de nimo Prediga hallazgos fsicos Prediga hallazgos de laboratorio Prediga las secuelas Identifique las causas subyacentes/el diagnstico Identifique la causa de las respuestas a los medicamentos Identifique el medicamento que se debe administrar

Ejemplos de preguntas introductorias y listas de opciones


Cul de los siguientes es (anormal)? Los grupos de opciones pueden incluir ubicacin de las lesiones, lista de nervios, lista de msculos, lista de enzimas, lista de hormonas, tipos de clulas, lista de neurotransmisores, lista de toxinas, molculas, vasos sanguneos, segmentos de la columna vertebral. Cul de los siguientes hallazgos es el ms probable? Los grupos de opciones pueden incluir la lista de resultados de laboratorio, lista de signos fsicos adicionales, resultados de autopsias, resultados de exmenes microscpicos de lquidos, tejido muscular o articular, resultados de anlisis de ADN, niveles sricos. Cul de las siguientes opciones sera la causa ms probable? Los grupos de opciones pueden incluir la lista de mecanismos subyacentes de la enfermedad, medicamentos que podran causar efectos secundarios, frmacos o clase de frmacos, agentes txicos, mecanismos hemodinmicos, virus, defectos metablicos. Cul de las siguientes opciones debera administrarse? Los grupos de opciones pueden incluir medicamentos, vitaminas, aminocidos, enzimas, hormonas. Cul de los siguientes presenta un defecto/deficiencia/mal funcionamiento? Los grupos de opciones pueden incluir una lista de enzimas, mecanismos de retroalimentacin, estructuras endocrinas, elementos de nutricin, vitaminas. Segn el estudio genealgico, cul es la probabilidad de que el prximo hijo (especificar sexo) padezca la enfermedad?

40

Redaccin de las opciones: cmo modificar la dificultad de un tem


Las opciones incorrectas en cada pregunta se llaman distractores. Algunos alumnos deberan seleccionar cada una de las respuestas incorrectas; por lo tanto, cada distractor debe ser convincente y no debe destacarse como evidentemente incorrecto. Los errores comunes y un mal razonamiento proporcionan una fuente adecuada de distractores convincentes. Estos distractores afectan directamente la dificultad de la pregunta. Considere la pregunta que est a la derecha. Quin fue el autor principal de la Declaracin de la Independencia? A. Abraham Lincoln B. Thomas Jefferson C. Franklin Roosevelt D. El rey Jorge II E. Catalina la Grande

En el ejemplo anterior, las opciones son bastante divergentes y se puede identificar a Thomas Jefferson fcilmente como la respuesta correcta. Alguien que conozca relativamente poco de la historia estadounidense puede responder a esta pregunta correctamente. Ahora considere la misma pregunta con un grupo diferente de opciones. En este ejemplo, la pregunta es ms difcil; todas las opciones son respuestas convincentes para cualquiera que tenga un conocimiento limitado. Para algunas reas de contenido, las opciones como las del primer ejemplo pueden ser adecuadas; para otras reas, las del segundo ejemplo son ms adecuadas.

Quin fue el autor principal de la Declaracin de la Independencia? A. George Washington B. Thomas Jefferson C. Alexander Hamilton D. Benjamin Franklin E. James Madison

Cuando redacte los distractores, asegrese de que: tengan un contenido homogneo con respecto a la respuesta correcta (por ejemplo, que todas se refieran a diagnsticos o a prximos pasos en la atencin del paciente) sean incorrectas o inferiores con respecto a la respuesta correcta sean convincentes y atractivas para el alumno que no est informado sean similares a la respuesta correcta en cuanto a la construccin y extensin tengan correspondencia gramatical con el enunciado y que sean lgicamente compatibles con ste

Captulo 4. Contenido de los tems: evaluacin de la aplicacin de conocimientos sobre ciencias bsicas

41

La forma del tem


Un tem con una forma adecuada incluye la mayor cantidad de informacin posible en el enunciado, que debe ser relativamente extenso, mientras que las opciones deberan ser relativamente breves. El enunciado debera incluir todos los hechos relevantes; no se debe proporcionar datos adicionales en las opciones. Item que tiene una forma adecuada:

Enunciado extenso

A. B. C. Opciones breves D. E.

Item que tiene una forma deficiente:

Enunciado breve
A. B. C. Opciones extensas D. E.

42

Aprendizaje basado en problemas y el uso de un grupo de preguntas en base a un caso


Un nmero creciente de facultades de medicina han adoptado el aprendizaje basado en problemas (PBL, sigla en ingls de Problem Based Learning) como una estrategia institucional para parte del plan de estudio en ciencias bsicas. A pesar de que el enfoque de cada facultad con respecto al PBL es de algn modo distinto, todos implican el uso de casos escritos de pacientes (problemas) en la enseanza de las ciencias bsicas. Los problemas estn diseados para estimular el aprendizaje del material de las disciplinas cientficas bsicas tradicionales (p. ej., anatoma, fisiologa, bioqumica) desde una perspectiva clnica y se recalca la aplicacin de los principios de ciencias bsicas a las situaciones clnicas. El material normalmente se cubre a travs del estudio independiente y se analiza en grupos reducidos con un tutor de la facultad. Los cursos y los planes de estudio que aplican el PBL por lo general recalcan el proceso de aprendizaje, aprender a aprender, la responsabilidad de los alumnos por su propio aprendizaje y la preparacin para el aprendizaje durante toda la vida. No obstante, existen variaciones importantes entre los programas que tienen implicancias para la evaluacin. El mtodo de aprendizaje por descubrimiento autnomo pone nfasis en el proceso de aprendizaje: los alumnos tienen la responsabilidad de determinar lo que deben aprender, al igual que cmo y cundo hacerlo. Se considera que lo ms importante es aprender a aplicar principios amplios a situaciones de resolucin de problemas, con un aporte mnimo de gua por parte de los profesores y otorgando el mximo de oportunidad a la exploracin por parte de los alumnos. A diferencia de ste, en el mtodo de aprendizaje por descubrimiento guiado, los diseadores de planes de estudio identifican objetivos especficos de aprendizaje para cada problema y se proporcionan dichos objetivos a los profesores que los utilizan para organizar debates grupales y el aprendizaje del alumno. Estos planes de estudio se pueden estructurar de forma importante mediante la disposicin secuencial de las experiencias de enseanza. Los alumnos pueden o no identificar la estructura y los objetivos especficos: su experiencia puede ser muy similar a la de los alumnos de programas que usan el mtodo de aprendizaje por descubrimiento autnomo. En la prctica, los mtodos de aprendizaje por descubrimiento autnomo y guiado probablemente se describiran mejor como extremos opuestos de un continuo. Los programas varan a lo largo del continuo, y dentro de un programa, los problemas (y grupos) tambin varan. La evaluacin en los programas que usan el mtodo de aprendizaje por descubrimiento autnomo a menudo se concentra en las variables del proceso, como autonoma, motivacin, esfuerzo, resolucin de problemas y actitudes. La evaluacin de los resultados del aprendizaje es genuinamente problemtica porque se alienta a cada alumno para que siga un programa de estudios, de algn modo, diferente. Particularmente, el uso de los exmenes tradicionales de opcin mltiple puede ser considerado como inapropiado porque impulsa a los alumnos a que estudien para el examen y de esta manera, puede disuadirlos de la autodeterminacin del material que deben aprender y del proceso para aprenderlo. La evaluacin de los resultados del aprendizaje presenta menos problemas cuando se usa el mtodo de aprendizaje por descubrimiento guiado ya que los mismos objetivos de aprendizaje que guan el desarrollo y uso de problemas tambin pueden guiar el desarrollo del examen. Para lograr un equilibrio con los objetivos curriculares, la evaluacin debera concentrarse en

Captulo 4. Contenido de los tems: evaluacin de la aplicacin de conocimientos sobre ciencias bsicas

43

la comprensin por parte de los alumnos de los mecanismos bsicos de salud, enfermedad y tratamiento. Los exmenes bien redactados de opcin mltiple pueden tener una funcin destacada en la evaluacin siempre que recalquen la aplicacin de conocimientos de las ciencias bsicas a la atencin mdica de los pacientes. Los exmenes que usan grupos de preguntas en base a un caso es decir, preguntas de opcin mltiple asociadas con la misma presentacin del paciente son particularmente apropiados para los cursos de PBL. A continuacin se da un ejemplo de un grupo de preguntas en base a un caso. Consta de una breve presentacin de un caso seguida de una serie de tres preguntas de opcin mltiple. Cada pregunta se refiere a un aspecto de algn modo diferente del caso, considerando la situacin clnica desde diferentes perspectivas. Al igual que en el PBL, generalmente el uso de este tipo de material de examen recalca el aprendizaje de informacin sobre ciencias bsicas de manera que est organizada para ser til para la atencin mdica de un paciente. Una mujer de 34 aos de edad sufre de diarrea acuosa severa desde hace cuatro das. Hace dos meses tuvo mononucleosis infecciosa. Es drogadicta, usa drogas por va intravenosa y tiene anticuerpos contra el VIH en la sangre. El examen fsico muestra deshidratacin y debilidad muscular evidente. 1. Es ms probable que los anlisis de laboratorio muestren: A. disminucin de la concentracin de K+ en suero. B. disminucin de la concentracin de Ca2+ en suero. C. aumento de la concentracin de HCO3en suero. *D. aumento de la concentracin de Na+ en suero. E. aumento del pH srico. Para evaluar la causa de la diarrea, cul de las siguientes opciones es la ms adecuada? A. Biopsia de colon para identificar la presencia de Giardia lamblia B. Cultivo de material de la cavidad bucal para detectar Candida albicans C. Biopsia del duodeno para identificar la presencia de Entamoeba histolytica D. Aspiracin gstrica para identificar la presencia de Mycobacterium aviumintracellulare *E. Muestra de heces para identificar la presencia de Cryptosporidium 3. Otros estudios ms especficos que se realizaron para evaluar su infeccin por VIH muestran que la proporcin de linfocitos T cooperadores con respecto a los linfocitos T supresores es de 0.3. Esto se debe a que el VIH A. provoca la proliferacin de linfocitos T cooperadores B. provoca la proliferacin de linfocitos T supresores *C. infecta las clulas con receptores CD4 D. infecta los macrfagos E. estimula la sntesis de leucotrienos

2.

44

Adems de los principios que se describieron anteriormente en este manual, existen dos consideraciones adicionales que se deben tener en cuenta al preparar un grupo de preguntas en base a un caso: las pistas y la dependencia. Primero, es preferible evitar dar pistas, es decir, proporcionar datos para las respuestas a preguntas anteriores en preguntas posteriores. Es muy probable que los alumnos lean hacia adelante para tratar de encontrar estas pistas, por lo que los autores deben evitarlas. Por ejemplo, en un grupo que describe a un paciente con dolor en el pecho, si la primera pregunta se refiere a la causa ms probable del dolor y la segunda requiere de una seleccin del tratamiento farmacolgico ms adecuado, es importante que cada uno de los diagnsticos asociados con la primera pregunta incluyan un medicamento coincidente en la segunda (y viceversa); los alumnos que saben rendir exmenes descartarn los diagnsticos (y medicamentos) simplemente mediante la comparacin de las listas de opciones. Segundo, es preferible evitar la dependencia, es decir, redactar preguntas en las que los alumnos deben conocer la respuesta a una pregunta para poder responder a las otras, a menos que el tema que se evale sea tan importante que el redactor del tem desee que los alumnos reciban todos los puntos asociados con un grupo de preguntas, o ninguno. El grupo que se presenta en la siguiente pgina, preparado por los doctores David Felten y Ralph Jozefowicz para el examen final en la Universidad de Rochester del primer ao del curso de Ciencias Neurales, ejemplifica una estrategia para evitar la dependencia. Cada uno de los tres primeros tems se concentra en un aspecto diferente de la presentacin del paciente; por lo tanto es probable que los alumnos respondan correctamente a unos e incorrectamente a otros y reciban crditos parciales en caso de que su conocimiento sea parcial. La ltima pregunta depende levemente de las anteriores ya que requiere que el alumno comprenda todo el cuadro para responder correctamente; sin embargo, esto parece razonable cuando se considera la importancia de la ltima pregunta. Tal vez sea difcil para un profesor universitario solo preparar grupos de preguntas en los que los tems se refieren a varias disciplinas de las ciencias bsicas; esto requiere de importante amplitud de conocimientos. Una estrategia para enfrentar este problema es la de adoptar un mtodo de trabajo en equipo para la preparacin del material del examen, que sea semejante al que generalmente se utiliza para la preparacin de los problemas que se usan en el PBL. Por ejemplo, el mdico clnico de un equipo puede preparar la descripcin del paciente con la que comienza el grupo de preguntas, junto con las preguntas relacionadas con la fisiopatologa. Los profesores universitarios de disciplinas del rea de las ciencias bsicas pueden aportar preguntas que se concentren en distintos aspectos de la situacin del paciente desde la perspectiva de su disciplina. El uso de este tipo de material no est, obviamente, restringido al plan de estudio ni a los cursos que se ensean con el mtodo PBL. Es completamente adecuado cada vez que se desea recalcar la aplicacin clnica de la informacin relacionada con las ciencias bsicas a la enseanza, el aprendizaje y la evaluacin. Desde nuestro punto de vista, esto incluye la mayora de los cursos de ciencias bsicas, incluso aquellos que se ensean en primer ao. Como lo ilustra satisfactoriamente el ejemplo relacionado con el rea de las ciencias neurales de la pgina siguiente, es sencillo y adecuado evaluar el conocimiento bsico de anatoma y fisiologa en el contexto de la atencin mdica de un paciente en un curso de enseanza tradicional.

Captulo 4. Contenido de los tems: evaluacin de la aplicacin de conocimientos sobre ciencias bsicas

45

Una mujer de 58 aos de edad que no responde a los estmulos llega al departamento de urgencias luego de sufrir un desmayo en un centro de compras de la zona. Sus familiares informan que a la maana se haba sentido bien pero que haba sentido un dolor de cabeza que empeor progresivamente mientras ella estaba de compras. Ha tenido hipertensin y fibrilacin auricular, y toma un medicamento antihipertensivo y un anticoagulante oral. Su presin arterial es de 220/130 mm Hg y su patrn respiratorio es de apnea que se alterna con hiperpnea. La paciente responde solamente a estmulos nocivos con postura extensora con compromiso del brazo y de la pierna derechos. El examen del fondo de ojo revela la presencia de papiledema con compromiso del disco ptico izquierdo. Las pupilas presentan 3,0/7,0 (D/I) sin reaccin a la luz en la izquierda. Existe una desviacin de la mirada hacia la izquierda. Se observa hiperreflexia difusa (derecha mayor que en la izquierda) y signo de Babinski bilateral. 1. La pupila izquierda dilatada y sin reaccin concuerda ms con una lesin de A. nervio ptico izquierdo B. cintilla ptica izquierda *C. nervio motor ocular izquierdo D. ncleo geniculado lateral izquierdo E. tubrculo cuadrigmino superior izquierdo 2. La postura extensora en el lado derecho es ms compatible con una lesin en A. el telencfalo izquierdo B. el diencfalo izquierdo *C. el mesencfalo izquierdo D. la protuberancia izquierda E. bulbo raqudeo izquierdo 3. Su patrn de respiracin se describe mejor como A. normal *B. de Cheyne-Stokes C. de hiperventilacin neurognica central D. apnustico E. atxico

4. Cul de los siguientes sndromes de herniacin concuerda ms con la presentacin clnica de la paciente? A. Circunvolucin del cuerpo calloso debajo de la hoz *B. Uncus del lbulo temporal a travs de la tienda C. Diencfalo a travs de la hendidura tentorial D. Tronco enceflico a travs de la hendidura tentorial E. Amgdala cerebelosa a travs del agujero occipital

Se puede consultar un anlisis complementario de la evaluacin en los cursos y planes de estudio de PBL en:
Swanson DB, Case SM, and van der Vleuten CM. Strategies for student assessment. En: Boud, Feletti, eds. The Challenge of Problem-Based Learning - Second Edition. Londres: Kogan Page Ltd; 1997:269-282.

46

Ejemplos de tems para las ciencias bsicas


1. Varias clulas contiguas estn identificadas con una tincin fluorescente que no atraviesa las membranas celulares. Una clula se blanquea experimentalmente con luz que destruye la tincin pero inmediatamente recupera la fluorescencia de la tincin. La presencia de cul de las siguientes estructuras entre la clula blanqueada y las vecinas fluorescentes es la que explica mejor esta recuperacin? A. B. *C. D. E. Una lmina basal Desmosomas (mculas adherentes) Uniones intercelulares comunicantes Glucosaminoglicanos Uniones intercelulares hermticas (zonulae occludentes) 3. Durante una ciruga, se controlan PCO2 arterial y el pH arterial de un paciente bajo anestesia. Est conectado a un respirador mecnico y los valores iniciales son normales (PCO2 = 40 mm Hg; pH = 7,42). Si la ventilacin disminuye, cul de las siguientes ser la consecuencia ms probable? PCO2 arterial A. B. C. *D. E. F. Disminucin Disminucin Disminucin Aumento Aumento Aumento pH disminucin aumento sin cambios disminucin aumento sin cambios

2. Un hombre de 30 aos de edad presenta prdida de sensacin de temperatura y de dolor desde el cuello hacia abajo en el lado derecho del cuerpo y en el lado izquierdo de la cara; parlisis parcial del velo del paladar, la laringe y faringe en la izquierda; y ataxia en la izquierda. Este sndrome es probablemente el resultado de una trombosis de cul de las siguientes arterias? A. B. *C. D. E. Basilar Cerebelosa inferior posterior derecha Cerebelosa inferior posterior izquierda Cerebelosa superior derecha Cerebelosa superior izquierda

4. En la va metablica ramificada, una sola enzima diferente cataliza cada uno de los pasos individuales. La enzima que se anticipa que tendr una inhibicin ms marcada como consecuencia del compuesto V es la enzima A. *B. C. D. E. A B C D E

Captulo 4. Contenido de los tems: evaluacin de la aplicacin de conocimientos sobre ciencias bsicas

47

5.

Un paciente que padece cirrosis posthepatitis desarrolla un agrandamiento rpido del hgado asociado con el deterioro de la funcin heptica. La concentracin srica de cul de los siguientes es ms probable que sea anormal? A. B. C. *D. E. 1-Antitripsina Antgeno carcinoembrinico Gonadotropina corinica -Fetoprotena Gastrina

7. Los anlisis de laboratorio de un hombre edematoso de 35 aos de edad muestran una concentracin srica normal del complemento y un aumento en la concentracin srica del colesterol. El anlisis de orina muestra protena 4+, 0-5 eritrocitos/hpf y varios cilindros hialinos. Es ms probable que los anlisis de tejidos obtenidos de una biopsia renal muestren A. glomerulonefritis postestreptoccica (proliferativa) aguda B. glomerulonefritis membranoproliferativa *C. glomerulonefritis membranosa D. enfermedad con cambios mnimos (nefrosis lipoidea) E. glomerulonefritis rpidamente progresiva

6. El primer beb nacido de una mujer de 26 aos con Rh negativo que tuvo dos abortos previos en el segundo trimestre tiene insuficiencia circulatoria y hemlisis severa. Esta enfermedad podra haberse prevenido si se hubiera administrado a la madre tratamiento con A. IgG anti-RhD durante el embarazo ms reciente *B. IgG anti-RhD al terminar cada uno de los dos primeros embarazos C. IgM anti-RhD durante el embarazo ms reciente D. IgM anti-RhD al terminar el primer embarazo

8. Los genes en el cromosoma bacteriano tienen los siguientes ligamientos en la transferencia conjugativa: x e y, 25% de las veces; y e z, 50% de las veces. Si el orden del gen es x-y-z, qu porcentaje aproximado de las veces se transferirn x e z juntos? A. B. *C. D. E. 1% de las veces 5% de las veces 13% de las veces 20% de las veces 40% de las veces

48

9.

En una fiesta, el men inclua pollo frito, papas fritas caseras, arvejas, pasteles de chocolate y caf. Despus de 2 horas, la mayora de los comensales se sintieron muy enfermos y presentaron nauseas, vmitos y dolor estomacal. Cul de los siguientes organismos es ms probable que est presente en grandes cantidades en los anlisis de los alimentos contaminados? A. B. C. *D. E. Escherichia coli Proteus mirabilis Salmonella typhimurium Staphylococcus aureus Streptococcus faecalis

11. Un paciente que se presenta en el departamento de urgencias no sabe cul es el medicamento para el corazn que toma. Su ritmo cardaco es mayor a 80/min y los intervalos de PR y QRS en un ECG son prolongados. El paciente informa que siente un zumbido en los odos. Cul de los siguientes medicamentos es ms probable que haya tomado el paciente? A. B. C. D. *E. Digoxina Lidocana Fenitona Propranolol Quinidina

10. El frmaco Y tiene un volumen de distribucin (Vd) de 75 L, tanto en hombres jvenes como adultos ancianos. En los adultos ms jvenes, tiene un ndice de depuracin de 15 L/h; el 50% de sta se realiza a travs del hgado y el resto a travs de los riones. Para los hombres ms jvenes, el rgimen de mantenimiento es de 100 mg cada 6 horas. Cul de los siguientes regmenes producir esencialmente la misma concentracin en estado estable en un hombre de ms edad, cuya eliminacin de creatinina se reduce a la mitad de la de un hombre ms joven, pero que no presenta trastornos de la funcin heptica? A. *B. C. D. E. F. 75 mg cada 3 horas 75 mg cada 6 horas 75 mg cada 9 horas 100 mg cada 3 horas 100 mg cada 6 horas 100 mg cada 12 horas

12. Un nio de 8 aos de edad necesita que lo convenzan para ir a la escuela y a menudo, cuando est all, se queja de dolor intenso de cabeza o de estmago. Algunas veces, su madre tiene que llevarlo a casa debido a los sntomas. A la noche, trata de dormir con sus padres. Cuando ellos insisten en que duerma en su habitacin, les dice que hay monstruos en el armario. Cul de los siguientes diagnsticos es ms coherente con estos datos? A. Esquizofrenia de la niez B. Preocupaciones normales de nios de edad latente *C. Trastorno de ansiedad de separacin D. Trastorno de conducta socializada E. Psicosis simbitica

Captulo 4. Contenido de los tems: evaluacin de la aplicacin de conocimientos sobre ciencias bsicas

49

Captulo 5 Contenido de los tems: evaluacin de la aplicacin de conocimientos sobre ciencias clnicas
Mtodos de evaluacin
A pesar de la controversia continua sobre lo que es apropiado en los exmenes de opcin mltiple, todos los exmenes de tres pasos de la USMLE no dejan de incluir preguntas de opcin mltiple. En una bsqueda de mejores instrumentos de evaluacin, la NBME ha llevado a cabo una investigacin continua sobre otros formatos de evaluacin. Durante los ltimos 25 aos, el enfoque principal de esta investigacin ha sido el proyecto de Computer Based Examination (CBX, examen por computadora) actualmente conocido como Computer-based Case Simulations (CCS, simulacin de casos por computadora) que se introdujo como un componente del Paso 3 en 1999. Desde mediados de 1970, una segunda rea de investigacin se concentr en pacientes estandarizados (PE), cuya inclusin en la secuencia de los exmenes para la licenciatura en medicina se ha programado para 2004 2005. Al igual que con otros formatos de evaluacin ms autntica, los exmenes que se basan en PE y CCS aparentan tener ventajas significativas para la evaluacin de los distintos aspectos de la competencia clnica ya que implementan tareas para el alumno de una manera ms real. Otros proyectos se han concentrado en mejorar el formato de opcin mltiple. Como resultado de las investigaciones sobre el desarrollo de exmenes, las preguntas de opcin mltiple de la actualidad son muy diferentes de aquellas que se usaban en el pasado. Debido a razones psicomtricas y de contenido, los formatos de preguntas de verdadero/falso, tales como las de tipo K (verdadero/falso mltiples) y de tipo C (A, B, Ambas, Ninguna), ya no se usan en los exmenes de licenciatura en medicina. Si bien la mayora de las preguntas del Paso 2 tienen las cinco opciones tradicionales, tanto las preguntas de tipo A como las de emparejamiento ampliado pueden incluir un mximo de hasta 26 opciones, transformando as la tarea del alumno en algo ms cercano a una respuesta libre sin pistas. Virtualmente, cada tem del Paso 2 proporciona una vieta del paciente que se concentra en una tarea que es relevante para un interno nuevo, como la determinacin del diagnstico o del prximo paso en la atencin mdica del paciente. Estos tems requieren de la interpretacin y sntesis de los datos que son provistos y tambin de la aplicacin de conocimientos a situaciones conocidas o desconocidas (segn sea la experiencia del alumno). Al igual que los tems de casos basados en PE y CCS, los de opcin mltiple modernos encuadrados como vietas breves del paciente, presentan a los alumnos simulaciones de baja fidelidad de problemas que requieren la toma de decisiones mdicas.

Captulo 5. Contenido de los tems: evaluacin de la aplicacin de conocimientos sobre ciencias clnicas

51

Temas generales con respecto a lo que se debe evaluar


Existen varias tensiones que influyen en la construccin de cada Paso del examen USMLE que pueden ser importantes para usted al considerar lo que debe incluir en sus exmenes. Como lo establecen las declaraciones de objetivos, el sistema de exmenes USMLE est diseado para la licenciatura general; el Paso 2, por ejemplo, est diseado para evaluar la aplicacin de conocimientos requeridos para comenzar el primer ao de residencia luego de graduarse de la facultad de medicina, sin tener en cuenta la especialidad. El enfoque en el contenido que es necesario para la prctica en lugar de concentrarse explcitamente en los contenidos que han sido enseados (si fuera posible catalogar dicho cuerpo de informacin), significa que podra haber preguntas incluidas en el examen que evalan un conocimiento que no se ensea de manera uniforme en las facultades de medicina. Por otra parte, algunos temas que se ensean en algunas facultades de medicina podran quedar omitidos en el examen. En la realidad, existe una asociacin estrecha entre lo que se ensea y lo que se incluye en el examen, pero el examen est diseado para ser una evaluacin independiente de lo que los alumnos necesitan saber al comenzar su internado mdico, sin considerar si se les ensea o no. Se presenta una situacin similar para las distintas facultades al considerar la medida en que es apropiado incluir contenido en un examen que no se haya enseado explcitamente. Por ejemplo, es necesario que decida en qu medida desea que sus alumnos sean responsables del aprendizaje independiente adems de lo que ha sido tratado en las clases. Una segunda tensin en el Paso 2 se debe a su nfasis en los conocimientos que necesita tener un mdico general no diferenciado. Cuando los crticos han cuestionado si es apropiado un examen general en una poca de especializacin, refutamos estas expresiones indicando que la licenciatura en medicina es general y no una licenciatura especfica en una especialidad, lo que en consecuencia obliga a que los exmenes sean generales. La iniciativa generalista ha llevado a una disminucin de los comentarios en esta rea y nos han hecho aparecer como clarividentes; lo que no es una acusacin frecuente contra la NBME. Se plantea una situacin similar para las distintas facultades al considerar la medida en que las prcticas clnicas deberan concentrarse exclusivamente en su disciplina, sin un intento de integracin con otras disciplinas. Adems, los profesores deben decidir hasta qu punto las prcticas clnicas deben servir como una experiencia previa al internado mdico para los alumnos, en lugar de proporcionar un panorama general de los temas que son relevantes para todos los mdicos practicantes sin tener en cuenta la especialidad. El enfoque en preguntas que son adecuadas para todos los alumnos que terminan sus estudios, en lugar de aquellos que ingresan a especialidades especficas, ha tenido una influencia considerable en el contenido de las preguntas del Paso 2. Por ejemplo, es ms probable que el comit de ciruga redacte tems sobre situaciones de manejo de pacientes cuando la intervencin inmediata es fundamental, o casos en los que es importante seleccionar al paciente que necesita ir al piso quirrgico en lugar de recibir el alta del departamento de urgencias, que preguntas relacionadas con las tcnicas quirrgicas que no seran consideradas como de conocimiento fundamental para los que no son cirujanos. Es menos probable que el comit de obstetricia y ginecologa redacte tems sobre el manejo de embarazos de alto riesgo poco comunes (incluso si la mayora de los alumnos ha estado expuesta a estos casos durante las prcticas clnicas) que sobre el diagnstico de embarazos ectpicos, situacin que todo interno nuevo, sin importar la especialidad, debera reconocer. Es poco probable que las preguntas de psiquiatra se

52

refieran a un paciente psiquitrico hospitalizado, pero es posible que evalen la capacidad de diferenciar enfermedades psiquitricas de las que no lo son en pacientes que presentan comportamientos anormales. Todas las preguntas del Paso 2 deben ser aceptadas por todo el comit especfico de esa disciplina y, adems, por otro comit interdisciplinario. Esta revisin especfica a una disciplina seguida de otra interdisciplinaria ayuda a lograr el equilibrio en el contenido del examen. En una facultad, es importante que integrantes de otros departamentos revisen las preguntas del examen. Es probable que sea til que los integrantes del cuerpo docente universitario que no pertenezcan a un departamento revisen las preguntas del examen, aunque la logstica necesaria para implementar este procedimiento puede ser desalentadora.

Evaluacin de la retencin de hechos aislados o de la aplicacin de conocimientos


Adems de considerar los temas que es importante incluir en un examen, debe pensar sobre cmo estructurar esas preguntas para evitar evaluar solamente la retencin de hechos aislados. Tradicionalmente, las preguntas del examen se clasificaban segn los procesos cognitivos necesarios para responderlas, es decir, retencin, interpretacin o resolucin de problemas (memoria, comprensin y razonamiento). Las definiciones tpicas indican que las preguntas de retencin son aquellas que evalan el conocimiento que tiene el alumno de definiciones o de hechos aislados. Las preguntas de interpretacin requieren que el alumno revise algn tipo de informacin, con frecuencia en forma de grfico o tabla, y que llegue a una conclusin (por ejemplo, un diagnstico). Las preguntas de resolucin de problemas presentan una situacin en la que los alumnos deben tomar medidas (por ejemplo, decidir el siguiente paso en el tratamiento de un paciente). La dificultad que se presenta con estas clasificaciones es que los procesos cognitivos necesarios para responder a la pregunta dependen tanto de la formacin del alumno como del contenido de la pregunta. Los expertos en un rea de contenido quizs simplemente recuerden una respuesta sin pensarla mucho conscientemente, mientras que otros pueden necesitar razonar la respuesta a partir de principios bsicos. Los procesos cognitivos involucrados en la respuesta a una pregunta son especficos a cada alumno, por lo cual este enfoque taxonmico resulta difcil de usar. Un mtodo alternativo divide a las preguntas en dos categoras: la aplicacin de conocimientos o la retencin de hechos aislados. Si una pregunta requiere que el alumno llegue a una conclusin, haga una prediccin o seleccione un conjunto de medidas, se clasifica como una pregunta de aplicacin de conocimientos. Si una pregunta evala solamente la memoria pura de un hecho aislado (sin la necesidad de su aplicacin), se clasifica como una pregunta de retencin. Las preguntas que requieren que se recuerden hechos Cul de los siguientes hallazgos es ms probable que se aislados a menudo comienzan con la mencin de la observe en pacientes posquirrgicos con embolia pulmonar? enfermedad y luego preguntan sobre los hallazgos anticipados en el paciente. Estas preguntas estn estructuradas de forma similar en la mayora de los libros de texto; el alumno podra buscar la enfermedad y encontrar la respuesta en un solo prrafo. El defecto de estas preguntas es que parecen estar clnicamente al revs. Muy pocas veces, los pacientes informan a su mdico qu enfermedad tienen y luego le preguntan cules son los signos y sntomas.

Captulo 5. Contenido de los tems: evaluacin de la aplicacin de conocimientos sobre ciencias clnicas

53

Otro tipo de preguntas que establecen una tarea poco apropiada se conoce como preguntas de sala de espera. En ellas, se pide que el alumno seleccione entre cinco pacientes cul es el ms apropiado para realizar un cariotipo fetal, casi como si se le pidiera al alumno que realice un cariotipo fetal en una persona y para hacerlo slo tiene que observar la sala de espera y seleccionar al paciente ms apropiado.

Cul de las siguientes es una indicacin para realizar un cariotipo fetal a una mujer de 28 aos de edad? A. Edad paterna de 55 aos B. Higroma qustico fetal en un examen de ultrasonido C. Hijo anterior con espina bfida D. Prdida de embarazo previa de un feto triploide E. Trisoma 21 en el hermano de la mujer

En contraposicin a esto, la siguiente pregunta describe a una paciente y solicita que se indique cul es el estudio ms adecuado, una tarea ms razonable.

Una maestra sana de 28 aos de edad se encuentra en la semana 11 de gestacin. Los antecedentes familiares no presentan particularidades, excepto que sus dos hermanos padecen de retardo mental grave, su madre muri a los 55 aos a causa de un cncer de mama y su padre est distanciado. No estn disponibles las historias clnicas de la familia. Cul de los siguientes estudios es el adecuado? A. Anlisis de sangre para determinar la condicin de portador de X frgil B. Anlisis de sangre para determinar la condicin de portador de fenilcetonuria C. Muestreo de la vellosidad corinica para detectar distrofia muscular de Duchenne D. Muestreo de vellosidad corinica para el anlisis de cromosomas E. Amniocentesis para medir la -fetoprotena

Las preguntas que evalan la aplicacin de conocimientos estn estructuradas de una manera ms clnicamente real; por ejemplo, se proporcionan los hallazgos y se pregunta al alumno sobre la enfermedad subyacente. Normalmente, sera necesario que los alumnos puedan sintetizar la informacin de varias pginas de un libro de texto para responder a estas preguntas.

Un hombre de 62 aos de edad desarrolla dificultad respiratoria aguda y dolor pleurtico en el pecho 4 horas despus de haberse sometido a una colecistectoma. Cul de las siguientes opciones sera el diagnstico ms probable?

54

El uso de preguntas con vietas clnicas en el enunciado tiene varios beneficios. Primero la validez aparente del examen mejora en gran medida al usar preguntas que requieren que los alumnos resuelvan problemas clnicos. Segundo, es ms probable que las preguntas se concentren en la informacin importante en lugar de la trivial. Tercero, estas preguntas ayudan a identificar a esos alumnos que han memorizado una cantidad importante de informacin objetiva pero que no tienen una capacidad para usar esa informacin de manera eficaz en situaciones clnicas. Las preguntas que se basan en vietas clnicas son por lo general ms adecuadas para exmenes de nivel superior. En medicina, la redaccin de preguntas para la aplicacin de conocimientos es relativamente sencilla. Cuando se describe a un paciente y se redacta una pregunta referida a ese paciente, se evala la aplicacin de conocimientos. La nica instancia en la que el uso de una vieta clnica implica simplemente la retencin de un hecho aislado es si la vieta describe a un paciente exactamente igual a otro sobre el cual el alumno ley antes (por ejemplo, si se usa una vieta que est en un libro de texto o que fue analizada en clase).

Pautas relacionadas con el contenido de tems sobre ciencias clnicas


Evaluar la aplicacin de conocimientos por medio de las vietas clnicas para plantear decisiones mdicas en situaciones de atencin del paciente. Enfocarse en problemas potencialmente graves o comunes; evitar rompecabezas y temas esotricos. Presentar tareas que implican la toma de decisiones clnicas que un alumno exitoso debera poder hacer. Evitar situaciones clnicas que deberan ser tratadas por un (sub)especialista.

Las preguntas se concentran en tareas especficas que el alumno exitoso debe poder enfrentar en la siguiente etapa de la capacitacin (por ejemplo, determinar el diagnstico ms probable, indicar los anlisis complementarios de laboratorio que se deben pedir, formular el prximo paso en el tratamiento, predecir el hallazgo adicional ms probable). Para cada rea, los casos en los cuales se cometen errores con ms frecuencia deben convertirse en el tema de una pregunta.

Captulo 5. Contenido de los tems: evaluacin de la aplicacin de conocimientos sobre ciencias clnicas

55

Cmo redactar tems de seleccin de la mejor respuesta


Elaboracin del enunciado La gran mayora de las preguntas debe ser redactada sobre la base de una vieta clnica. El enunciado debera comenzar con la presentacin de un problema de un paciente. Luego, debera continuar con la historia clnica (que incluya la duracin de los signos y sntomas), los datos del examen fsico, resultados de los estudios de diagnstico, tratamiento inicial, hallazgos posteriores, etc. Las vietas pueden incluir solamente un subgrupo de esta informacin, pero se debera presentar la informacin en el orden especificado. El enunciado debe referirse a un solo problema formulado con claridad. La pregunta introductoria del enunciado debe plantear una pregunta clara para que el alumno pueda responderla sin necesidad de ver las opciones. Un componente esencial de una buena pregunta es que cumpla con la regla de ocultar las opciones. Ejemplo de un enunciado bien elaborado: Este enunciado proporciona suficiente informacin y puede responderse sin consultar las opciones. Un hombre de 52 aos de edad presenta aumento de disnea y tos con esputo purulento desde hace 2 das. Fuma un paquete de cigarrillos por da desde hace 30 aos. Su temperatura es de 37,2 C (99 F). Los ruidos respiratorios son distantes con algunos roncus y sibilancias. Su recuento de leucocitos es de 9000/mm3 con diferencial normal. La tincin de Gram del esputo muestra numerosos neutrfilos y diplococos gramnegativos. Las radiografas de trax revelan hiperinflacin. Cul de los siguientes es el diagnstico ms probable? Enunciado que evala hechos aislados: El enunciado a continuacin contiene informacin insuficiente; para poder responder a la pregunta, el alumno debe usar las opciones como marco de referencia. Cul de las siguientes opciones es correcta sobre la seudogota? Las vietas del paciente deben incluir algunos o todos los componentes indicados a continuacin en el orden indicado: Edad, sexo (por ejemplo, hombre de 45 aos de edad) Lugar de atencin mdica (por ejemplo, llega al departamento de urgencias) Motivo de presentacin (por ejemplo, debido a un dolor de cabeza) Duracin (por ejemplo, constante desde hace 2 das) Historia clnica del paciente (incluir antecedentes familiares?) Determinaciones del examen fsico +/- Resultados de los estudios de diagnstico +/- Tratamiento inicial, hallazgos posteriores, etc.

56

Asegrese de que el enunciado: se concentre en conceptos importantes en lugar de hechos triviales pueda responderse sin mirar las opciones incluya todos los hechos relevantes; no se deben proporcionar datos adicionales en las opciones no sea engaoso ni excesivamente complicado no est redactado en forma de oracin negativa (por ejemplo, evite usar excepto o no en la pregunta introductoria)

Puntos precisos sobre los enunciados de los tems


Uso de pacientes reales. Creemos que generalmente es mejor no tomar como referente de las preguntas de opcin mltiple a pacientes reales, particularmente en los exmenes para alumnos. Como regla general, los pacientes reales son demasiado complicados y los elementos que son complicados no son necesariamente importantes para la evaluacin. Como se observ anteriormente, a veces se adornan las preguntas (por ejemplo, con datos incidentales) pero no se introducen pistas falsas (es decir, informacin que intenta desviar a los alumnos de la respuesta correcta). Lamentablemente, los pacientes reales a veces tienen pistas falsas entre sus hallazgos. Uso de materiales de referencia. Creemos que es apropiado proporcionar informacin en una pregunta de examen si en la vida real, es probable que alguien consulte una fuente de referencia para obtener la informacin. Por ejemplo, en muchos casos creemos que es apropiado proporcionar una tabla con valores normales de laboratorio o un cuadro que muestre el cronograma recomendado para pruebas de deteccin de enfermedades o para vacunas. Obviamente, no se debe simplemente pedir a los alumnos que busquen informacin en la tabla que se proporciona; sin embargo, podra preguntar, por ejemplo, sobre cmo se vacunara a un nio de 6 aos que nunca las recibi. Uso de las palabras propias de un paciente o de un mdico. Generalmente, no consideramos til incluir las propias palabras del paciente, particularmente si la tarea del alumno es la de interpretar matices de la lengua que pueden estar afectados por el tono. Por otra parte, puede ser til pedirle al alumno que seleccione la respuesta ms adecuada del mdico al paciente mediante una pregunta al alumno que signifique elegir entre opciones redactadas como preguntas con final abierto, final cerrado o direccionadas. Pacientes que mienten. Consideramos que todos los pacientes representados en preguntas de opcin mltiple deben decir la verdad; de lo contrario, se debe proporcionar la interpretacin del mdico sobre el relato del paciente. Los mdicos utilizan varias pistas para determinar la veracidad de lo que dice un paciente y muchas de ellas no se pueden expresar de manera escrita. De esta manera, nuestras preguntas describiran el consumo de bebidas alcohlicas de un paciente como El paciente bebe 16 oz de cerveza con la cena todas las noches o bien La descripcin del paciente de su propio consumo de bebidas alcohlicas es contradictoria. No redactaramos algo ambiguo como El paciente dice que bebe solamente una botella de cerveza por noche.

Captulo 5. Contenido de los tems: evaluacin de la aplicacin de conocimientos sobre ciencias clnicas

57

Verbosidad, adornos y pistas falsas: mejoran el tem?


La mayora de los educadores recalca la importancia de redactar enunciados de preguntas que sean lo ms breves posibles, evitando la verbosidad (las palabras de ms), los adornos (material extrao) y las pistas falsas (informacin pensada para despistar al alumno). En oposicin de cierto modo a esta sugerencia, hemos hecho hincapi en el uso de las vietas clnicas para la redaccin de tems. Para el Paso 2 del USMLE, estas vietas constan de descripciones que tienen un prrafo de extensin y se refieren a situaciones clnicas generalmente seguidas de una pregunta relacionada con el diagnstico o el prximo paso en la atencin mdica del paciente. Dichas preguntas recalcan la aplicacin de conocimientos y requieren que los alumnos tomen decisiones clnicas en lugar de simplemente recordar hechos aislados. Estn diseadas para reflejar tareas de la vida real y desafan a los alumnos para que primero identifiquen los hallazgos que son importantes, y que luego integren esos datos en un diagnstico o decisin clnica. Dichas preguntas a menudo requieren de mltiples pasos en el proceso de razonamiento. Hemos descubierto que las preguntas que se elaboran a partir de las vietas tienden a tener menos errores tcnicos que las tpicas preguntas que no las usan, probablemente porque las vietas respetan una estructura estndar y presentan preguntas que son clnicamente naturales. A pesar de estas ventajas, algunas personas han cuestionado el uso de las vietas porque consideran que solamente contribuyen a que la pregunta sea ms engorrosa ya que disfrazan la pregunta real con adornos. Algunos alegan que se debe evitar completamente el uso de las vietas; mientras que otros estn a favor de usar vietas breves e incluir slo hallazgos positivos relevantes, expresados en pocas palabras; el resto opina que se deben usar vietas extensas que incluyen informacin ms completa que el alumno debe revisar y sintetizar. Se llevaron a cabo varios estudios1 para comparar las caractersticas psicomtricas de tems desarrollados en tres formatos: sin vieta, con vieta breve y con vieta extensa. La progresin se dise de modo de exigir niveles crecientes de interpretacin, anlisis y sntesis de los hallazgos (observe el ejemplo presentado a continuacin en los tres formatos). Como se esperaba, los tems se hacen cada vez ms difciles a medida que los datos se presentan en una forma menos interpretada; no obstante, las diferencias en la discriminacin no fueron estadsticamente significativas. Sin considerar los resultados psicomtricos combinados, consideramos que los tems que se basan en vietas son generalmente ms adecuados porque evalan la aplicacin de conocimientos a situaciones de pacientes y plantean desafos clnicos apropiados; dichos tems podran considerarse como simulaciones clnicas de baja fidelidad que mejoran la validez del contenido de los exmenes. Un tem elaborado en un formato que no contempla la vieta, normalmente se redacta desde una perspectiva desde arriba hacia abajo (es decir, dada la enfermedad, cules son los hallazgos asociados). Para un experto, los tems que se redactan de esta manera pueden parecer idnticos a los que se elaboran a partir de una vieta del paciente.
1Extrado

de: Case SM, Swanson DB, Becker DF. Verbosity, window dressing, and red herrings: do they make a better test item? Academic Medicine.
1996;71:528-530.

58

Las siguientes tres preguntas muestran que, para los alumnos de bajo rendimiento, las preguntas no son idnticas. Los tems a continuacin se incluyeron en el examen de licenciatura para los alumnos de medicina que estaban en el ltimo ao de estudio. La cuadrcula que est a la derecha de cada tem muestra el porcentaje de alumnos superiores e inferiores que seleccion cada opcin (los alumnos superiores son aquellos que tuvieron un puntaje superior al 80% en todo el examen; los inferiores tuvieron un puntaje inferior al 20%). Casi todos los alumnos del grupo superior (99%) y del inferior (90%) seleccionaron la opcin correcta en el formato sin vieta. Los formatos de vieta breve y extensa no fueron notablemente ms difciles para el grupo superior pero s lo fueron para los del grupo inferior; el 82% en el formato de vieta breve y el 66% en el formato de vieta extensa seleccionaron la respuesta correcta. Sin vieta La anomala renal ms probable en nios con sndrome nefrtico y funcin renal normal es A. glomerulonefritis postestreptoccica aguda B. sndrome urmico hemoltico *C. sndrome nefrtico con cambios mnimos D. sndrome nefrtico debido a glomeruloesclerosis focal y segmentaria E. Prpura de Schnlein-Henoch con nefritis A B C D Sup. 1 0 99 0 Inf. 8 1 90 1 E 0 0

Vieta breve Un nio de 2 aos de edad presenta edema desde hace 1 semana. Su presin arterial es de 100/60 mm Hg y presenta edema generalizado y ascitis. Las concentraciones en suero son: creatinina 0,4 mg/dL, albmina 1,4 g/dL y colesterol 569 mg/dL. El anlisis de orina presenta protena 4+ y no presenta sangre. El diagnstico ms probable es (se present la misma lista de opciones). A B C D Sup. 0 0 98 2 Inf. 6 2 82 9 E 0 1

Captulo 5. Contenido de los tems: evaluacin de la aplicacin de conocimientos sobre ciencias clnicas

59

Vieta extensa A B C D Un nio de 2 aos de edad de raza negra ha presentado ojos y tobillos inflamados en la ltima semana. La presin arterial es de 100/60 mm Hg, el pulso es de 110/min y la frecuencia respiratoria es de 28/min. Adems de la inflamacin de los ojos y edema con fvea 2+ en los tobillos, presenta distensin abdominal con onda lquida positiva. Las concentraciones en suero son: creatinina 0,4 mg/dL, albmina 1,4 g/dL y colesterol 569 mg/dL. El anlisis de orina presenta protena 4+ y no presenta sangre. (A continuacin se presenta la misma pregunta introductoria y opciones). Sup. 0 1 98 1 E 0

Inf. 10 9 66 10 5

Aunque esta ltima pregunta se denomina vieta extensa, es bastante breve si se compara con la mayora de las preguntas en el Paso 2 de USMLE. Consideramos que los recin graduados de la facultad de medicina necesitan demostrar su nivel de competencia para analizar la informacin del paciente, sintetizar los hallazgos importantes y llegar a una conclusin. Como resultado, nuestras preguntas tienden a tener una combinacin de hallazgos importantes y sin importancia. Por otra parte, algunas veces resumimos los hallazgos en una oracin como los antecedentes familiares no aportan datos.

60

Cmo redactar tems relacionados con las tareas del mdico


Cada rea requiere de un mtodo levemente diferente para la redaccin de los tems. Le proporcionamos algunos ejemplos de preguntas introductorias y otras sugerencias para guiarlo en la redaccin de los tems para cada tarea del mdico.

La salud y el mantenimiento de la salud


Los tems de esta rea temtica evalan la capacidad de evaluar los factores de riesgo, comprender los datos epidemiolgicos y aplicar medidas preventivas. Los tems sobre la salud y el mantenimiento de la salud por lo general se encuadran en una de las siguientes categoras: 1) pruebas de deteccin de enfermedades, 2) interferencia constructiva, 3) vacunas/medicina para viajes, y 4) intervenciones de urgencia. En general, comience el tem con una vieta clnica que describa al paciente. Adems de los datos de la historia clnica y del examen fsico, esta vieta puede incluir informacin sobre los antecedentes de vacunas, factores de riesgo y antecedentes familiares. La informacin sobre la comunidad puede ser relevante y, por lo tanto, puede incluirse; no obstante, la pregunta debe concentrarse en el paciente individual. Las preguntas NO deben concentrarse en la evaluacin directa de hechos aislados. Por ejemplo, evite preguntar sobre la causa principal de muerte en algunos subgrupos de poblaciones; debe concentrarse en la aplicacin de este conocimiento. Al preguntar sobre las vacunas o las pruebas de deteccin de enfermedades, considere la posibilidad de proporcionar una tabla de prcticas habituales para evitar la necesidad de memorizar recomendaciones contradictorias. Las siguientes preguntas introductorias son ejemplos de las que usan en esta categora: Cul de las siguientes vacunas se debera administrar en esta oportunidad? Cul de las siguientes pruebas de deteccin sera la ms adecuada? Cul de los siguientes anlisis podra haber predicho estos hallazgos? Cul de las siguientes intervenciones sera la ms adecuada? Este paciente tiene ms riesgos de contraer cul de las siguientes enfermedades? Cul de las siguientes opciones es ms probable que hubiese prevenido esta afeccin? Cul de las siguientes opciones sera el paso siguiente ms adecuado en el tratamiento para prevenir [morbilidad/mortalidad/incapacidad]? Cul de las siguientes opciones se debe recomendar para prevenir una incapacidad como resultado de esta lesin/afeccin? Cul de los siguientes tratamientos precoces es ms probable que hubiese prevenido esta afeccin del paciente? Cul de los siguientes suplementos es ms probable que hubiese prevenido esta afeccin?

Captulo 5. Contenido de los tems: evaluacin de la aplicacin de conocimientos sobre ciencias clnicas

61

Un adolescente de 15 aos de edad sufri dos episodios de shock anafilctico severo luego de sufrir picaduras de abejas. Cul de las siguientes intervenciones sera la ms adecuada? A. Administracin de corticosteroides durante el verano B. Terapia profilctica con antihistamnicos de largo plazo C. Vestimenta protectora *D. Desensibilizacin con extracto de veneno de abejas E. No dejar que el paciente salga de la casa durante los meses de verano

Una mujer de 33 aos de edad, grvida 1, para 1, tiene un parto espontneo de una beba de 2460 g (5 lb 7 oz) a las 38 semanas de gestacin. La recin nacida tiene hepatoesplenomegalia, conducto arterioso permeable y cataratas. Durante la octava semana de gestacin, la madre present una erupcin maculopapular, linfadenopata cervical, dolor de garganta y artralgias que se resolvieron espontneamente luego de 1 semana. El desarrollo prenatal posterior no present complicaciones. Cul de los siguientes anlisis durante el embarazo hubiera predicho los hallazgos en el feto? A. Amniocentesis para determinar el cariotipo B. Cultivo para detectar la presencia del virus del herpes simple *C. Valoraciones en serie de rubola D. Anlisis de orina para detectar la presencia del citomegalovirus E. Prueba VDRL para investigar enfermedades venreas

En un examen de deteccin de enfermedades de rutina se determina que una mujer sana de 75 aos de edad moderadamente activa presenta una concentracin de colesterol srico total de 208 mg/dL y una concentracin de colesterol HDL srico de 70 mg/dL. El electrocardiograma no presenta anomalas. Cul de las siguientes recomendaciones dietticas sera la ms adecuada? A. Disminucin de la ingesta de colesterol B. Disminucin de la ingesta de grasas saturadas C. Disminucin de la ingesta de carbohidratos simples D. Aumento de la ingesta de fibras *E. No realizar cambios en la dieta

Un hombre asintomtico de 33 aos de edad tiene una presin arterial de 166/112 mm Hg. Los niveles de electrolitos sricos se encuentran dentro de los valores normales. Es probable que un tratamiento antihipertensivo eficaz reduzca la probabilidad de desarrollar cul de las siguientes afecciones? A. Aneurisma artico B. Insuficiencia cardiaca congestiva C. Infarto de miocardio D. Insuficiencia renal *E. Hemorragia cerebral

62

Mecanismos de las enfermedades


Estos tems deben evaluar el conocimiento que tienen los alumnos sobre fisiopatologa en su sentido ms amplio, que incluye etiologa, patognesis, historia natural, curso clnico, hallazgos asociados, complicaciones, gravedad de la enfermedad, y efectos de las intervenciones teraputicas, sean o no intencionales. Las preguntas sobre los mecanismos deben encuadrarse en un contexto clnico. En general, comience los tems sobre el mecanismo de una enfermedad con una vieta clnica del paciente y sus signos, sntomas, historia clnica, hallazgos de laboratorio, etc. Luego, realice una pregunta como las que se presentan a continuacin: Cul de las siguientes opciones sera la explicacin ms probable de estos hallazgos? Cul de las siguientes opciones sera la ubicacin ms probable de la lesin del paciente? Cul de los siguientes patgenos sera el ms probable? Cul de los siguientes hallazgos es ms probable que experimente un aumento/disminucin? Cul de las siguientes opciones sera el hallazgo ms probable en una biopsia?

Una nia de 10 aos de edad presenta hematuria macroscpica 14 das luego de un dolor de garganta. Tiene una presin arterial de 170/100 mm Hg y edema 2+ pedal y pretibial. El nivel de nitrgeno ureico srico es de 3,2 mg/dL. Cul de las siguientes opciones sera la causa ms probable? A. Coartacin de la aorta B. Disminucin en la produccin del factor relajante derivado del endotelio C. Aumento de la produccin de aldosterona D. Aumento de la produccin de catecolamina *E. Volumen de expansin intravascular

Un hombre de 32 aos tiene secreciones uretrales purulentas. Un cultivo presenta Neisseria gonorrhoeae sensible a la penicilina. Una semana despus de terminar el tratamiento con penicilina, el paciente manifiesta una recidiva de las secreciones uretrales. Un cultivo presenta nuevamente Neisseria gonorrhoeae sensible a la penicilina. Tanto el paciente como su pareja sexual son VIH negativos. El examen fsico de la pareja sexual del paciente mostr la presencia de una fisura anal; el cultivo de la uretra no presenta N. gonorrhoeae. Cul de las siguientes opciones sera la causa ms probable de la recidiva de la infeccin uretral? A. Infeccin con virus del herpes concurrente B. Desarrollo de resistencia bacteriana C. Tratamiento inadecuado con penicilina *D. Nueva infeccin causada por la pareja

Captulo 5. Contenido de los tems: evaluacin de la aplicacin de conocimientos sobre ciencias clnicas

63

Diagnstico
Para esta categora, redacte tems que exijan que los alumnos interpreten la historia clnica, los hallazgos fsicos y los resultados de estudios de laboratorio, imgenes y otros, a fin de determinar el diagnstico ms probable (diagnstico diferencial) o el siguiente paso en el diagnstico (pruebas de diagnstico). Al redactar preguntas sobre esta rea, piense si desea evaluar la capacidad para integrar conocimientos entre las distintas prcticas clnicas. El tem clsico de diagnstico comienza con la descripcin del paciente (se incluye la edad, sexo, signos y sntomas con su duracin, historia clnica, hallazgos fsicos del examen, datos sobre los estudios de diagnstico y de laboratorio) y finaliza con una pregunta: Cul de las siguientes opciones sera el diagnstico ms probable? Cul de las siguientes opciones sera el paso siguiente ms adecuado para el diagnstico? Cul de los siguientes datos es ms probable que confirme el diagnstico? Un hombre de 52 aos de edad presenta aumento de disnea y tos con esputo purulento desde hace 2 das. Fuma un paquete de cigarrillos por da desde hace 30 aos. Su temperatura es de 37,2C (99F). Los ruidos respiratorios son distantes con algunos roncus y sibilancias. El recuento de leucocitos es de 9.000/mm3 con diferencial normal. La tincin de Gram del esputo muestra numerosos neutrfilos y diplococos gramnegativos. Las radiografas de trax revelan hiperinflacin. Cul de las siguientes opciones sera el diagnstico ms probable? A. Asma B. Bronquiectasia *C. Bronquitis D. Embolia pulmonar E. Neumona estreptoccica Una mujer de 28 aos de edad tiene palpitaciones aproximadamente una vez por semana con una duracin de 1 a 5 minutos, en forma de latidos cardiacos fuertes, rpidos y regulares. Los episodios comienzan y se detienen repentinamente y no estn asociados con malestar en el pecho ni disnea. No hay antecedentes de problemas cardiacos. La paciente bebe de dos a tres tazas de caf por da. Muy pocas veces consume bebidas alcohlicas y no fuma. Su presin arterial es de 120/88 mm Hg y el pulso es de 96/min y regular. Se observan intervalos de mirada fija y asinergia oculopalpebral. La glndula tiroides est firme y 1,5 veces ms grande que lo normal. Se observa un chasquido mesosistlico en el pex del corazn y un soplo sistlico precoz de grado 2/6 en el borde esternal superior izquierdo. El electrocardiograma es normal excepto por la evidencia de una taquicardia sinusal. Cul de las siguientes opciones sera el paso siguiente ms adecuado para el diagnstico? A. Monitoreo electrocardiogrfico ambulatorio B. Determinacin del nivel srico de la hormona estimulante de la tiroides C. Determinacin del nivel de catecolamina en la orina D. Ventriculografa nuclear MUGA E. Ecocardiografa

64

Tratamiento
Estos tems evalan los principios de la atencin mdica crnica y aguda en pacientes hospitalizados o ambulatorios. Cuando se redactan preguntas relacionadas con el tratamiento, es especialmente importante concentrarse en aspectos de la atencin que sean importantes para el nivel de prctica del alumno (supervisados, con supervisin limitada, de prctica independiente, subespecialista). Para el Paso 2, nos concentramos en aspectos de la atencin que seran adecuados para todos los residentes nuevos sin considerar la especialidad, y en la atencin mdica de urgencia en el medio de la noche cuando la ayuda tal vez no est disponible. Las preguntas abarcan decisiones de farmacoterapia, decisiones de tratamiento ms generales en donde las opciones incluyen una combinacin de farmacoterapia con otros tratamientos, y tambin se incluye una combinacin de tratamientos con otros aspectos, como las pruebas de diagnstico. Las preguntas que se deben hacer incluyen: Cul de las siguientes opciones sera el paso inicial o siguiente ms adecuado para la atencin del paciente? Cul de las siguientes opciones sera el tratamiento ms eficaz? Cul de las siguientes opciones sera la farmacoterapia ms adecuada? Cul de las siguientes opciones sera la prioridad ms importante en la atencin mdica de este paciente? (por ejemplo en el departamento de urgencias) Una mujer de 55 aos con cirrosis descompensada del hgado es hospitalizada. Recibe tratamiento con espironolactona, elixir de cloruro de potasio y furosemida. En este momento, apenas responde y est hipotensa sin distrs respiratorio. Tiene signos que coinciden con enfermedad heptica crnica, ascitis y edema perifrico leve. El ECG muestra un ritmo cardiaco lento (55/min) y regular, sin ondas P y un complejo QRS ancho y empastado que contina con ondas ST y T anchas y empastadas. Cul de las siguientes opciones debera administrarse por va endovenosa? A. Calcio B. Lidocana C. Magnesio D. Solucin salina al 0,9% *E. Potasio Un adolescente de 15 aos de edad, anteriormente sano, tiene dolor periumbilical espasmdico; luego de varias horas, el dolor se traslada hacia el cuadrante inferior derecho y se convierte en un dolor constante. Vomita varias veces y lo trasladan al departamento de urgencias. El abdomen es sensible a la palpacin profunda del cuadrante inferior derecho. Los hallazgos de las radiografas de trax y abdomen son normales. El recuento de leucocitos es de 15.000/mm3. El anlisis de orina muestra 3 leucocitos/hpf. Cul de las siguientes opciones sera el tratamiento inicial ms adecuado? A. Tratamiento de apoyo domiciliario; regreso de inmediato si el dolor aumenta B. Enema de bario C. Tomografa computarizada del abdomen D. Cistografa y pielografa endovenosas *E. Exploracin quirrgica del abdomen

Captulo 5. Contenido de los tems: evaluacin de la aplicacin de conocimientos sobre ciencias clnicas

65

Cmo redactar tems sobre temas difciles


Una de las creencias ms comunes es que muchas reas no se prestan para el formato de opcin mltiple. Hemos tenido bastante xito en la elaboracin de tems en muchas de estas reas y recomendamos la siguiente estrategia. 1. Luego de identificar un tema que le presenta dificultades, analice todas las fuentes de materiales para el examen y seleccione varias preguntas sobre este tema que usted considere aceptables. 2. Identifique las caractersticas clave de estas preguntas y trate de elaborar una plantilla con la que los profesores universitarios puedan redactar preguntas similares. 3. Para los temas en que no hay ejemplos de tems, piense en lo que desea evaluar. Vaya ms all del simple listado de las reas, designando tareas relacionadas con el tema que los alumnos de medicina deben saber realizar (es decir, cada componente de la lista debe incluir un verbo). Para ejemplificar este proceso, los siguientes prrafos proporcionan una idea general del proceso que usamos para redactar las preguntas del Paso 2 sobre tica Mdica y Jurisprudencia. El esquema del contenido incluye los siguientes temas: 1) consentimiento y consentimiento informado para el tratamiento (por ejemplo, informacin completa, tratamientos alternativos, riesgos y beneficios); 2) relacin mdico-paciente (por ejemplo, informar la verdad, confidencialidad, privacidad, autonoma, informes pblicos); 3) muerte y forma de morir (por ejemplo, diagnstico de muerte, sostn de vida, autopsia, donacin de rganos, eutanasia, suicidio); 4) temas relacionados con el nacimiento (por ejemplo, diagnstico prenatal, aborto, conflicto materno-fetal); y 5) temas relacionados con la investigacin (por ejemplo, consentimiento, placebos, conflicto de intereses, poblaciones vulnerables). Tpicamente los autores redactan preguntas como las siguientes: Cul de las siguientes es la definicin de consentimiento informado? O de lo contrario, definen un escenario y preguntan cul es el principio tico que el escenario muestra. ste es un ejemplo de una pregunta legal que se haca anteriormente.

Irrespetuosamente se hace referencia a estas preguntas como preguntas de a quin le importa.

Las bases legales que llevaron a la disminucin de las restricciones sobre el aborto en Estados Unidos pueden ligarse ms estrechamente a A. la legislacin federal B. un fallo de un tribunal federal C. leyes estatales D. fallos de juzgados estatales E. fallos de la AMA (sigla en ingls de la American Medical Association)

66

Examinamos el banco de tems y decidimos que estbamos menos interesados en saber si los alumnos saban o no las definiciones; queramos evaluar si podan aplicar los principios ticos en sus decisiones relacionadas con la atencin del paciente. Luego, convocamos a un grupo de redactores de tems que observaron los modelos y elaboraron nuevas preguntas para el examen. Todas las preguntas se basaron en una vieta del paciente y pedan que el alumno indique lo que deba hacer el mdico o que evale si las medidas adoptadas por el mdico que se indicaban en la vieta eran las adecuadas; ninguna pregunta se concentr en la definicin de trminos. Los siguientes son algunos ejemplos de tems que redactamos.

Una enfermera es internada para una apendicectoma en el centro mdico donde trabaja. Una semana despus del alta mdica, el asistente del administrador del hospital le pregunta al cirujano cul fue el diagnstico definitivo. Cul de las siguientes es la respuesta ms adecuada por parte del cirujano? A. Responder, porque acelera el manejo de los temas relacionados con el seguro en el centro mdico B. Responder, porque como empleado del centro mdico, el administrador tiene acceso a la informacin de los pacientes C. Responder debido a la posibilidad de difundir informacin errnea sobre el paciente D. Negarse a responder porque el administrador no es mdico *E. Negarse a responder porque la informacin es confidencial

Un nio de 8 aos que padece leucemia linfoblstica aguda experiment tres recidivas en los ltimos 2 aos. El nico tratamiento disponible es la quimioterapia experimental. Sin tratamiento, el nio no tiene posibilidades de sobrevivir por ms de 6 semanas; con el tratamiento, su pronstico es desconocido. Los padres no desean continuar con ningn tratamiento y prefieren llevar a su hijo a casa; el nio tambin quiere irse a su casa. Cul de las siguientes opciones sera la ms adecuada? A. Dar el alta mdica al nio en contra del asesoramiento mdico *B. Darle el alta mdica de rutina C. Pedir a un tribunal de justicia que dicte una orden de tratamiento D. Denunciar a los padres ante los servicios sociales por negligencia mdica

Captulo 5. Contenido de los tems: evaluacin de la aplicacin de conocimientos sobre ciencias clnicas

67

Seccin III Items de emparejamiento ampliado

La seccin III proporciona informacin sobre los tems de emparejamiento ampliado. El captulo 6 presenta el formato de estos tems en el cual se dan instrucciones a los alumnos para que seleccionen la mejor respuesta. El captulo 7 presenta el formato en el cual se dan instrucciones a los alumnos para que seleccionen una cantidad determinada de opciones, generalmente ms de una.

Captulo 6 Items de emparejamiento ampliado (tipo R)


Los tems de emparejamiento ampliado son tems de opcin mltiple organizados en conjuntos que usan una sola lista de opciones para todos los tems del conjunto. Un conjunto de emparejamiento ampliado elaborado correctamente incluye cuatro componentes: 1. un tema general; 2. una lista de opciones; 3. una oracin introductoria, y 4. al menos dos enunciados, como se ejemplifica a continuacin. Tema general: Opciones: Cansancio A. B. C. D. E. F. G. Leucemia aguda Anemia de enfermedad crnica Insuficiencia cardiaca congestiva Depresin Infeccin por el virus de Epstein-Barr Deficiencia de folato Deficiencia de glucosa 6 fosfato deshidrogenasa H. I. J. K. L. M. N. Esferocitosis hereditaria Hipotiroidismo Deficiencia de hierro Enfermedad de Lyme Anemia hemoltica microangioptica Tuberculosis miliar Deficiencia de vitamina B12 (cianocobalamina)

Oracin introductoria: Enunciados: 1.

Para cada paciente que padece cansancio, seleccione el diagnstico ms probable. Una joven de 19 aos de edad presenta cansancio, fiebre y dolor de garganta desde hace una semana. La fiebre es de 38,3C (101F); adems presenta linfadenopata cervical y esplenomegalia. Los estudios iniciales de laboratorio muestran un recuento de leucocitos de 5.000/mm3 (80% de linfocitos, muchos de ellos con caractersticas atpicas). La actividad de la aspartato aminotransferasa (AST, GOT) srica es de 200 U/L. La concentracin de bilirrubina srica y la actividad de la fosfatasa alcalina srica se encuentran dentro de los valores normales. Respuesta: E

2. Una adolescente de 15 aos de edad manifiesta cansancio y dolor de espalda desde hace dos semanas. Tiene moretones, reas de palidez y sensibilidad diseminados sobre el rea de las vrtebras y ambos fmures. El recuento sanguneo completo muestra una concentracin de hemoglobina de 7,0 g/dL, un recuento de leucocitos de 2.000/mm3 y un recuento plaquetario de 15.000/mm3. Respuesta: A

Captulo 6. Items de emparejamiento ampliado (tipo R)

71

Los tems de emparejamiento ampliado se redactan de manera diferente a los tems tradicionales de seleccin de la mejor respuesta. Normalmente, el tema general, la oracin introductoria y las opciones se escriben primero, y el enunciado se escribe en ltimo lugar. Por ejemplo, si desea redactar algunas preguntas relacionadas con el diagnstico de cansancio, debera comenzar por enumerar los diagnsticos que podran ser la causa del cansancio. Luego, debera redactar una vieta para todas (o para muchas) de las opciones de la lista. El ejemplo anterior incluye las vietas para la infeccin causada por el virus de Epstein-Barr y para leucemia aguda. Se pueden redactar tems adicionales para algunos de los diagnsticos restantes; para los que son comunes y que pueden tratarse, se debera preparar ms de un tem. Estas vietas de muestra tienen una extensin moderada; sin embargo, se pueden usar vietas ms breves y ms concentradas. Por otra parte, se podra desafiar a los alumnos para que identifiquen la informacin clave del diagnstico, que es entremezclada con hallazgos incidentales, a travs del uso de vietas ms extensas.

Cmo evitar errores al redactar tems de emparejamiento ampliado para su evaluacin


Los cuatro componentes (tema general, opciones, oracin introductoria y enunciados) son todos fundamentales para la construccin de un conjunto de emparejamiento ampliado de buena calidad. Los conjuntos que no tienen oracin introductoria (o que tienen una oracin introductoria no especfica, como por ejemplo, Vincule cada tem con la mejor opcin) NO se deben usar ya que generalmente plantean tareas ambiguas o incoherentes para los alumnos. El siguiente conjunto es defectivo. Las opciones son heterogneas, no hay oracin introductoria y es necesario leer las opciones para poder responder al enunciado. Las reglas que se deben seguir para los tems de emparejamiento ampliado son exactamente iguales a las que se usan con los tems de seleccin de la mejor respuesta.

Ejemplo de un conjunto de emparejamiento ampliado defectivo


A. B. C. D. E. F. G. H. 1. 2. 3. es cinetosis no tiene efectos en las personas aumentan indirectamente el CO2 causan la muerte aumento en la sensibilidad al olor es una disminucin en la visibilidad esttica, economa, salud productos de la combustin fsil I. J. K. L. M. N. O. estn completamente controlados ocasionan daos en ojos y plantas son insignificantes aumentan el riesgo de contraer cncer de piel no se puede controlar un exceso de enfermedades respiratorias agudas entre los nios se opone a la opinin pblica

Factores que las personas consideran al evaluar la calidad del aire El efecto principal de la materia particulada en el aire Los productos de la niebla fotoqumica

72

Despus de leer el enunciado del primer tem, los alumnos solamente tienen una idea muy superficial de sobre qu se trata la pregunta. En un intento por determinar la mejor respuesta, los alumnos tienen que decidir si la opcin es cinetosis es ms o menos verdadera que no tiene efectos en las personas. La tarea no se puede realizar. En estas circunstancias, a menos que la opcin sea absolutamente 100% verdadera o falsa, no puede ordenarse en la misma escala que las otras opciones. El enunciado en s no es claro; no se puede responder sin observar las opciones. Al igual que con los tems de seleccin de la mejor respuesta, los enunciados deben ser extensos y las opciones breves. Es NECESARIO que haya una oracin introductoria que determine la relacin entre el enunciado y las opciones. NO debe haber verbos en las opciones. La regla de ocultar las opciones se aplica tanto a los tems de emparejamiento ampliado como a los de seleccin de la mejor respuesta.

Captulo 6. Items de emparejamiento ampliado (tipo R)

73

Ejemplos de oraciones introductorias y temas para las listas de opciones


Las vietas de pacientes proporcionan una estructura ideal para los enunciados, no solamente en las ciencias clnicas sino tambin para evaluar el conocimiento de las ciencias bsicas. Las oraciones introductorias generalmente comienzan con una frase como, por ejemplo, Para cada uno de los siguientes pacientes. A menudo, los conjuntos se organizan sobre la base de una dolencia principal o algn otro factor que permita una frase de introduccin ms especfica, como Para cada uno de los siguientes pacientes que padecen cansancio o bien Para cada uno de los siguientes pacientes con deficiencia de enzimas. La segunda parte de la oracin introductoria describe la tarea y el conjunto de opciones: seleccione el diagnstico ms probable, seleccione la protena que tenga ms probabilidades de ser anormal. Los siguientes son algunos ejemplos adicionales de oraciones introductorias y algunos temas generales sugeridos para las listas de opciones. Para cada uno de los siguientes pacientes seleccione el/la [por ejemplo, nervio] que es ms probable que sea [anormal/defectuoso/deficiente/no funcione]. Los conjuntos de opciones pueden incluir una lista de nervios, lista de msculos, lista de enzimas, lista de hormonas, lista de protenas, lista de tipos de clulas, lista de neurotransmisores, lista de procesos patolgicos. Para cada uno de los siguientes pacientes, seleccione el [hallazgo] que se anticipara. Los grupos de opciones pueden incluir una lista de resultados de laboratorio, lista de signos fsicos adicionales, resultados de autopsias, resultados de exmenes microscpicos de lquidos, tejido muscular o articular, resultados de anlisis de ADN, niveles hormonales. Para cada uno de los siguientes pacientes, seleccione la [causa] ms probable. Los grupos de opciones pueden incluir una lista de mecanismos subyacentes de la enfermedad, medicamentos que podran causar efectos secundarios, lista de frmacos o clase de frmacos, agentes txicos, mecanismos hemodinmicos. Para cada uno de los siguientes pacientes, seleccione el [por ejemplo, medicamento] que se debera administrar. Los grupos de opciones pueden incluir una lista de medicamentos, vitaminas, aminocidos, enzimas, hormonas. Para cada uno de los siguientes pacientes con [dolencia principal], seleccione el diagnstico ms probable. Los conjuntos de opciones pueden incluir una lista de diagnsticos, que a menudo se organiza en funcin de la dolencia principal, como por ejemplo, enfermedades que causan dolor en el pecho o enfermedades que provocan fiebre. Para cada uno de los siguientes pacientes, seleccione el siguiente paso ms adecuado en la atencin mdica del paciente. Los grupos de opciones pueden incluir una lista de tratamientos farmacolgicos, anlisis de laboratorio, alternativas de disposicin; las opciones tambin podran contener un conjunto combinado de tratamientos y estudios complementarios para evaluar si el alumno sabe cundo se han reunido los datos suficientes.

74

Aspectos adicionales sobre las opciones para los conjuntos R


Generalmente, cualquier dato que pueda enumerarse en una lista puede servir como base de las opciones en un conjunto R. A continuacin se detallan algunos temas generales que se han usado como base para las listas de opciones. Arterias Nervios Msculos Aminocidos Pptidos Hormonas Enzimas Componentes celulares Tipos de clulas Componentes sanguneos Molculas Cariotipos Protenas Lpidos Patgenos/Bacterias/Hongos Virus Citoquinas Toxinas Vitaminas/Minerales Diagnsticos Tipos de tejido conectivo Estructuras anatmicas Estructuras endocrinas Neurotransmisores Anomalas metablicas Trastornos inmunolgicos Componentes del sistema motriz Estructuras cardiacas Organelas Anomalas congnitas Segmentos de la mdula espinal Componentes del sistema nervioso central Productos secretores Componentes de la matriz extracelular Alternativas de tratamiento Medicamentos/Tipos de medicamentos Procesos patolgicos Estados fisiopatolgicos Anomalas electrolticas Anlisis de diagnstico

Las opciones en la lista deben constar de una sola palabra o una frase muy breve. Adems, deben ser homogneas (todas deben ser sobre diagnstico, todas sobre tratamiento, todas sobre ubicaciones anatmicas, todas sobre vitaminas, etc.). Pueden ser reas marcadas en un grfico o una fotografa. Las opciones, especialmente aquellas que involucran valores de laboratorio, a menudo se expresan en forma tabular (ver el ejemplo de fisiologa). Incluya todas las opciones relevantes que sean apropiadas para los alumnos; las distinciones sutiles y los diagnsticos poco comunes pueden ser inapropiados. Para algunos temas generales, tres opciones son suficientes; para otros, podra necesitarse una lista de 26 opciones (una para cada letra del alfabeto ingls).

Captulo 6. Items de emparejamiento ampliado (tipo R)

75

Cmo redactar los enunciados de los tems


Las vietas de pacientes proporcionan una estructura ideal para los enunciados, tanto de los tems referidos a las ciencias bsicas como a las ciencias clnicas. En el rea de las ciencias clnicas, la vieta comnmente proporciona la edad del paciente, su sexo, dolencia principal y lugar de atencin mdica; tambin contiene la historia clnica personal y los antecedentes familiares (si son relevantes), luego la informacin proveniente del examen fsico y los datos de laboratorio (si se suministran). Segn sea el objetivo del conjunto, las vietas pueden ser presentaciones prototpicas breves o descripciones ms completas que desafan al alumno para que identifique la informacin clave. Generalmente, estos tems incluiran al menos la edad, sexo, dolencia principal y la historia clnica relevante del paciente. Los tems que evalan el conocimiento de las ciencias bsicas, particularmente para los cursos de primer ao de la facultad de medicina, pueden incluir descripciones prototpicas menos detalladas. La descripcin de cada paciente debera tener una estructura similar a las otras del conjunto. Por ejemplo, si un tem incluye datos de raza, etnia u ocupacin, se deben incluir estos datos en todos los dems. Lo mismo sucede con los datos de laboratorio que se mencionan en un tem; se deben incluir en todos los tems. Se sugiere no combinar los casos de adultos con los de pediatra en el mismo conjunto; muy a menudo solamente la edad proporciona demasiadas pistas y elimina un gran nmero de opciones del anlisis. Una ventaja de usar vietas de pacientes es que ayudan a garantizar que el contenido evala la aplicacin de conocimientos. Estos tems no deben parecerse a un crucigrama, en donde las opciones y los enunciados se componen de una sola palabra o de una frase breve. Evite la reconstruccin de esas preguntas a las que se enfrentaba en la escuela secundaria, en donde tena que trazar una lnea desde un elemento de la columna A hacia otra opcin coincidente en la columna B. Particularmente, es importante que los tems sean claros. No existen justificativos para redactarlos de manera engaosa; la lista de opciones ampliada dificulta los tems lo suficiente como para permitirle distinguir el alumno que sabe del que no sabe, sin recurrir a artimaas. Al igual que con las preguntas de tipo A correctamente redactadas, la regla de ocultar las opciones es primordial. Los alumnos que saben podrn elaborar una respuesta a la pregunta y luego encontrarla en la lista de opciones en orden alfabtico. Se debe preparar un tem para la mayora de las opciones; para las opciones comunes o importantes, se puede redactar ms de un tem. Para evitar el nfasis excesivo en un tema especfico cuando se elabora un examen que evala la capacidad general, se usarn todas las opciones pero solamente dos o tres tems; el resto de los tems se guardarn para exmenes posteriores. Por otra parte, si desea evaluar el conocimiento con ms profundidad en un nmero ms reducido de temas generales, se pueden incluir de 10 a 20 tems en cada conjunto y determinar un subtotal del puntaje para cada tema general. Cuando revise los tems, verifique que exista solamente una sola respuesta mejor para cada pregunta. Recuerde tambin que debe haber al menos cuatro distractores razonables para cada tem. Como control final, se recomienda que pida a sus colegas que revisen los tems (sin indicar cul es la respuesta correcta). Si ellos manifiestan alguna dificultad para determinar la respuesta correcta, modifique la lista de opciones o el tem para eliminar la ambigedad.

76

Ejemplos de enunciados buenos y malos que usan la misma lista de opciones


El siguiente es un buen conjunto de tems sobre la microbiologa. Las opciones son una lista homognea de patgenos, y es razonable incluir virus y bacterias. Contiene una oracin introductoria que presenta una tarea clara para el alumno. Los dos enunciados requieren que los alumnos apliquen sus conocimientos sobre la microbiologa bsica para averiguar la causa ms probable de la enfermedad de cada paciente. A. B. C. D. E. F. G. H. I. J. K. Adenovirus Aspergillus fumigatus Bacillus anthracis Candida albicans Chlamydia psittaci Coccidioides immitis Coronavirus Corynebacterium diphtheriae Coxiella burnetii Coxsackievirus Virus de Epstein-Barr L. M. N. O. P. Q. R. S. T. U. Haemophilus influenzae Histoplasma capsulatum Mycobacterium tuberculosis Mycoplasma pneumoniae Neisseria gonorrhoeae Neisseria meningitidis Pneumocystis carinii Rinovirus Streptococcus pneumoniae Streptococcus pyogenes (grupo A)

Para cada paciente con fiebre, seleccione el patgeno que sea la causa ms probable de su enfermedad. 1. Una nia de 7 aos de edad tiene fiebre elevada y dolor de garganta. Presenta enrojecimiento de la faringe, la amgdala derecha hinchada con exudados cremosos y linfadenopata submandibular dolorosa en el lado derecho. El cultivo de agar sangre de la garganta produce pequeas y numerosas colonias -hemolticas que se inhiben mediante bacitracina. Respuesta: U 2. Durante la ltima semana, un joven de 18 aos tuvo fiebre, dolor de garganta y malestar con amgdalas agrandadas en ambos lados, exudado amigdalino, linfadenopata cervical difusa y esplenomegalia. Se observa linfocitosis con presencia de linfocitos atpicos. La prueba de anticuerpos heterfilos es positiva. Respuesta: K El siguiente enunciado, desarrollado para el mismo conjunto, evala la retencin de hechos aislados en lugar de la aplicacin de conocimientos. Tiene el aspecto de una pregunta de crucigrama en lugar de ser una pregunta de un examen de la facultad de medicina. 3. Organismo grampositivo encapsulado que normalmente se desarrolla en pares o en cadenas cortas. Respuesta: T

Captulo 6. Items de emparejamiento ampliado (tipo R)

77

El siguiente conjunto incluye dos enunciados. El primero requiere que el alumno sintetice la informacin para determinar el diagnstico; el segundo solamente necesita la retencin de datos aislados. A. B. C. D. E. F. G. H. Vitamina A Vitamina B1 Vitamina B2 Vitamina B6 Vitamina C Vitamina D Vitamina E Vitamina K I. J. K. L. M. N. O. P. Biotina Cobre Folato Yodo Hierro Magnesio Niacina Zinc

Para cada paciente que tenga rasgos clnicos causados por anomalas metablicas, seleccione la vitamina o el mineral que es ms probable que est involucrado. 1. Un hombre viudo de 70 aos de edad tiene equimosis, petequias perifoliculares e inflamacin de las encas. Su alimentacin se compone mayormente de gaseosas y perros calientes. Respuesta: E 2. Involucrado en la sntesis del factor de coagulacin. Respuesta: H

78

El siguiente conjunto incluye dos enunciados. El primero requiere que el alumno sintetice la informacin para determinar el diagnstico de trisoma 21; el segundo proporciona esta informacin. Cuando se elaboran los enunciados, es necesario decidir el nivel de sntesis que se proporcionar. A. B. C. D. Comunicacin interauricular Coartacin de la aorta Transposicin completa de las grandes arterias Defecto del cojn endocrdico E. F. G. H. Conducto arterioso permeable Estenosis de la vlvula pulmonar Tetraloga de Fallot Comunicacin interventricular

Para cada paciente, seleccione la anomala cardiaca congnita ms probable. 1. Un beb nace a trmino con un peso de 3.090 gramos. El examen fsico muestra que el nio es hipotnico con succin dbil. No se observa cianosis. Presenta pliegues prominentes del epicanto, lengua grande y quinto dedo pequeo, con curvatura hacia el anular (clinodactilia) en ambas manos. Se observa un soplo holosistlico fuerte en todo el precordio y vibracin (thrill) palpable en el borde superior izquierdo del esternn, desde el segundo hasta el cuarto espacio intercostal. Respuesta: D 2. Un beb con trisoma 21 tiene hendiduras de las vlvulas auriculoventriculares derecha e izquierda; no se observa cianosis. Respuesta: D

Captulo 6. Items de emparejamiento ampliado (tipo R)

79

Este conjunto presenta una tarea que est clnicamente al revs. Al alumno se le indica una vacuna y se le pide que seleccione al mejor paciente para esta vacuna. Una tarea ms adecuada sera que se presente un paciente al alumno y que luego se le pregunte sobre el paso siguiente ms apropiado en la atencin mdica del paciente (por ejemplo, se puede describir un paciente en cada enunciado, usar la oracin introductoria: Para cada paciente, seleccione el siguiente paso ms apropiado en la atencin mdica y luego usar vacunas, por ejemplo, como opciones). Un segundo problema del conjunto es que no se proporciona informacin suficiente sobre cada paciente. Por ejemplo, el alumno necesitar conocer la historia de inmunizaciones de un paciente antes de decidir cules son las que se deben suministrar.
Ao de nacimiento Sexo Ocupacin Embarazo Enfermedades en la niez Historia clnica Alergias

A. B. C. D. E. F. G. H. I. J.

1980 1975 1970 1965 1960 1955 1950 1945 1940 1935

Masculino Femenino Femenino Femenino Masculino Femenino Femenino Masculino Masculino Femenino

Alumno Pintora Maestra Abogada Pintor Empleada Enfermera Ejecutivo Conductor Ama de casa

S S No No No No

Ninguna Ninguna Ninguna Ninguna Sarampin Paperas Varicela Sarampin Rubola No se conocen

Ninguna Ninguna Ninguna Ninguna Ninguna Diabetes Ninguna Hipertensin arterial Esplenectoma Gastritis

Productos con huevo Penicilina Ninguna Productos a base de gelatina Toxoide tetnico Ninguna Ninguna Ninguna Ninguna Sulfonamidas

Para cada vacuna, seleccione el perfil de paciente que represente su uso ms adecuado. 1. 2. Vacuna contra el sarampin Vacuna antimeningoccica

80

Resumen de los pasos para redactar tems de emparejamiento ampliado


1. Identifique el tema general para el conjunto. El tema puede ser una dolencia principal (por ejemplo, dolor en el pecho, cansancio), una situacin de disposicin (por ejemplo, admisin/alta del departamento de urgencias), una clase de medicamentos (por ejemplo, agentes antihipertensivos, antibiticos). 2. Redacte la oracin introductoria para el conjunto (por ejemplo, Para cada paciente que se describe a continuacin, seleccione el diagnstico ms probable). La oracin introductoria indica la relacin entre los enunciados y las opciones, y aclara la pregunta que se presenta a los alumnos. Es un componente esencial de un conjunto de emparejamiento ampliado. 3. Prepare la lista de opciones. La lista de opciones debe contener palabras nicas o frases muy breves. Enumere las opciones en orden alfabtico a menos que tengan un orden lgico. 4. Redacte los tems. Los tems que pertenecen a un mismo conjunto deben tener una estructura similar. Por lo general, las vietas de pacientes son ideales. 5. Revise los tems. Verifique que haya una sola respuesta mejor para cada tem. Recuerde que debe haber al menos cuatro distractores razonables para cada tem. Como control final, se recomienda que pida a sus colegas que revisen los tems (sin indicar cul es la respuesta correcta). Si ellos manifiestan alguna dificultad para determinar la respuesta correcta, modifique la lista de opciones o el tem para eliminar la ambigedad.

Puede encontrar ms informacin sobre la redaccin de tems de emparejamiento ampliado en:


Case SM, Swanson DB. Extended-matching items: a practical alternative to free-response questions. Teaching and Learning in Medicine. 1993;5(2):107-115. Case SM, Swanson DB, Woolliscroft JO. Assessment of diagnostic pattern recognition skills in medicine clerkships using a written test. En: Harden R, Hart I, Mulholland H, eds. Approaches to Assessment of Clinical Competence. Norwich, Inglaterra: Page Brothers; 1992:452-458.

Captulo 6. Items de emparejamiento ampliado (tipo R)

81

Ejemplos de conjuntos de emparejamiento ampliado


Ejemplo de un conjunto para anatoma
A. B. C. D. Arteria cerebral anterior izquierda Arteria cerebral anterior derecha Arteria cerebral media izquierda Arteria cerebral media derecha E. F. G. H. Arteria cerebral posterior izquierda Arteria cerebral posterior derecha Arterias lenticuloestriadas izquierdas Arterias lenticuloestriadas derechas

Para cada paciente con anomalas neurolgicas, seleccione la arteria que tenga ms probabilidades de estar comprometida. 1. Un hombre diestro de 72 aos de edad tiene debilidad e hiperreflexia en la extremidad inferior derecha, un reflejo plantar extensor en la derecha, fuerza normal en el brazo derecho y movimientos faciales normales. Respuesta: A 2. Un hombre diestro de 68 aos de edad tiene hemiparesia espstica derecha, reflejo plantar extensor en la derecha y parlisis de los dos tercios inferiores del rostro del lado derecho. El habla es fluida y la comprensin escrita y oral de las rdenes es normal. Respuesta: G

82

Ejemplo de un conjunto para farmacologa


A. B. C. D. E. F. G. H. I. Paracetamol Amiodarona Inhibidores de la ECA Aspirina Atenolol Bleomicina Citosina arabinosida Furosemida Metronidazol J. K. L. M. N. O. P. Q. R. cido nalidxico Nitrofurantona Penicilina Prednisona Procainamida Propranolol Sulfasalazina Tetraciclina Verapamilo

Para cada paciente, seleccione el medicamento que tenga ms probabilidades de haber causado las reacciones adversas. 1. Hace 5 meses, un hombre de 56 aos de edad con arritmias ventriculares recurrentes comenz a tomar un medicamento antiarrtmico. Ahora padece de disnea progresiva, tos y fiebre leve. Aument la tasa de sedimentacin de eritrocitos. La radiografa de trax muestra una neumona intersticial difusa. Los anlisis de la funcin pulmonar muestran una disminucin en la capacidad de difusin del monxido de carbono. Respuesta: B 2. Un hombre de 62 aos que padece enfermedad pulmonar obstructiva crnica comienza un tratamiento con un medicamento antihipertensivo. Dos semanas despus, presenta un marcado empeoramiento de la disnea y sibilancias claramente audibles. Respuesta: O

Captulo 6. Items de emparejamiento ampliado (tipo R)

83

Ejemplo de un conjunto para fisiologa: Rasgos clnicos (Qu hallazgos complementarios son probables?)
pH A. B. C. D. E. F. G. 7,15 7,15 7,30 7,40 7,50 7,50 7,50 PO2 mm Hg 98 98 56 100 100 100 56 PCO2 mm Hg 33 24 80 40 33 24 33 HCO3- mEq/L 11 8 38 25 25 18 25

Para cada paciente que se describe a continuacin, seleccione los hallazgos ms probables en los anlisis de gases en sangre arterial. 1. Un hombre de 22 aos con antecedentes de poliuria y polidipsia desde hace tres semanas, ha tenido nuseas, vmitos y disminucin de la sensibilidad durante las ltimas 12 horas. El anlisis de orina (con tira reactiva) revela glucosa 4+ y cetonas 4+. Respuesta: B 2. Una mujer de 25 aos de edad llega al departamento de urgencias 12 horas despus de un intento de suicidio. Ingiri aproximadamente 100 tabletas de aspirina de 500 mg. Respuesta: F

84

Ejemplo de un conjunto para diagnstico


A. B. C. D. Espondilitis anquilosante Infeccin en disco intervertebral Mieloma mltiple Dolor miofascial E. F. G. H. Osteoporosis Estenosis de la columna vertebral Espondillisis Tuberculosis de la columna vertebral

Para cada paciente que padece dolor de espalda, seleccione el diagnstico ms probable. 1. Un hombre de 26 aos manifiesta la aparicin insidiosa de lumbalgia y rigidez a la maana temprano. El dolor cambia de un lado a otro y ocasionalmente se irradia a las nalgas y la parte posterior de los muslos pero no llega debajo de las rodillas. El paciente tiene uvetis anterior aguda, sensibilidad difusa sacroilaca y en la parte inferior de la espalda, y restriccin de movimientos en las caderas. La tasa de sedimentacin de eritrocitos es de 40 mm/h; el anlisis de fijacin de ltex es negativo y se observa anemia hipoproliferativa leve. Respuesta: A 2. Doce horas despus de sufrir un choque desde atrs mientras conduca, una mujer de 28 aos tiene dolor poco claro en la zona lumbar y cervical asociado a dolor de cabeza y movilidad cervical restringida. Ahora, manifiesta ansiedad. Hay bandas de msculo tipo soga presentes en el rea lumbar y por encima de la nalga izquierda; las bandas son dolorosas. Respuesta: D Otros tems adicionales cubriran algunos de los diagnsticos restantes. Estos ejemplos de vietas tienen una extensin promedio; sin embargo, se pueden usar vietas ms breves y ms concentradas. Por otra parte, se podra desafiar a los alumnos para que identifiquen la informacin clave del diagnstico usando vietas ms extensas.

Captulo 6. Items de emparejamiento ampliado (tipo R)

85

Ejemplo de un conjunto para diagnstico


A. B. C. D. E. F. G. H. I. J. Aneurisma abdominal Apendicitis Obstruccin intestinal Colecistitis Cncer de colon Estreimiento Diverticulitis Ruptura de embarazo ectpico Endometriosis Hernia K. L. M. N. O. P. Q. R. S. T. Clculo renal Adenitis mesentrica Trombosis de la arteria mesentrica Ruptura de quiste ovrico Pancreatitis Enfermedad inflamatoria plvica lcera pptica lcera pptica perforada Pielonefritis Torsin

Para cada paciente que padece dolor abdominal, seleccione el diagnstico ms probable. 1. Una mujer de 25 aos tiene un inicio repentino de dolor abdominal persistente en la regin inferior derecha que se agrava cada vez ms. Tiene nuseas sin vmitos. Sus deposiciones eran normales justo antes del comienzo del dolor. Los exmenes muestran una sensibilidad profunda muy intensa a la palpacin en la regin inferior derecha del abdomen con contractura pero sin rebote; hay ruidos intestinales presentes. El examen plvico muestra una masa de 7 cm ubicada en la regin derecha con sensibilidad muy intensa. El hematocrito es de 32%. El recuento de leucocitos es de 18.000/mm3. La actividad de la amilasa srica est dentro de los parmetros normales. El anlisis de las heces para determinar presencia de sangre oculta es negativo. Respuesta: B 2. Un hombre de 84 aos de edad que est en un hogar de ancianos tiene un dolor abdominal en la regin inferior con mala localizacin cada vez ms fuerte y que se repite cada 3 4 horas desde hace 3 das. No tiene nuseas ni vmitos; no se registr la ltima deposicin. El examen fsico muestra un abdomen blando con una masa palpable, levemente sensible en la regin inferior izquierda. El hematocrito es de 28%. El recuento de leucocitos es de 10.000/mm3. La actividad de la amilasa srica est dentro de los parmetros normales. El anlisis de las heces para determinar presencia de sangre oculta es positivo. Respuesta: E

86

Ejemplo de un conjunto para tratamiento: Disposicin


A. B. C. D. E. F. Observar en el departamento de urgencias Admitir para ciruga Admitir para tratamiento mdico Admitir para endoscopia Admitir para laparoscopia Pedir estudios de contraste G. H. I. J. K. Pedir IRM Pedir tomografa computada Pedir ultrasonografa Enviar a su domicilio con analgsicos indicados Enviar a su domicilio con indicaciones de seguimiento por parte de su mdico personal

Para cada uno de los siguientes pacientes, seleccione el siguiente paso ms adecuado en la atencin mdica del paciente. Los tems pueden describir pacientes con apendicitis, embarazo ectpico, endometriosis, enfermedad de Crohn, diverticulitis, absceso plvico, crisis drepanoctica, litiasis renal, quiste ovrico torcido u otros trastornos que comnmente se presentan en las salas de urgencias. Otros conjuntos de disposicin pueden concentrarse en el triage por telfono, decisiones de derivacin/alta hospitalaria, etc.

Captulo 6. Items de emparejamiento ampliado (tipo R)

87

Ejemplo de un conjunto para tratamiento: Pruebas de diagnstico


A. B. C. D. E. F. Anlisis de las heces para determinar presencia de sangre oculta Nivel de glucemia en ayunas Nivel de hemoglobina Nivel de antgeno prosttico especfico Nivel de colesterol srico Nivel de hierro srico G. H. I. J. K. L. M. Anlisis de funcin tiroidea Prueba de tolerancia de ejercicio Examen digital de prstata ECG Espirometra Radiografa de trax Sigmoidoscopia

Para cada paciente que visita al mdico por un examen de mantenimiento de la salud, seleccione el estudio de diagnstico ms apropiado. 1. Un hombre de 22 aos que pesa 89 kg (196 libras) y mide 1,75 m (69 pulgadas) de altura fuma un paquete de cigarrillos por da desde hace 8 aos; no realiza ejercicios fsicos. Su ltimo examen fue hace 5 aos. Su padre tuvo un infarto de miocardio a la edad de 48 aos. El examen fsico no presenta anomalas. Respuesta: E 2. Una mujer de 28 aos que pesa 70 kg (154 libras) y mide 1,73 m (68 pulgadas) de altura fuma un paquete de cigarrillos por da desde hace 12 aos; no realiza ejercicios fsicos. Su ltimo examen fue hace 5 aos, aunque tuvo una prueba de Papanicolaou hace 9 meses que mostr resultados normales. Su padre tuvo un infarto de miocardio a la edad de 48 aos. A su abuela se le diagnostic un cncer de colon a la edad de 62 aos. El examen fsico no presenta anomalas. Respuesta: E

Ejemplo de una lista de opciones para anomalas electrolticas


A. B. C. D. Hipocalcemia Hipocaliemia Hipomagnesemia Hiponatremia E. F. G. H. Hipercalcemia Hipercaliemia Hipermagnesemia Hipernatremia

Para cada uno de los siguientes pacientes, seleccione la anomala electroltica que es ms probable que est presente.

88

Ejemplo de una lista de opciones para las ciencias del comportamiento/pediatra


Habilidades de lenguaje/cognitivas A. B. C. D. E. F. G. H. Normal Normal Normal Normal Tarda Tarda Tarda Tarda Habilidades motrices gruesas Normal Normal Tarda Tarda Normal Normal Tarda Tarda Habilidades sociales Normal Tarda Normal Tarda Normal Tarda Normal Tarda

Para cada nio, seleccione la mejor descripcin del desarrollo.

Ejemplo de una lista de opciones para la nutricin/bioqumica


A. B. C. D. E. F. G. H. Vitamina A Vitamina B1 (tiamina) Vitamina B2 (riboflavina) Vitamina B6 Vitamina C Vitamina D Vitamina E Vitamina K I. J. K. L. M. N. O. P. Folato Biotina Niacina Hierro Magnesio Cobre Zinc Yodo

Para cada nio que tenga anomalas metablicas, seleccione la vitamina o el mineral que es ms probable que est comprometido. Si desea ms ejemplos, consulte el Apndice B.

Captulo 6. Items de emparejamiento ampliado (tipo R)

89

Pasos para organizar un grupo que redacte tems tipo R de contenido clnico
Se pueden seguir los siguientes pasos al utilizar un grupo para que redacte conjuntos de tems tipo R (de emparejamiento ampliado) clnicos. Algunos grupos se reunieron durante la cena, siguieron los pasos que se detallan a continuacin y generaron un primer borrador de una docena o ms de tems por cada participante de la reunin. Otros programaron este evento como un retiro de da completo fuera del campus universitario con el objetivo de generar un grupo de tems casi finales. El organizador de la fiesta de redaccin de tems debe tener en cuenta de qu manera se puede aprovechar mejor el tiempo disponible de los participantes. El rendimiento ser sustancialmente mayor tanto en cuanto a la calidad como a la cantidad siempre que se realicen trabajos previos. Por ejemplo, el rendimiento ser significativamente menor si los participantes tienen que decidir los temas sobre los que debern redactar tems; se ahorrar tiempo si los temas de cada grupo se determinan antes de la reunin (paso 1 a continuacin). Del mismo modo, se elaborarn ms tems si se cuenta de antemano con un borrador de las opciones para cada conjunto (paso 3 a continuacin). Lo mismo suceder si se redacta un ejemplo de tem que sirva como modelo para cada conjunto (paso 4 a continuacin). Hay casos en los que es mejor dar bastante flexibilidad a los redactores de los tems al determinar lo que deben redactar. En estos casos, puede ser viable el proporcionar uno o ms conjuntos de opciones y permitirles generar uno o ms conjuntos de opciones propias. Tambin ser necesario que se decida sobre la composicin de las parejas de redactores de tems. En algunas ocasiones, ser mejor permitir que los participantes elijan ellos mismos su pareja. En otras circunstancias, ser ms provechoso asignar las parejas. Hemos tenido ms xito cuando reunimos parejas de redaccin de tems con intereses similares e incluimos como revisores a personas con diversas especializaciones. Si el objetivo es generar tems interdisciplinarios para un examen, se debe proporcionar la misma lista de opciones a las diferentes parejas de redactores. Por ejemplo, las opciones relacionadas con el dolor abdominal pueden entregarse a un par de gineclogos, un par de cirujanos o un par de mdicos internistas. Cuando se elabora un examen, se puede incluir uno o ms tems de cada disciplina en el mismo conjunto para que los alumnos tengan en cuenta todas las disciplinas al momento de determinar el diagnstico ms probable de los pacientes. Se puede ahorrar mucho tiempo si los participantes escriben los tems en computadoras. Esto hace posible aprovechar muchas horas que se pierden cuando se trata de descifrar la letra de los redactores. Normalmente, tenemos dos redactores por cada computadora (hemos descubierto que automticamente uno de los dos toma el control del teclado). Luego, existen varios mtodos posibles para la revisin: los revisores pueden reunirse alrededor de la computadora para leer las preguntas en la pantalla; el autor puede leerlas en voz alta para los revisores que no tienen una copia de los tems; o tambin se pueden imprimir copias para que todos las usen en la sesin de revisin.

90

1. Definir el campo del contenido del examen. Por ejemplo, cuando se elabora un examen para evaluar la capacidad de diagnosticar problemas clnicos comunes, debe definir el campo mediante una lista de las dolencias principales. Se ahorrar tiempo si esto se realiza antes de la reunin de redaccin de tems. Masa abdominal Dolor abdominal Anemia Ascitis Dolor de espalda Dolor en el pecho Confusin Tos Retardo en el desarrollo Diarrea Mareos Aparicin fcil de hematomas Cansancio Fiebre Hemorragias gastrointestinales Dolor de cabeza Hematemesis/Melena Picazn Ictericia Dolor articular Linfadenopata Anomalas en los movimientos Nuseas Palpitaciones Disfuncin sexual Disnea Lesin en la piel Sncope Cambio de peso Sibilancias

2. Capacitar a un grupo de docentes para que sean redactores de tems. La capacitacin debera incluir un anlisis breve del objetivo del examen, algunos ejemplos de tems y los procedimientos que se deben seguir durante la redaccin. 3. Dividir al grupo en parejas para redactar los tems. A cada pareja se le asignar de 2 a 4 dolencias principales para las que debe redactar tems. Las parejas generarn (o modificarn) una lista de diagnsticos para cada dolencia asignada; asimismo redactarn una o ms descripciones de pacientes para los diagnsticos que hayan incluido en la lista de opciones. Se esperar que cada pareja redacte entre 20 y 60 enunciados (de 10 a 20 por dolencia). El uso de equipos de computacin ahorrar mucho tiempo a la larga.

Captulo 6. Items de emparejamiento ampliado (tipo R)

91

4. Recalcar las siguientes pautas para la redaccin de los enunciados. Cada tem debera describir a un paciente con uno de los diagnsticos en la lista de opciones. La descripcin deber comenzar con la edad del paciente, sexo, dolencia principal y lugar de atencin mdica; tambin deber contener la historia clnica personal, antecedentes familiares (si son relevantes), la informacin sobre los exmenes fsicos, y los datos de laboratorio (si se suministran). Segn cul sea el objetivo del conjunto, las vietas pueden ser presentaciones prototpicas breves o descripciones ms completas que desafen al alumno para que identifique la informacin clave. La descripcin de cada paciente debera tener una estructura similar a la de las dems del conjunto. Por ejemplo, si un tem incluye datos de raza, etnia u ocupacin, se deben incluir estos datos en todos los dems; lo mismo sucede con los datos de laboratorio que se mencionan en un tem; se deben incluir en todos los tems. 5. Unir las parejas de redactores en un grupo ms grande cuyo objetivo sea revisar los tems. Un mtodo de revisin es que el autor lea el tem en voz alta y otros intenten proporcionar la respuesta correcta. El grupo debe revisar la lista de opciones y modificar el tem o la lista de opciones para eliminar toda ambigedad posible. Ms arriba se describen otros mtodos. 6. Escribir a mquina, corregir y enviar los tems para su revisin por parte de terceros. Una vez que los tems ya tengan el formato final, se deben revisar sin incluir la respuesta correcta. 7. Elaborar el examen. Seleccione una muestra de tems para cada dolencia; guarde el resto para exmenes posteriores. Los tems pueden convertirse en tems del tipo de seleccin de la mejor respuesta si se les agrega una pregunta introductoria y las mejores cinco (o ms) opciones de la lista.

92

Formulario para redactar los conjuntos tipo R


Tema general: _________________________________________________________________ (por ejemplo, la dolencia que presenta el paciente) Oracin introductoria:__________________________________________________________________ (por ejemplo, Para cada paciente con fiebre, seleccione el diagnstico ms probable.)

Opciones A B C D E F G H I J K L M N O P Q R S T U V W X Y Z

(Escriba los tems en hojas por separado)

Captulo 6. Items de emparejamiento ampliado (tipo R)

93

Muestra de cdigo SPSSX para determinar el puntaje de los exmenes de opcin mltiple que incluyen tems de emparejamiento ampliado
El siguiente cdigo SPSSX se puede usar como modelo para otorgar el puntaje a un examen hipottico que incluya hasta 100 tems de opcin mltiple; cada tem puede tener hasta 26 opciones pero solamente una respuesta correcta. Es sencillo modificar el cdigo para cualquier extensin de examen. Se supone que: la clave de respuestas est en un archivo llamado KEY.DAT (formato: cdigo de identificacin del examen de ocho caracteres, seguido de un espacio y las 100 respuestas correctas); las respuestas de los alumnos estn en un archivo llamado RESPONSE.DAT (formato: el nmero de seguro social o cualquier cdigo de identificacin del alumno que tenga nueve dgitos, un espacio, el cdigo de identificacin del examen de ocho caracteres, un espacio, las respuestas del alumno a las 100 preguntas); el resultado del anlisis de los tems muestra la distribucin de las respuestas a cada pregunta, adems de un coeficiente de confianza (coeficiente alfa); y los informes de puntaje con el nmero de identificacin del alumno, adems del puntaje expresado como porcentaje correcto y puntaje estndar (ubicado en el archivo llamado REPORT.LIS). REPORT.LIS se puede importar en un procesador de textos para mejorar la presentacin.

Para los usuarios avanzados de SPSSX, el cdigo debera ser fcil de entender. SPSSX proporciona servicios de gestin de archivos que se parecen ligeramente a aquellos de las bases de datos relacionales (ilustrado en los comandos AGGREGATE y MATCH FILES); estos combinan la clave de respuestas con el registro de cada alumno, calculan la DE y la media, y los adjuntan a cada registro del alumno. Es sencillo generalizar el cdigo para manejar los puntajes secundarios, algoritmos de puntaje ms complicados, etc.

94

TITLE SUBTITLE SET COMMENT FILE HANDLE DATA LIST FILE HANDLE SAVE COMMENT FILE HANDLE DATA LIST

COMMENT MATCH FILES COMMENT COMMENT VECTOR DO REPEAT COMPUTE IF END REPEAT COMMENT TABLES

SAMPLE SPSSX SETUP FOR SCORING A HYPOTHETICAL 100-ITEM MULTIPLE CHOICE TEST DAVE SWANSON, SEPTEMBER 6, 1991 LENGTH=64/WIDTH=132 READ IN AND SAVE THE ANSWER KEY KEYDAT/NAME=KEY.DAT FILE=KEYDAT/ EXAMCODE,KEY1 TO KEY100 (A8,1X,100A1) KEYSYS/NAME=KEY.SYS OUTFILE=KEYSYS READ IN EXAMINEE RESPONSE STRINGS RESPDAT/NAME=RESPONSE.DAT FILE=RESP/ SSN,EXAMCODE,RESP1 TO RESP100 (F9.0,1X,A8,1X,100A1) ADD THE ANSWER KEY TO RESPONSE STRING RECORDS FILE=*/TABLE=KEYSYS/BY EXAMCODE COMPARE THE KEY TO RESPONSES AND CREATE A 0/1 VECTOR OF INCORRECT/CORRECT ANSWERS SCORE(100,F1.0) K=KEY1 TO KEY100/R=RESP1 TO RESP100/S=SCORE1 TO SCORE100 S=0 (K EQ R) S=1 PRINT A CROSSTABULATION OF RESPONSES FOR EACH ITEM FORMAT=CWIDTH(10,3) NSPACE LIGHT/ TABLE= RESP1 + RESP2 + ... + (habr que mecanografiar todas las respuestas) RESP100 BY (LABELS)/ STATISTICS=COUNT((F3.0) ) GENERATE RELIABILITY STATISTICS VARIABLES=SCORE1 TO SCORE100/ SCALE(TOTAL)=SCORE1 TO SCORE100/ STATISTICS=DESCRIPTIVE,SCALE,ANOVA/ SUMMARY=ALL CALCULATE A PERCENT CORRECT SCORE FOR THE TOTAL TEST PCSCORE=100*MEAN(SCORE1 TO SCORE100) CALCULATE A STANDARD SCORE FOR THE TOTAL TEST MEANSD/NAME=MEANSD.TMP OUTFILE=MEANSD/BREAK=EXAMCODE/ PCMEAN=MEAN(PCSCORE)/PCSD=SD(PCSCORE) FILE=*/TABLE=MEANSD/BY EXAMCODE STDSCORE=500+100*(PCSCORE-PCMEAN)/PCSD WRITE OUT A SCORE REPORT REPORT/NAME=REPORT.LIS OUTFILE=REPORT/ SSN,PCSCORE,STDSCORE (3F9.0)

COMMENT RELIABILITY

COMMENT COMPUTE COMMENT FILE HANDLE AGGREGATE MATCH FILES COMPUTE COMMENT FILE HANDLE WRITE EXECUTE FINISH

Captulo 6. Items de emparejamiento ampliado (tipo R)

95

Comparacin de los tems del formato de cinco opciones con los de emparejamiento ampliado
En varios estudios realizados para investigar el nmero ptimo de opciones para los tems de opcin mltiple, hemos descubierto sistemticamente que, si las otras condiciones son iguales, un mayor nmero de opciones es mejor que pocas opciones. Sobre la base de los tems usados en los exmenes de la NBME, los de emparejamiento ampliado son ms discriminatorios que los de los dems formatos; los de tipo A de cinco opciones son los siguientes en orden de preferencia, y los diferentes formatos de tems de verdadero/falso son los peores. En estudios controlados que comparaban los tipos de tems de 5 opciones con los de emparejamiento ampliado (que en todo otro sentido eran equivalentes), stos ltimos demostraron ser ms discriminatorios que los de 5 opciones; se pueden alcanzar niveles comparables de reproducibilidad con el formato de emparejamiento ampliado mediante el uso de un tercio menos de tems que con los tems de 5 opciones. Se encontr que los tems de emparejamiento ampliado tambin son ms difciles que los de 5 opciones con un contenido paralelo: la probabilidad de adivinar la respuesta correcta es menor y los redactores de tems no siempre pueden seleccionar los distractores ms funcionales cuando se reduce a cinco el nmero de opciones. La siguiente tabla muestra las respuestas de los alumnos a un tem presentado en un formato de 5 opciones y en un formato de 15 opciones. Originalmente, el tem fue redactado como uno de 15 opciones; el autor redujo el nmero de opciones a cinco mediante la seleccin de lo que l consider eran los mejores distractores (B, F, G, J, N). El tem fue evidentemente ms sencillo con el formato de 5 opciones (valor p de 81 frente a un valor de 59) y la discriminacin fue notablemente menor (no se muestra). Hay un aumento en la probabilidad de que los alumnos seleccionen la respuesta correcta en el formato de 5 opciones, especialmente porque los redactores no identifican de manera uniforme los distractores ms destacados (ejemplo, opcin D). Los exmenes elaborados a partir de tems de emparejamiento ampliado tienden a esparcir las respuestas de los alumnos de menor capacidad; la lista ampliada de opciones les proporciona ms oportunidades de demostrar lo que no saben. Respuestas de los alumnos Formato 5 opciones 15 opciones 2 A B 81 59 2 C D * 13 1 E F 4 2 G 0 1 0 1 H I J 14 7 2 2 K L M * 8 N 1 0 0 O

Para obtener ms informacin, consulte:


Case SM, Swanson DB, Ripkey DR. Comparison of items in five-option and extended-matching format for assessment of diagnostic skills. Academic Medicine. 1994;69(supl.):S1-S3. Case SM, Swanson DB. Extended-matching items: a practical alternative to free-response questions. Teaching and Learning in Medicine. 1993;5(2):107-115. Swanson DB, Case SM. Trends in written assessment: a strangely biased perspective. En: Harden R, Hart I, Mulholland H, eds. Approaches to Assessment of Clinical Competence. Norwich, Inglaterra: Page Brothers; 1992:38-53.

96

Desde los A hasta los R y de vuelta


Si bien la investigacin ha demostrado sistemticamente una ventaja psicomtrica de los tems de emparejamiento ampliado (tipo R) con respecto a los de cinco opciones (tipo A), es probable que existan casos en los que se necesite convertir los tems de un formato a otro. Esto debera ser simple. Por ejemplo, el siguiente tem fue escrito como uno de tipo A de cinco opciones: Un paciente con caractersticas fenotpicas clsicas de trisoma 21 (sndrome de Down) tiene 46 cromosomas en cada uno de 100 cariotipos en metafase. Cul de las siguientes opciones sera la explicacin ms probable de este hallazgo? A. B. C. Eliminacin Mosaicismo Mutacin somtica *D. E. Translocacin Trisoma no detectada

Esto puede transformase sin problemas en el siguiente tem de emparejamiento ampliado. Una vez que est en este formato, se pueden redactar enunciados adicionales para ampliar el conjunto. A. B. C. Eliminacin Mosaicismo Mutacin somtica D. E. Translocacin Trisoma no detectada

Para cada paciente con anomalas genticas, escoja el patrn gentico que es ms probable que est asociado. Un paciente con caractersticas fenotpicas clsicas de trisoma 21 (sndrome de Down) tiene 46 cromosomas en cada uno de 100 cariotipos en metafase. Se pueden escribir opciones adicionales para plantear un desafo mayor a los alumnos. A. B. C. D. E. Eliminacin Impronta genmica Mosaicismo Pleiotropa Penetrancia reducida F. G. H. I. Mutacin somtica Translocacin Trisoma no detectada Expresividad variable

Para cada paciente con anomalas genticas, escoja el patrn gentico que es ms probable que est asociado. Los tems de este conjunto pueden volver a convertirse a los de tipo A con cinco o ms opciones.

Captulo 6. Items de emparejamiento ampliado (tipo R)

97

Captulo 7 Items de seleccin de N respuestas: una ampliacin del formato de emparejamiento ampliado
El formato de seleccin de N respuestas puede ser similar al formato de emparejamiento ampliado o al de tipo A; la diferencia principal es que el alumno debe elegir 2, 3, 4 hasta 5 de las opciones enumeradas. Al igual que con los conjuntos de emparejamiento ampliado, la lista de opciones puede incluir hasta 26 opciones. El formato fue desarrollado para reemplazar a los tems negativos o de opcin doble, particularmente en reas de Mantenimiento de salud o Prevencin de enfermedades. Los tems podran referirse a varios pacientes con diferentes factores de riesgo que asisten a un examen de rutina; al alumno se le pedir que seleccione los anlisis de laboratorio o las vacunas que se deben pedir para cada paciente. El formato tambin podra usarse para tems de manejo de urgencias en los que se realizan varios procedimientos simultneos; el alumno deber seleccionar un nmero especfico de medidas de la lista de opciones. Las reglas de redaccin de tems son las mismas que para los conjuntos de emparejamiento ampliado. Las opciones deben ser breves (normalmente una sola palabra o una frase muy breve); las vietas del paciente pueden ser extensas. Si el conjunto implica la toma de decisiones con respecto al tratamiento, cada vieta debe contener todos los datos relevantes de la historia clnica y de los exmenes fsicos. Al igual que con los tems de emparejamiento ampliado, el formato funciona perfectamente para tems que aparentemente son muy fciles; se deben evitar las vietas engaosas o innecesariamente complejas. El formato de seleccin de N respuestas est diseado para especificar exactamente cuntas opciones se deben seleccionar. Las razones para esta decisin derivan de la diferencia esencial entre los tems de verdadero/falso y los de seleccin de la mejor respuesta: en los primeros el alumno debe indicar todas las respuestas que son adecuadas, y en los segundos el alumno debe indicar un nmero especfico de respuestas. Al especificar el nmero de opciones a seleccionar, se modifica la tarea desde una de verdadero/falso a una de seleccin de la mejor respuesta. Las investigaciones indican que se prefiere el puntaje de crditos parciales, pero las consideraciones logsticas podran llevar a descartar esto. Si utiliza un puntaje de todo o nada, los tems pueden ser muy difciles y es mejor que los alumnos seleccionen no ms de dos o tres opciones. Para obtener ms informacin, consulte:
Ripkey DR, Case SM, Swanson DB. A new item format for assessing aspects of clinical competence. Academic Medicine. 1996;71 (supl.):S34-S36.

Captulo 7. Items de seleccin de N respuestas: una ampliacin del formato de emparejamiento ampliado

99

A. B. C. D. E.

Polineuropata diabtica Enfermedad de Huntington Sndrome medular lateral Encefalopata por intoxicacin con plomo Meduloblastoma

F. G. H. I. J.

Esclerosis mltiple Enfermedad de Parkinson Glioma pontino Tabes dorsalis Encefalopata de Wernicke

Un hombre de 50 aos de edad tiene inicio gradual de confusin mental, desorientacin y prdida de la memoria de corto plazo. Tiene pie pndulo izquierdo. Un frotis de sangre revela microcitosis y punteado basfilo de eritrocitos. (Seleccione los dos diagnsticos ms probables.) En el ejemplo anterior, se presentara una contradiccin con respecto a los diagnsticos probables, pero la tarea se aclara si se pide al alumno que seleccione los dos diagnsticos ms probables. Las opciones se pueden diagramar de la siguiente manera:

Diagnsticos menos probables

Diagnsticos ms probables

Ejemplo de un conjunto de seleccin de N respuestas


A. B. C. D. E. F. Calcio Fluoruro cido flico Hierro Vitamina A Vitamina B1 (tiamina) G. H. I. J. K. Vitamina B6 Vitamina B12 (cianocobalamina) Vitamina C Vitamina D Vitamina E

Para cada nio, seleccione los suplementos de vitaminas o minerales adecuados. 1. Se lleva a un nio de 1 mes de edad al mdico para realizar un examen de nio sano. Ha sido alimentado nicamente con leche materna y el examen presenta hallazgos normales. (SELECCIONE 2 SUPLEMENTOS) Respuesta: B, J 2. Una nia de 6 aos presenta fibrosis qustica. No recibe medicamentos. (SELECCIONE 3 SUPLEMENTOS) Respuesta: E, J, K

100

Ejemplo de un conjunto de seleccin de N respuestas


A. B. C. D. E. Anlisis y cultivo de lquido cefalorraqudeo Hemocultivo Recuento sanguneo completo Anlisis de heces para determinar leucocitos Medicin de los niveles de electrolitos sricos F. G. H. I. Anlisis de orina Cultivo de orina Radiografa de abdomen Radiografa de trax

Para cada nio con fiebre, seleccione los estudios de diagnstico iniciales apropiados. 1. Una nia de 1 ao de edad que gozaba de buena salud llega al departamento de urgencias debido a que durante 1 da estuvo con fiebre. Su temperatura es de 41C (105,8F). No presenta otros sntomas. El examen fsico no presenta anomalas. (SELECCIONE 4 ESTUDIOS) Respuesta: B, C, G, I 2. Una nia recin nacida de 10 das que gozaba de buena salud llega al departamento de urgencias debido a que tiene fiebre desde hace 2 horas. Naci a trmino luego de un embarazo sin complicaciones. Su temperatura es de 39C (102,2F). El examen fsico no presenta anomalas. (SELECCIONE 6 ESTUDIOS). Respuesta: A, B, C, E, G, I 3. Un nio de 7 aos de edad que padece anemia falciforme llega al departamento de urgencias debido a la presencia de fiebre desde hace 1 da y dolor en el pecho desde hace 1 hora. Su temperatura es de 39,5C (103,1F). Presenta ruidos respiratorios levemente disminuidos en la parte inferior del pulmn derecho; no tiene distrs respiratorio. (SELECCIONE 3 ESTUDIOS). Respuesta: B, C, I

Captulo 7. Items de seleccin de N respuestas: una ampliacin del formato de emparejamiento ampliado

101

HOJA CLNICA Antecedentes


Sexo: Edad actual: Dolencia principal: Antecedentes sociales: Estado civil: Ocupacin: Consumo de bebidas alcohlicas: Tabaquismo: Programa de ejercicios: Historia clnica: Niez: Vacunas: masculino 28 aos examen de mantenimiento de la salud soltero programador de computacin de 2 a 4 cervezas/fin de semana 5 a 10 cigarrillos diarios entre las edades de 16 a 24 aos sedentario obesidad desde escuela primaria todas las vacunas de la niez; ltima vacuna de toxoide tetnico a la edad de 15 aos; ninguna vacuna desde la niez ninguna visita al mdico desde la universidad edad del padre 57; madre hipertensa y obesa de 55 aos no tiene no tiene ninguno ninguna 178 cm (70 pulgadas) 134 kg (295 libras) 148/86 mm Hg 90/min 16/min exantema eritematoso en la ingle obeso sin pedidos

Examen de deteccin de enfermedades: Antecedentes familiares: Padres: Hermanos: Hijos: Medicamentos actuales: Alergias:

Examen fsico
Altura: Peso: Signos vitales: Presin arterial Pulso Frecuencia respiratoria Piel: Abdominal: Anlisis de laboratorio:

102

1. Para el paciente cuya hoja clnica se presenta, seleccione las afecciones para las cuales presenta un aumento en el riesgo. (SELECCIONE 4 AFECCIONES) A. B. C. D. E. F. Alcoholismo Cncer de colon Enfermedad de las arterias coronarias Diabetes mellitus Hemocromatosis VIH G. H. I. J. K. L. Hipertensin arterial Hipotiroidismo Osteoartritis Cncer de piel Cncer de tiroides Infeccin del tracto urinario

2. Para el paciente cuya hoja clnica se presenta, seleccione las intervenciones ms adecuadas para el mantenimiento de la salud. (SELECCIONE 4 INTERVENCIONES) A. B. C. D. E. F. G. Recuento sanguneo completo Asesoramiento alimentario Asesoramiento en ejercicio fsico Perfil de lpidos sricos en ayunas Vacunacin contra la hepatitis B Anlisis de VIH Vacuna contra la gripe H. I. J. K. L. Mediciones de niveles de creatinina y nitrgeno ureico en suero (BUN) Vacuna de toxoide tetnico Anlisis de la funcin tiroidea Anlisis de orina Radiografa de trax

Captulo 7. Items de seleccin de N respuestas: una ampliacin del formato de emparejamiento ampliado

103

Seccin IV Temas complementarios

Esta seccin incluye algunos temas complementarios que estn relacionados con la evaluacin.

Captulo 8 Interpretacin de los resultados del anlisis de los tems


Muchas facultades proporcionan a los profesores un informe con los resultados del anlisis de los tems despus de cada examen de opcin mltiple. Este informe es una excelente fuente de informacin sobre el tem y es til en la evaluacin de su calidad, as como de la exactitud de la clave de respuestas. A continuacin se presentan ejemplos de resultados de cuatro tems; cada uno ejemplifica una situacin comn. Se dividi a los alumnos que rindieron el examen en un grupo Superior y un grupo Inferior, segn su rendimiento general en todo el examen. Si tiene un nmero reducido de alumnos, incluya el 50% superior de los alumnos en el grupo de nivel Superior y el otro 50% en el grupo de nivel Inferior. Si tiene un nmero elevado de alumnos, podra incluir el 25% superior de los alumnos en el grupo de nivel Superior y el 25% inferior en el grupo de nivel Inferior. Normalmente, el informe de los resultados del anlisis de los tems indica el porcentaje de alumnos en cada grupo que seleccion cada opcin. A menudo, tambin incluye alguna medicin de la dificultad del tem (p. ej., el valor p, o sea la proporcin de alumnos que respondieron correctamente a la pregunta) y alguna medicin de la discriminacin (p. ej., un biserial o biserial puntual). Recomendamos concentrar la atencin en el patrn de las respuestas en lugar del nivel de dificultad o ndice de discriminacin.

Captulo 8. Interpretacin de los resultados del anlisis de los tems

107

Para cada ejemplo de tem que se presenta a continuacin, se muestra el porcentaje de alumnos que seleccion cada opcin. La fila designada Total muestra el porcentaje del grupo entero que seleccion cada opcin. Por ejemplo, en el tem N. 1, 1% del grupo Superior seleccion la opcin A; 1% seleccion la opcin B; 91% seleccion la C; 4% seleccion la D; 1% seleccion la E y el 2% seleccion la F. En el mismo tem, el 20% del grupo Inferior seleccion la opcin A; 6% seleccion la B, etc. El asterisco en la opcin B indica que sta era la supuesta respuesta correcta. Item N. 1 Grupo Superior Inferior Total Valor p: 2 A 1 20 9 B* 1 6 2 C 91 51 76 D 4 14 8 E 1 6 3 F 2 3 2

ndice de discriminacin: -0,21

Interpretacin: Este es el modelo tpico de un tem que tiene una clave equivocada: si la respuesta es la opcin B, el tem es muy difcil y el ndice de discriminacin es negativo. Si la clave es B, solamente el 2% de los alumnos respondi correctamente. La respuesta correcta es casi con seguridad la opcin C, pero un experto en el contenido del tem deber revisarlo para estar seguro. Si la respuesta correcta es la opcin C, el valor p se transforma en 76 y el ndice de discriminacin es de 0,46; ambos datos son excelentes desde la perspectiva estadstica y no hay justificativos para realizar cambios en el texto del tem.

Item N. 2 Grupo Superior Inferior Total Valor p: 74 A 0 0 0 B 1 1 1 C* 90 60 74 D 3 25 12 E 3 8 7 F 3 6 6

ndice de discriminacin: 0,33

Interpretacin: El 90% del grupo Superior y el 60% del grupo Inferior seleccion la respuesta correcta. Estas son estadsticas generales excelentes. Se pueden volver a redactar las opciones A y B antes de volver a usar el tem porque muy pocos alumnos seleccionaron esas opciones.

108

Item N. 3 Grupo Superior Inferior Total Valor p: 34 A 44 20 32 B 1 15 7 C* 50 21 34 D 2 22 14 E 1 20 11 F 2 2 2

ndice de discriminacin: 0,30

Interpretacin: El 50% del grupo Superior y el 21% del grupo Inferior seleccion la respuesta correcta. Este es un tem muy difcil que probablemente NO ESTA BIEN REDACTADO. Un gran nmero de alumnos del grupo Superior seleccion la opcin A; el tem puede tener una redaccin deficiente. Verifique la imparcialidad de la opcin A. Asegrese de que la opcin A no sea igualmente correcta.

Item N. 4 Grupo Superior Inferior Total Valor p: 34 A 18 24 22 B 10 24 17 C* 51 21 34 D 17 25 22 E 2 4 3 F 2 2 2

ndice de discriminacin: 0,30

Interpretacin: El desglose de los grupos Superior e Inferior en la opcin C es igual al del tem N. 3; pero este tem puede estar BIEN REDACTADO. A diferencia del tem N. 3, los alumnos que no conocen la respuesta correcta se distribuyen ampliamente entre los diferentes distractores. Obviamente, sera preferible revisar las opciones A, B y D para controlar su correccin y claridad.

Captulo 8. Interpretacin de los resultados del anlisis de los tems

109

Captulo 9 Cmo establecer un estndar de aprobado/reprobado


Definiciones y principios bsicos
Los estndares pueden clasificarse como relativos o absolutos. Un estndar relativo se basa en el rendimiento del grupo que rinde el examen. Los alumnos aprueban o reprueban segn el nivel de su rendimiento con respecto a los otros alumnos que rinden el examen. Los siguientes son ejemplos de estndares relativos. Aquellos alumnos que obtengan un puntaje menor a 1,2 desviaciones estndares por debajo de la media, no aprobarn el examen. El 20 por ciento inferior del grupo no aprobar el examen. Por el contrario, un estndar absoluto no compara el rendimiento de un alumno con el de los otros que rinden el examen. Los alumnos aprueban o reprueban solamente segn el nivel de su rendimiento, sin tener en cuenta el desempeo de los otros alumnos. Todos los alumnos pueden aprobar o todos pueden reprobar. El siguiente es un ejemplo de un estndar absoluto: Aquellos que respondan en forma correcta a menos del 60 por ciento de las preguntas, no aprobarn. A menos que existan razones convincentes para reprobar a un nmero determinado de alumnos, es preferible un estndar absoluto (basado en el rendimiento del alumno) a uno relativo (basado en un ndice de reprobacin en particular).

Principios bsicos para establecer los estndares


Sin tener en cuenta el procedimiento utilizado, el establecimiento de los estndares requiere de un criterio determinado. En todos los casos, el establecimiento de estndares ser arbitrario pero no necesariamente caprichoso. A menos que exista una razn especfica para reprobar a un nmero determinado de alumnos (por ejemplo, solamente existe un nmero determinado de espacios disponibles), un estndar basado en el dominio que tiene el alumno del contenido del examen es preferible a un estndar basado en un ndice de reprobacin en particular.

Captulo 9. Cmo establecer un estndar de aprobado/reprobado

111

Es prudente que participen varios jueces informados en el proceso de establecimiento de estndares. Se presentarn diferencias de opinin, y el uso de varios jueces reducir los efectos conocidos como halcn/paloma (ms exigente/ms indulgente). Se debern proporcionar datos a los jueces sobre el rendimiento de los alumnos en algn momento del proceso de establecimiento de los estndares. El establecimiento de los estndares sin usar dichos datos podr generar estndares basados en informacin insuficiente y resultados poco razonables.

Una referencia til sobre cmo establecer estndares es:


Livingston SA, Zieky MJ. Passing Scores: A Manual for Setting Standards of Performance on Educational and Occupational Tests. Princeton, NJ: Educational Testing Service; 1982.

Dos mtodos de determinacin de estndares en base a juicios sobre los tems


El mtodo de Ebel modificado
Un grupo analiza las caractersticas del alumno en la frontera (entre aprobado y reprobado): es decir, aquel alumno cuya aptitud es apenas suficiente como para permitirle aprobar el examen. Los jueces clasifican a los tems como esencial, importante o indicado. Los jueces indican el nmero de tems en cada categora que obtendra un alumno en la frontera. El estndar de aprobado/reprobado se calcula como el porcentaje de puntos posibles que obtendra un alumno en la frontera.

112

El mtodo de Angoff modificado


Un grupo analiza las caractersticas del alumno en la frontera (entre aprobado y reprobado). Para cada tem del examen, los jueces calculan el porcentaje de alumnos en la frontera que responderan correctamente al tem. El estndar de aprobado/reprobado para el examen es el promedio de los porcentajes de los tems.

Variantes comunes en el mtodo de Angoff


Los jueces pueden o no tener las respuestas correctas a las preguntas. Los jueces pueden o no disponer de la informacin referente al porcentaje de los alumnos que respondieron correctamente a cada pregunta. Luego de un perodo de capacitacin, los jueces pueden continuar trabajando en grupo o individualmente.

Captulo 9. Cmo establecer un estndar de aprobado/reprobado

113

Estndares de compromiso relativo/absoluto: el mtodo de Hofstee


Recientemente, se han desarrollado varios modelos de compromiso que utilizan las ventajas de los procedimientos de establecimiento de estndares tanto relativos como absolutos. Uno de estos mtodos es el de Hofstee, que se describe a continuacin. 1. Los jueces revisan una copia del examen. 2. Luego, los jueces asignan los valores que se indican a continuacin, que definen los estndares aceptables: Porcentaje mnimo aceptable de alumnos reprobados (ndice mnimo de reprobacin) Porcentaje mximo aceptable de alumnos reprobados (ndice mximo de reprobacin) El puntaje ms bajo que permitira que un alumno apruebe el examen (punto mnimo de aprobacin) El puntaje ms elevado requerido para que un alumno apruebe (punto mximo de aprobacin) 3. Despus del examen, se grafica una curva que muestre el ndice de reprobacin en funcin del puntaje de aprobacin. (En la figura que se muestra, la curva se extiende desde la parte inferior izquierda hasta la parte superior derecha.) 4. Los cuatro valores obtenidos en el punto N. 2 se trazan para formar un rectngulo. A menudo, se usan los valores medianos del grupo de jueces. En el ejemplo, se estableci que el ndice de reprobacin apropiado estaba entre 0 y el 20% (ver las lneas horizontales); se determin que el punto adecuado de aprobado/reprobado estaba entre un 50% y un 60% de respuestas correctas (ver las lneas verticales).

% de reprobados

% de respuestas correctas

5. Se traza una lnea en la diagonal desde la parte superior izquierda hasta la parte inferior derecha. El punto de interseccin con la curva es el estndar (es decir, un poco ms del 55% de respuestas correctas en la figura).

Una referencia til en los mtodos de compromiso es:


de Gruijter D. Compromise models for establishing examination standards. Journal of Educational Measurement. 1985;22:263-269.

114

Captulo 10 Reflexiones varias sobre temas relacionados con la evaluacin


A continuacin se presentan unos comentarios sobre una mezcla variada de temas relacionados con exmenes. En general, los puntos que se tratan son conjeturas y se basan en ancdotas ms que en la evidencia. Es decir, reflejan nuestros prejuicios en lugar de los resultados de una investigacin.

Exmenes de mltiples estaciones (tambin conocidos como Exmenes prcticos, Carreras de obstculos, OSCE (examen clnico objetivo estructurado)) Si bien es complejo instalar y administrar este tipo de exmenes, desde el punto de vista logstico, son muy tiles en el rea de las ciencias bsicas, particularmente para evaluar las destrezas prcticas que no se pueden medir en exmenes de papel y lpiz (por ejemplo, la capacidad de usar un microscopio, de realizar un procedimiento de laboratorio). Adems, la reproduccin de algunos tipos de materiales (por ejemplo, resultados de estudios de diagnstico por imgenes, materiales ilustrativos en colores) es muy costoso; en dichas situaciones, el mtodo de mltiples estaciones se puede usar para reducir los costos de la administracin del examen. Exmenes para completar en casa Los exmenes para completar en la casa pueden constituir una experiencia de aprendizaje importante ya que estimulan a los alumnos a leer en profundidad y ampliamente los temas importantes. Lamentablemente, los alumnos tienden a producir libros como respuestas y no queda claro si las respuestas presentadas por los alumnos representan su propio trabajo. Se pueden obtener las mismas ventajas mediante la distribucin de (un gran conjunto de) preguntas de examen con anticipacin y la administracin de (un subconjunto de) estas preguntas en forma de examen de tiempo fijo. Exmenes a libro abierto Estos exmenes pueden ser una buena idea debido al impacto que tienen sobre el tipo de preguntas que prepara el profesor. En los exmenes a libro abierto, no tiene sentido realizar preguntas sobre hechos aislados que pueden buscarse rpidamente en una sola pgina del libro de texto; por lo tanto, el material de evaluacin desarrollado para este tipo de exmenes tiende a concentrarse ms en la comprensin de principios y conceptos fundamentales de situaciones problemticas.

Captulo 10. Reflexiones varias sobre temas relacionados con la evaluacin

115

Pruebas breves frecuentes o exmenes poco frecuentes Las evaluaciones poco frecuentes convierten a cada examen en un acontecimiento importante; es posible que los alumnos dejen de asistir a clases para prepararse, y esta situacin es indeseable. Adems, con los exmenes poco frecuentes, los alumnos quizs no podrn determinar si estudian el material correcto o aprenden con la suficiente profundidad. A pesar de que podrn exigir ms tiempo al profesor, las evaluaciones peridicas reducen la importancia de cada examen individual y ayudan a los alumnos a evaluar mejor su avance. En general, las evaluaciones frecuentes son preferibles, aunque es probable que los alumnos se quejen de todos modos sin considerar el mtodo adoptado.

Guardar los exmenes de manera segura o permitir que los alumnos se queden con ellos Debido a que los exmenes pueden tener un efecto de direccin importante en el aprendizaje de los alumnos, el permitir que ellos retengan el material de evaluacin puede ayudarles a concentrarse en temas clave y a reforzar los objetivos del plan de estudios y del curso (si se supone que los materiales de examen los reflejan). Sin embargo, la preparacin de preguntas adecuadas para un examen implica dedicar mucho tiempo y la calidad del material de la prueba se puede deteriorar con el paso de los aos si el profesor tiene que desarrollar nuevos materiales de examen cada vez que ensea un curso. El mtodo ms apropiado puede ser el de disponer de una muestra de preguntas de buena calidad a fin de guiar el aprendizaje de los alumnos pero mantener un banco de preguntas seguras para uso repetido. Es necesario recordar que es probable que la seguridad sea deficiente ya que los alumnos muchas veces memorizan las preguntas y las intercambian.

Uso de exmenes acumulativos Los exmenes acumulativos que responsabilizan a los alumnos de todo el material presentado hasta la fecha fomentan la concentracin en las interrelaciones entre los temas, particularmente si las preguntas del examen requieren de la comprensin tanto de los temas recientemente presentados como de los anteriores. El uso de exmenes que abarcan solamente el material presentado desde el examen anterior estimula a los alumnos para que estudien los temas aislados; se pueden perder las relaciones entre los temas de unidades diferentes. Ya que los alumnos pueden tener un mal rendimiento en una serie de exmenes porque nunca dominan el material bsico, este mtodo tambin puede, por otra parte, motivar a los alumnos para que corrijan sus deficiencias.

Uso de exmenes integradores entre cursos diferentes Al igual que con el uso de los exmenes acumulativos, los integradores entre cursos motivan a los alumnos para que analicen las interrelaciones entre las disciplinas y los temas; esto debera ser muy til para la retencin a largo plazo y para la aplicacin de conocimientos en el rea de las ciencias bsicas a situaciones clnicas. Generalmente, se necesitan profesores de ciencias bsicas y de departamentos clnicos para la preparacin de dichos exmenes. Si bien implican dedicar mucho tiempo, este esfuerzo en conjunto puede dar como resultado mejores materiales de examen y adems producir discusiones tiles entre los profesores sobre el material que debe incluirse en el plan de estudios.

116

Apndice A El cementerio de formatos de tems de la NBME

Apndice A El cementerio de formatos de tems de la NBME


El primer examen de la NBME fue en el ao 1916. Tena una duracin de cinco das y constaba de pruebas escritas, orales, de laboratorio y clnicas. Luego de la Primera Guerra Mundial, la NBME design una comisin que viaj a Inglaterra, Escocia y Francia para estudiar los mtodos de evaluacin usados para otorgar la licenciatura en medicina. En el ao 1922, la NBME administr un examen nuevo. La Parte I consista en un examen del tipo de redaccin de ensayos de tres das sobre las ciencias bsicas; la Parte II era un examen del tipo de redaccin de ensayos de dos das de duracin sobre las principales ciencias clnicas; y la Parte III tena una duracin de un da y era un examen oral que se llevaba a cabo al lado del paciente. Estos formatos se usaron hasta la dcada de 1950 cuando los exmenes de opcin mltiple se popularizaron. En el ao 1951, la NBME, con la colaboracin de los Educational Testing Services, comenzaron un estudio de tres aos en el que comparaban los exmenes del tipo de redaccin de ensayos con los de opcin mltiple. La Parte I y la II se transformaron al formato de opcin mltiple en el ao 1953; la Parte III se revis durante los primeros aos de la dcada de 1960 a medida que se introducan y se suspendan nuevos formatos. El primer examen constaba principalmente de tems de tipo A que implicaban la retencin de informacin, como se muestra en el siguiente ejemplo: En los 40 aos desde el primer examen con tems de opcin mltiple, la NBME ampli el alcance de los tems de tipo A para evaluar las aptitudes de razonamiento y de resolucin de problemas con la inclusin de una vieta clnica en la mayora de los enunciados de los tems. En la actualidad, el formato tipo A sigue siendo el formato ms usado en los exmenes de los Pasos. Muchos otros formatos que se desarrollaron durante este perodo han sido suspendidos. Estos formatos (designados con una letra segn el orden de origen) se describen en las siguientes pginas. Los tems que se muestran en esta seccin fueron extrados de:
Hubbard JP, Clemens WV. Multiple-Choice Examinations in Medicine. Philadelphia PA: Lea & Febiger;1961.

De los siguientes agentes, el agente profilctico ms eficaz para la prevencin de una recidiva de la fiebre reumtica es A. cido acetilsaliclico B. cido paraaminobenzoico C. hormona adrenocorticotrpica D. cortisona E. sulfadiazina

Las crticas de estos formatos fueron extradas de:


Hubbard JP. Measuring Medical Education. Philadelphia PA: Lea & Febiger;1971. Hubbard JP, Levit EJ. The National Board of Medical Examiners: The First Seventy Years. NBME;1985.

Tambin de varios artculos escritos por Morton publicados en el Federation Bulletin en 1985 y 1986.

Apndice A

119

Items de tipo B Estos eran tems de emparejamiento que consistan en una lista de ttulos identificados con una letra, seguida de una lista de palabras o frases identificadas con nmeros. El alumno deba elegir el nico ttulo que se asociaba ms estrechamente a cada palabra o frase. Ya que cada respuesta poda utilizarse ms de una vez o nunca, los tems de tipo B no se podan resolver por eliminacin. Se consideraba que estos tems ampliaban el alcance de un examen con preguntas de opcin mltiple ya que permitan la evaluacin de un nmero de temas relacionados a travs de una simple serie de preguntas. A diferencia de los formatos de emparejamiento que se usan en la actualidad, los tems de tipo B normalmente no incluan una pregunta introductoria; en consecuencia, algunas veces la pregunta resultaba poco clara. Generalmente, estos tems funcionaban satisfactoriamente, y solamente se interrumpi su uso hace poco tiempo debido a que el formato de emparejamiento ampliado se torn ms comn.

Ejemplo de un tem de tipo B


INSTRUCCIONES: Cada conjunto de tems de emparejamiento en esta seccin consta de una lista de tres a cinco opciones identificadas con letras (algunas de las cuales pueden hallarse dentro de figuras), seguida de varios tems numerados. Para cada tem numerado, seleccione la UNICA opcin identificada con una letra que est asociada ms estrechamente con el tem y llene el crculo que contenga la letra correspondiente en la hoja de respuestas. Cada opcin con letra puede seleccionarse una sola vez, ms de una vez o nunca.

A. B. C. D. E. 1. 2. 3. 4. 5.

Coartacin de la aorta Conducto arterioso permeable Tetraloga de Fallot Anillo vascular artico Atresia tricspide Se beneficia con anastomosis arterial pulmonar-sistmica Tipo ms comn de cardiopata ciantica congnita Corregido quirrgicamente mediante reseccin y anastomosis termino-terminal Causa posible de disfagia en bebs y nios Hipertensin en los brazos e hipotensin en las piernas

120

Items de tipo D Estos eran tems de emparejamiento complejos que constaban de tres alteraciones funcionales (designadas con una letra) y cinco situaciones (en una lista numerada). El alumno deba 1) seleccionar la alteracin funcional o la categora con la que se relacionaban cuatro de las cinco situaciones que se planteaban, y 2) indicar la nica situacin que no perteneca a esa categora. Se consideraba que estos tems requeran de una comprensin discriminatoria de un nmero de factores similares. No obstante, los tems de tipo D eran difciles de redactar y las instrucciones eran confusas. Adems, no discriminaban entre los alumnos que tenan los conocimientos para responder y los que no.

Ejemplo de un tem de tipo D


INSTRUCCIONES: Hay dos respuestas para cada una de las siguientes preguntas. En la lista de la izquierda se presentan tres categoras identificadas con letras. Exactamente cuatro de las cinco opciones numeradas en la lista de la derecha estn relacionadas de alguna manera con UNA de estas categoras. (1) En la lnea correspondiente de la hoja de respuestas rellene el espacio debajo de la letra correspondiente a la categora a la que pertenecen estas cuatro opciones. (2) Luego rellene el espacio debajo del nmero de la opcin en la lista de la derecha que NO pertenece a la misma categora que las otras cuatro.

A. Eosinofilia de importancia diagnstica B. Plasmocitosis de importancia diagnstica C. Linfocitosis de importancia diagnstica

1. 2. 3. 4. 5.

Triquinosis Mieloma mltiple Sndrome de Loeffler Enfermedad de Hodgkin Esquistosomiasis

Apndice A

121

Items de tipo K Estos tems eran del formato de tems mltiples de verdadero/falso ms comnmente usado por la NBME. Constaban de un enunciado seguido de cuatro opciones, de las cuales una o ms eran correctas. Se crea que los tems de tipo K evaluaban el conocimiento y la comprensin profundos de varios aspectos de una enfermedad, proceso o procedimiento, y el alumno deba conocer varios datos diferentes sobre un tema determinado. No obstante, los tems de tipo K fueron criticados por ser demasiado complicados ya que los alumnos deban recordar constantemente el cdigo de respuestas. Adems, las combinaciones posibles de respuesta introdujeron un efecto de pistas implcitas que reduca la discriminacin de los tems y disminua la confiabilidad del examen. No era una tarea fcil redactar tems de verdadero/falso que fueran satisfactorios y sin ambigedad. Como los tems de tipo K solamente podan incluir datos absolutamente falsos o verdaderos, no podan usarse para evaluar criterios clnicos excepto en las comparaciones (por ejemplo, El medicamento X es mejor que el medicamento Y para el tratamiento de la enfermedad K). Los tems de tipo K eran ms difciles y menos discriminatorios que otros tipos. Adems, eran menos eficientes que otros formatos de opcin mltiple y la confiabilidad relativa por unidad de tiempo de examen era inferior.

Ejemplo de un tem de tipo K


Instrucciones resumidas A Solamente 1, 2, 3 B Solamente 1, 3 C Solamente 2, 4 D Solamente 4 E Todas son correctas

1.

Un nio que sufre de exacerbacin aguda de fiebre reumtica normalmente tiene (1) un ndice de sedimentacin elevado (2) un intervalo de PR prolongado (3) un ttulo elevado de la antiestreptolisina O (4) ndulos subcutneos

122

Items de tipo C Los tems de tipo C eran similares a los de tipo B en apariencia, pero eran de verdadero/falso mltiples en lo que respecta a la tarea que exiga a los alumnos. El tem de tipo C constaba de una lista de ttulos identificados con una letra, seguida de una lista de palabras o frases identificadas con nmeros. Para cada tem numerado, los alumnos deban decidir si la opcin A era verdadera, si B era verdadera, ambas eran verdaderas (opcin C) o si ninguna era verdadera (opcin D). Este tipo de tem se utilizaba para comparar y contrastar dos enfermedades, signos y sntomas, hallazgos de laboratorio, etc. Los tems de tipo C coinciden con los de tipo K en cuanto al nivel de dificultad. El problema principal con los tems de tipo C era el de decidir hasta qu punto una opcin tena que ser verdadera para ser elegida. Por ejemplo, si un dato se asociaba con las opciones A y B pero estaba ms estrechamente relacionado con la opcin A, el alumno tena que decidir si la respuesta adecuada era solamente A, o tanto A como B. Con asociaciones relativamente dbiles, el alumno tena que decidir si la relacin era lo suficientemente evidente o si ninguna era la respuesta adecuada. Estas decisiones no estaban relacionadas con el conocimiento mdico sino que forzaban al alumno a pensar en cul era la intencin del redactor del tem.

Ejemplo de un tem de tipo C


INSTRUCCIONES: Cada conjunto de tems de emparejamiento en esta seccin consta de una lista de cuatro opciones identificadas con letras seguida de varios tems numerados. Para cada tem numerado, seleccione la UNICA opcin identificada con una letra que est asociada ms estrechamente con el tem y llene el crculo que contenga la letra correspondiente en la hoja de respuestas. Cada opcin con letra puede seleccionarse una sola vez, ms de una vez o nunca. A. B. C. D. 1. 2. 3. 4. Malaria por Plasmodium vivax Malaria por Plasmodium falciparum Ambas Ninguna La combinacin de primaquina y cloroquina es el tratamiento de eleccin en caso de un ataque agudo. Las crisis clnicas se suprimen con la ingesta de cloroquina una vez por semana mientras permanece en el rea endmica. Se cura totalmente mediante tratamiento con cloroquina. La infeccin se previene mediante la ingesta de cloroquina una vez por semana.

Apndice A

123

Items de tipo E Estos eran tems de formato mltiple de verdadero/falso que se basaban en el anlisis de las relaciones. Los alumnos que contestaban este tipo de preguntas an hoy las llaman Verdadero, Verdadero y no relacionado. Los tems de tipo E constaban de una oracin con dos partes principales: una afirmacin y la razn para esa afirmacin. El alumno deba seleccionar la opcin A si ambas partes eran oraciones verdaderas y la razn era la explicacin correcta de la afirmacin; B, si ambas partes eran oraciones verdaderas pero la razn no era la explicacin correcta de la afirmacin; C, si la afirmacin era verdadera pero la razn era una oracin falsa; D, si la afirmacin era falsa pero la razn era una oracin verdadera; E, si tanto la afirmacin como la razn eran oraciones falsas. Se crea que las buenas aptitudes de razonamiento y la comprensin de los principios bsicos eran elementos necesarios para responder correctamente a este tipo de tems. No obstante, los tems de tipo E eran difciles de elaborar y eran confusos para los alumnos.

Ejemplo de un tem de tipo E


Instrucciones resumidas A B C D E Verdadero Verdadero Verdadero Falso Falso Verdadero Verdadero Falso Verdadero Falso La razn es la explicacin correcta. La razn NO es la explicacin correcta.

Afirmacin 1. El herpes simple es generalmente considerado como una infeccin autgena 2. La leche de vaca es preferible a la leche materna en la alimentacin de los bebs PORQUE

Razn los pacientes que reciben tratamiento mediante fiebre con frecuencia desarrollan herpes. la leche de vaca tiene un contenido ms elevado de calcio.

PORQUE

124

Items de tipo H Estos eran tems de comparacin que constaban de oraciones en pares que describan dos entidades que se comparaban en sentido cuantitativo. El alumno deba seleccionar la opcin A, si A era mayor que B; la opcin B si B era mayor que A; y la opcin C, si ambas eran aproximadamente iguales. Si bien estaba por lo general acordado que las preguntas que dependan de la memorizacin de cantidades absolutas deban ser limitadas, los tems de tipo H eran considerados tiles para los casos en los cuales la retencin de informacin cuantitativa era considerada importante. La dificultad para los alumnos radicaba en la decisin de cun grande deba ser la diferencia para considerarse relevante.

Ejemplo de un tem de tipo H


INSTRUCCIONES: Las siguientes oraciones en pares describen dos entidades que se deben comparar en un sentido cuantitativo. En la lnea correspondiente de la hoja de respuestas rellene el espacio debajo de A si (A) es mayor que (B), B si (B) es mayor que (A), C si las dos son iguales o casi iguales.

1. (A) (B) 2. (A) (B)

La dosis teraputica habitual de epinefrina La dosis teraputica habitual de efedrina La expectativa de vida con glioblastoma del lbulo occipital La expectativa de vida con glioblastoma del lbulo frontal

Apndice A

125

Items de tipo I Estos eran similares a los tems de tipo H. Constaban de pares de frases que describan condiciones o cantidades cuya relacin entre s poda variar. Los alumnos deban seleccionar la opcin A, si las dos frases estaban relacionadas directamente (es decir, un aumento en la primera estaba acompaado de un aumento en la segunda, o una disminucin en la primera estaba acompaada de una disminucin en la segunda); la opcin B, si las frases estaban relacionadas de manera inversa (es decir, un aumento en la primera estaba acompaado de una disminucin en la segunda, o una disminucin en la primera estaba acompaada de un aumento en la segunda); o la opcin C, si los cambios eran independientes uno del otro.

Ejemplo de un tem de tipo I


INSTRUCCIONES: Cada uno de los siguientes pares de frases describe condiciones o cantidades que pueden o no estar relacionadas. En la lnea correspondiente de la hoja de respuestas rellene el espacio debajo de A si un aumento en la primera est acompaado de un aumento en la segunda, o si una disminucin en la primera est acompaada de una disminucin en la segunda B si un aumento en la primera est acompaado de una disminucin en la segunda, o si una disminucin en la primera est acompaada de un aumento en la segunda C si los cambios en la primera no estn necesariamente acompaados de cambios en la segunda.

1.

(A) (B) (A) (B)

Volumen de orina Peso especfico de la orina Concentracin proteica en plasma Presin osmtica coloidal del plasma

2.

Ni el formato H ni el I fueron particularmente populares. Esto se deba a que existan ms posibilidades de adivinar cul era la respuesta correcta porque se presentaban menos opciones que en otros tipos de tems. Adems, los tems se concentraban en detalles de menor importancia en lugar de contemplar los conceptos cientficos.

126

En su serie publicada en el Federation Bulletin, Morton (1985-86) afirmaba que se incluan diferentes tipos de tems en los exmenes de licenciatura en medicina simplemente para sumar variedad a una evaluacin extensa. Sin embargo, 25 aos despus de dejar de tener un examen basado en la redaccin de ensayos para adoptar otro con preguntas de opcin mltiple, la NBME analiz las investigaciones realizadas sobre los diferentes tipos de preguntas de opcin mltiple usadas, y como consecuencia la variedad de tipos de tems luego se redujo hasta incluir los de tipo A, B, C, G, K, X y M. A mediados de la dcada de 1980, el personal revis nuevamente los tipos de tems. El consenso general, en esos momentos, era de que cuatro tipos bsicos proporcionaban la suficiente variedad como para evaluar los conocimientos considerados importantes para recibir el certificado otorgado por la NBME. Estos cuatro tipos bsicos incluan los tems de tipo A, B, C y K. Los de tipo G (conjuntos de tems de tipo A), N (conjuntos de tems de tipo K) y M ya no se consideraban como formatos separados. En los ltimos aos, la variedad de tipos de tems nuevamente ha sido revisada. Los exmenes actuales de los Pasos incluyen solamente tems de tipo A y R. Algunos de los pasos que se siguen para mejorar los exmenes son: concentrarse en tipos de tems que sean psicomtricamente bien fundados, educar a los redactores en diferentes tcnicas de redaccin de tems, concentrarse en tems que impliquen la toma de decisiones clnicas en lugar de la retencin de datos, y someter a preexamen a los tems recientemente redactados.

Apndice A

127

Apndice B Ejemplos de plantillas, enunciados, preguntas introductorias y listas de opciones para la redaccin de tems en el rea de ciencias bsicas y clnicas

Apndice B Ejemplos de plantillas, enunciados, preguntas introductorias y listas de opciones para la redaccin de tems en el rea de ciencias bsicas y clnicas
Anatoma macroscpica
Plantilla para tems de anatoma macroscpica Enunciado del tem (vieta del paciente): (Describe un paciente con su problema) Pregunta introductoria: En cul de las siguientes estructuras o procesos es ms probable que se presente una anomala? Opciones: (Lista de estructuras o procesos)

Un hombre de 65 aos de edad tiene dificultades para levantarse cuando est sentado y para enderezar su tronco, pero no tiene inconvenientes para flexionar sus piernas. Cul de los siguientes msculos es ms probable que haya sufrido una lesin? A.* Glteo mayor D. Iliopsoas B. Glteo menor E. Obturador interno C. Posterior del muslo Un hombre de 30 aos de edad presenta prdida de sensacin de temperatura y de dolor en el lado izquierdo de la cara y desde el cuello hacia abajo en el lado derecho del cuerpo; parlisis parcial del velo del paladar, la laringe y faringe en la izquierda; y ataxia en el lado izquierdo. Este sndrome es probablemente el resultado de una trombosis de cul de las siguientes arterias? A. Basilar D. Cerebelosa superior derecha B. *Cerebelosa inferior posterior derecha E. Cerebelosa superior izquierda C. Cerebelosa inferior posterior izquierda

Apndice B

131

Introducir una vieta que describe un paciente con un problema. Cul de las siguientes opciones sera la causa ms probable del trastorno en la rodilla? A. Condromalacia patelar F. Osteocondritis disecante B. Luxacin (tibiofemoral) G. Osteosarcoma C. Fractura de rtula H. Bursitis prerrotuliana D. Quiste ganglionar I. Artritis sptica E. Tumor seo de clulas gigantes J. Desgarro de menisco

Introducir una vieta que describe un paciente con un problema. Cul de los siguientes vasos sanguneos es el que se debera usar para inyectar el medio de contraste durante una fluoroscopia para visualizar el sitio de la anomala? A Tronco celaco D. Arteria mesentrica superior B. Arteria ilaca interna E. Arteria renal C. Arteria mesentrica inferior F. Vena porta

Introducir una vieta que describe un paciente con un problema. Cul de los siguientes trastornos gastrointestinales sera la causa ms probable de estos hallazgos? A. Esofagitis por Candida F. Estenosis pilrica B. Diverticulitis G. Enteritis regional C. Hernia de hiato H. Absceso subfrnico D. lcera pptica I. Colitis ulcerosa E. Enterocolitis pseudomembranosa Versin para datos aislados: Cul de las siguientes reas recibe suministro sanguneo a travs de la arteria cerebelosa inferior posterior? Versin para la aplicacin de conocimientos: Un hombre de 62 aos presenta ataxia en extremidad izquierda, sndrome de Horner, nistagmo y prdida de la sensacin de dolor y temperatura en la cara. Cul de las siguientes arterias es ms probable que est obstruida?

132

Ciencias del comportamiento


Introducir una vieta de paciente que describe la edad del nio y lo que puede hacer. Cul de las siguientes opciones es la que mejor describe el nivel de desarrollo? Habilidades de Habilidades Habilidades lenguaje/cognitivas motoras gruesas sociales A. Normal Normal Normal B. Normal Normal Tarda C. Normal Tarda Normal D. Normal Tarda Tarda E. Tarda Normal Normal F. Tarda Normal Tarda G. Tarda Tarda Normal H. Tarda Tarda Tarda

Un hombre de 55 aos de edad llega al departamento de urgencias debido a dolor de pecho. No presenta distrs aparente. La temperatura es de 36,9 C (98,5 F), el pulso es de 68/min, la frecuencia respiratoria es de 16/min y la presin arterial es de 130/74 mm Hg. Cul de las siguientes preguntas sera la ms adecuada para comenzar? A. Tiene antecedentes de cardiopatas o infarto de miocardio? B. Ha tenido anteriormente dolor de pecho? C. Cul fue la duracin del dolor de pecho? D. Es el dolor de pecho intenso o sordo? E. Cunteme sobre su dolor de pecho. F. Dnde se localiza el dolor de pecho?

Apndice B

133

Una mujer de 35 aos visita al mdico despus de haber encontrado una protuberancia en una mama. Llora y le dice al mdico que su madre muri de cncer de mama. Cul de las siguientes respuestas del mdico sera ms adecuada? A. Veo que el hecho de encontrar una protuberancia la perturba. B. Dudo de que esta protuberancia sea cncer de mama. C. Cunteme cmo se sinti al encontrar la protuberancia. D. Cunteme ms detalles sobre la protuberancia. E. No hay razones para preocuparse hasta que se realice la biopsia de la protuberancia. F. El tratamiento contra el cncer de mama ha mejorado mucho desde que su madre falleci a causa de esta enfermedad.

Versin para datos aislados: Cul de los siguientes datos caracteriza al trastorno de ansiedad por separacin?

Versin para la aplicacin de conocimientos: Un nio de 8 aos necesita que lo convenzan para ir a la escuela y, cuando est en la escuela, a menudo se queja de intensos dolores de cabeza o estmago. Algunas veces, su madre tiene que llevarlo a casa debido a sus sntomas. A la noche, trata de dormir con sus padres. Cuando ellos insisten en que duerma en su habitacin, les dice que hay monstruos en su armario. Cul de las siguientes opciones es la que mejor explica este comportamiento? A. Comportamiento acorde a la edad B. Esquizofrenia de la niez C.* Trastorno de ansiedad por separacin D. Fobia social E. Trastorno sictico compartido

134

Bioqumica
Introducir una vieta que describe un paciente con un problema. Cul de las siguientes enzimas digestivas o cofactores es ms probable que est involucrado/a? A. Amilasa F. Lipasa B. Quimotripsina G. Pepsina C. Colipasa H. Sucrasa D. Enteroquinasa I. Tripsina E. Lactasa

Introducir una vieta que describe un paciente con un problema. Cul de los siguientes hallazgos de laboratorio sera el ms probable? A. Hipercalcemia E. Hipernatremia B. Hipocalcemia F. Hiponatremia C. Hipermagnesemia G. Hipercaliemia D. Hipomagnesemia H. Hipocaliemia Versin para datos aislados: La porfiria aguda intermitente es el resultado de un defecto en la va biosinttica A. B. C. D. E. F. del colgeno del corticosteroide del cido graso de la glucosa del heme de la tiroxina Versin para la aplicacin de conocimientos: Un hombre sano de 33 aos presenta desde hace 6 meses episodios de debilidad leve y dolor abdominal severo y continuo con algunos calambres pero sin diarrea. Una ta y un primo han tenido episodios similares. Durante una crisis, su abdomen se distiende, y disminuyen los sonidos intestinales. El examen neurolgico revela debilidad leve en la parte superior de los brazos. Estos hallazgos sugieren la presencia de una anomala en la va biosinttica de cul de los siguientes? A. B. C. D. E.* F. Colgeno Corticosteroide cido graso Glucosa Heme Tiroxina (T4)

Apndice B

135

Versin para datos aislados: Cul de los siguientes datos caracteriza a la -fetoprotena?

Versin para la aplicacin de conocimientos: Una mujer de 45 aos presenta un agrandamiento rpido del hgado asociado a un deterioro de la funcin heptica y un cuadro de cirrosis que es el resultado de un episodio de hepatitis. Cul de las siguientes opciones es ms probable que presente una concentracin srica anormal? A. 1-Antitripsina B. Antgeno carcinoembrinico C. Gonadotropina corinica D.* -Fetoprotena E. Gastrina

Versin sin vieta: Un trastorno heredado del metabolismo de los carbohidratos se caracteriza por un aumento anormal en la concentracin del glucgeno heptico con estructura normal y sin incremento detectable en la concentracin de la glucosa srica luego de la administracin por va oral de fructosa. Estas dos observaciones sugieren que la enfermedad es el resultado de la ausencia de cul de las siguientes enzimas? A. Fructoquinasa B. Glucoquinasa C.* Glucosa 6 fosfatasa D. Fosfoglucomutasa E. Transglucosilasa UDPG-glucgeno

Versin con vieta: Un beb de 6 meses de edad presenta hgado agrandado. La evaluacin para detectar enfermedades metablicas muestra un aumento anormal en la concentracin del glucgeno heptico con una estructura normal y sin incremento detectable en la concentracin de la glucosa srica despus de la administracin por va oral de fructosa. Estas dos observaciones sugieren que la enfermedad es el resultado de la ausencia de cul de las siguientes enzimas? A. Fructoquinasa B. Glucoquinasa C.* Glucosa 6 fosfatasa D. Fosfoglucomutasa E. Transglucosilasa UDPG-glucgeno

136

Retencin de datos aislados: Cul de los siguientes minerales o vitaminas est involucrado en la sntesis del factor de coagulacin?

Aplicacin de conocimientos: Un viudo de 70 aos de edad tiene equimosis, petequias perifoliculares e inflamacin de las encas. Su alimentacin se compone mayormente de gaseosas y perros calientes. De cul de los siguientes minerales o vitaminas es ms probable que haya una deficiencia? I. Biotina J. Cobre K.* Folato L. Yodo M. Hierro N. Magnesio O. Niacina P. Zinc Las opciones alternativas (beriberi; kwashiorkor; pelagra; raquitismo; escorbuto) son aparentemente menos apropiadas.

A. Vitamina A B. Vitamina B1 C. Vitamina B2 D. Vitamina B6 E.* Vitamina C F. Vitamina D G. Vitamina E H. Vitamina K

En la va metablica ramificada, una sola enzima diferente cataliza cada uno de los pasos individuales. La enzima que se anticipa que tendr una inhibicin ms marcada como consecuencia del compuesto V es la enzima A. A B.* B C. C D. D E. E

(A) (A)

II
II (E) (E)

(B) (B)

III
III

(C) (C)

IV
IV

(D) (D)

IV
IV

Una mujer de 25 aos tiene un embarazo en la octava semana de gestacin. Su ingesta de calcio por va oral es inadecuada. Si no recibe suplementos, cul de las siguientes opciones ser la fuente principal de calcio para el feto en desarrollo? A.* hueso esponjoso D. tbulos renales B. hgado E. intestino delgado C. glndulas paratiroideas

Apndice B

137

Gentica
Un varn recin nacido presenta insuficiencia circulatoria y hemlisis severa. La madre de 26 aos es Rh negativa y tuvo dos abortos previos durante el segundo trimestre. Esta enfermedad podra haberse prevenido si se hubiera administrado a la madre tratamiento con cul de las siguientes opciones? A. IgG anti-RhD durante el embarazo ms reciente B.* IgG anti-RhD al terminar cada uno de los dos primeros embarazos C. IgM anti-RhD durante el embarazo ms reciente D. IgM anti-RhD al terminar el primer embarazo

Los genes en el cromosoma bacteriano tienen los siguientes ligamientos en la transferencia conjugativa: x e y, 25% de las veces; y e z, 50% de las veces. Si el orden del gen es x-y-z, qu porcentaje aproximado de las veces se transferirn x e z juntos? A. 1% B. 5% C. 13% D. 20% E.* 40%

La hiperamoniemia hereditaria se caracteriza por la presencia de un EEG marcadamente anormal y un aumento de la concentracin de amonaco en sangre. Es muy probable que la causa sea la deficiencia de cul de las siguientes enzimas? A. Asparagina sintetasa B.* Carbamoil fosfato sintetasa I C. Fumarasa D. Glutamato-oxalacetato aminotransferasa E. Glutaminasa

138

Histologa/Biologa celular
Introducir la descripcin de una accin. Cul de los siguientes organelos celulares est involucrado ms directamente? A. Complejo de Golgi E. Envoltura nuclear B. Lisosoma F. Grnulos secretores (zimgeno) C. Peroxisoma G. Retculo endoplsmico rugoso D. Mitocondria H. Retculo endoplsmico liso

Vieta de laboratorio
Varias clulas contiguas estn identificadas con una tincin fluorescente que no atraviesa las membranas celulares. Una clula se blanquea experimentalmente con luz que destruye la tincin pero inmediatamente recupera la fluorescencia de la tincin. La presencia de cul de las siguientes estructuras entre la clula blanqueada y las vecinas fluorescentes es la que explica mejor esta recuperacin? A. Lmina basal D. Glucosaminoglicanos B. Desmosomas (mculas adherentes) E. Uniones intercelulares hermticas (zonulae occludentes) C.* Uniones intercelulares comunicantes

Apndice B

139

Microbiologa
Plantilla para tems de microbiologa: Enunciado del tem (vieta del paciente): (Describe un paciente con su problema) Pregunta introductoria: Cul de los siguientes organismos sera la causa ms probable de la infeccin? Opciones: (Lista de patgenos)

En una fiesta, el men inclua pollo frito, papas fritas caseras, arvejas, pasteles de chocolate y caf. Despus de 2 horas, la mayora de los comensales se sintieron muy enfermos y presentaron nauseas, vmitos y dolor estomacal. Cul de los siguientes organismos es ms probable que est presente en grandes cantidades en los anlisis de los alimentos contaminados? A. Escherichia coli B. Proteus mirabilis C. Salmonella typhimurium D.* Staphylococcus aureus E. Streptococcus faecalis F. Enterococcus

Introducir una vieta que describe un paciente con un problema. Cul de las siguientes toxinas es ms probable que est involucrada en la patogenia? A. Toxina botulnica F. Enterotoxina del clera B. Toxina de la difteria G. Enterotoxina de Clostridium difficile C. Toxina de la tos ferina H. Enterotoxina termoestable de Escherichia coli D. Toxina Shiga I. Enterotoxina de Staphylococcus aureus E. Toxina tetnica

140

Introducir una vieta que describe un paciente con un problema. Cul de los siguientes agentes teraputicos sera el ms apropiado? A. Aciclovir H. Pamoato de pirantel B. Anfotericina B I. Pirazinamida C. Eritromicina J. Rifampina D. Ganciclovir K. Trimetoprima-sulfametoxazol E. Gentamicina L. Vancomicina F. Ketoconazol M. Zidovudina (AZT) G. Miconazol Item sobre datos aislados: Cul de los siguientes patgenos es un organismo grampositivo encapsulado que normalmente se desarrolla en pares o en cadenas cortas? (La misma lista de opciones que se presenta a continuacin.) Respuesta: T Item para la aplicacin de conocimientos: Una nia de 7 aos de edad tiene fiebre elevada y dolor de garganta. Presenta enrojecimiento de la faringe, la amgdala derecha hinchada con exudados cremosos y linfadenopata submandibular dolorosa en el lado derecho. El cultivo de agar sangre de la garganta produce pequeas y numerosas colonias -hemolticas que se inhiben mediante bacitracina. Cul de los siguientes patgenos es ms probable que haya causado la enfermedad? Respuesta: U A. B. C. D. E. F. G. H. I. J. K. Adenovirus Aspergillus fumigatus Bacillus anthracis Candida albicans Chlamydia psittaci Coccidioides immitis Coronavirus Corynebacterium diphtheriae Coxiella burnetii Coxsackievirus Virus de Epstein-Barr L. M. N. O. P. Q. R. S. T. U. Haemophilus influenzae Histoplasma capsulatum Mycobacterium tuberculosis Mycoplasma pneumoniae Neisseria gonorrhoeae Neisseria meningitidis Pneumocystis carinii Rinovirus Streptococcus pneumoniae Streptococcus pyogenes (grupo A)

Apndice B

141

Ejemplo de un conjunto de tems Un nio de dos aos de edad ha tenido infecciones recurrentes desde los 6 meses de edad. Las concentraciones del complemento srico, la funcin fagoctica y la actividad bactericida de los neutrfilos son normales. Una prueba cutnea con antgenos de Candida arroj como resultado una induracin de 2 cm a las 48 horas. Cul de las siguientes opciones proporciona la mejor explicacin del inicio de la enfermedad a la edad de 6 meses? A. B. C. D. E. Desarrollo de una infeccin viral Exposicin a una bacteria poco comn Prdida de la inmunidad pasiva que recibe de la madre Una anomala en la maduracin en el timo Transferencia transplacentaria de anticuerpos IgM

Cul de los siguientes anlisis de laboratorio es ms probable que sea anormal? A. B. C. D. E. Actividad de la mieloperoxidasa Proporcin de linfocitos T CD4/CD8 Concentracin de cloruro en el sudor Receptores Fc de macrfagos Concentracin de IgG srica

142

Neurociencia (Neuroanatoma y Neuropatologa)


Plantilla para tems de neuroanatoma Enunciado del tem (vieta del paciente): (Describe un paciente con su problema) Pregunta introductoria: En qu sitio sera ms probable que se presente una anomala? Opciones: (Lista de sitios)

Introducir una vieta que describe un paciente con un problema. Cul de los siguientes nervios del crneo es el sitio ms probable de la lesin subyacente? A. Nervio olfatorio G. Nervio facial B. Nervio ptico H. Nervio vestibulococlear C. Nervio motor ocular comn I. Nervio glosofarngeo D. Nervio troclear J. Nervio vago E. Nervio trigmino K. Nervio espinal accesorio F. Nervio motor ocular externo L. Nervio hipogloso

Introducir una vieta que describe un paciente con una anomala neurolgica. Cul de las siguientes ramas del plexo braquial sera ms probable que est afectada? A. Axilar G. Musculocutneo B. Dorsal de la escpula H. Radial C. Supraescapular I. Torcico largo D. Subescapular superior J. Toracodorsal E. Subescapular inferior K. Cubital F. Mediano

Apndice B

143

Introducir una vieta que describe un paciente con un problema. Cul de los siguientes componentes del sistema motor es ms probable que est involucrado? A. Ganglios basales F. Corteza motora sensorial B. Hemisferio cerebeloso G. rea pretectal/tubrculo cuadrigmino superior C. Vermis cerebeloso H. rea motora complementaria D. Relevo motor del tlamo I. Asta anterior de la mdula espinal E. rea premotora

Introducir una vieta que describe un paciente con un problema. La oclusin de cul de las siguientes arterias sera la causa ms probable? A. Cerebral anterior izquierda E. Cerebral posterior izquierda B. Cerebral anterior derecha F. Cerebral posterior derecha C. Cerebral media izquierda G. Lenticuloestriada izquierda D. Cerebral media derecha H. Lenticuloestriada derecha

Introducir una vieta que describe un paciente con un problema. Cul de las siguientes opciones sera el diagnstico ms probable? A. Esclerosis amiotrfica lateral E. Poliomielitis B. Enfermedad de Huntington F. Polineuropata C. Esclerosis mltiple G. Retinitis pigmentosa D. Enfermedad de Parkinson

144

Ejemplo de un conjunto de tems Una mujer de 58 aos de edad que no responde a los estmulos llega al departamento de urgencias luego de sufrir un desmayo en un centro de compras de la zona. Sus familiares informaron que a la maana se haba sentido bien pero que luego desarroll un intenso dolor de cabeza que empeor progresivamente. Ha tenido hipertensin y fibrilacin auricular, y toma un medicamento antihipertensivo y un anticoagulante oral. Su presin arterial es de 220/130 mm Hg y padece apneas que se alternan con hiperpneas. La paciente responde solamente a estmulos nocivos con postura extensora con compromiso del brazo y de la pierna derechos. El funduscopio revela la presencia de papiledema con compromiso del disco ptico izquierdo. Las pupilas presentan 3,0/7,0 (D/I) sin reaccin a la luz en la izquierda. Existe una desviacin de la mirada hacia la izquierda. Se observa hiperreflexia difusa, en la derecha mayor que en la izquierda, y signo de Babinski bilateral. 1. La pupila izquierda dilatada y sin reaccin concuerda ms con una lesin de cul de las siguientes estructuras en la izquierda? A. Nervio ptico D. Ncleo geniculado lateral B. Cintilla ptica E. Tubrculo cuadrigmino superior C.* Nervio motor ocular comn 2. La postura extensora en el lado derecho es ms compatible con una lesin en cul de las siguientes reas de la izquierda? A. Telencfalo D. Protuberancia B. Diencfalo E. Bulbo raqudeo C.* Mesencfalo 3. Cul de las siguientes opciones describe mejor su perfil respiratorio? A.* de Cheyne-Stokes B. Hiperventilacin neurognica central C. Apnestico D. Atxico 4. Cul de los siguientes sndromes de hernia concuerda ms con la presentacin clnica de la paciente? A. Circunvolucin del cuerpo calloso debajo de la hoz B.* Uncus del lbulo temporal a travs de la tienda C. Diencfalo a travs de la hendidura tentorial D. Tronco enceflico a travs de la hendidura tentorial E. Amgdala cerebelosa a travs del agujero occipital

Apndice B

145

Patologa
Introducir la descripcin de una accin. Cul de los siguientes organelos celulares est involucrado ms directamente? A. Complejo de Golgi E. Envoltura nuclear B. Lisosomas F. Grnulos secretores (zimgeno) C. Peroxisomas G. Retculo endoplsmico rugoso D. Mitocondria H. Retculo endoplsmico liso

Introducir una vieta que describe un paciente con un problema. Cul de los siguientes sera el hallazgo ms probable en un examen de tejido obtenido en una biopsia renal? A. Vasculitis necrotizante aguda F. Granulomas B. Depsitos amiloides G. Neutrfilos intersticiales C. Necrosis cortical H. Glomeruloesclerosis nodular D. IgA mesangial glomerular I. Tbulos regeneradores E. IgG perifrica glomerular J. Cristales de cido rico tubulares

Introducir una vieta que describe un paciente con un problema. Cul de los siguientes trastornos endocrinos sera el ms probable? A. Adenoma cortical suprarrenal F. Feocromocitoma suprarrenal B. Carcinoma cortical suprarrenal G. Adenoma de clulas del islote pancretico C. Hiperplasia suprarrenal H. Adenoma paratiroideo D. Necrosis suprarrenal I. Carcinoma paratiroideo E. Neuroblastoma suprarrenal J. Hiperplasia paratiroidea Un hombre de 32 aos muere cuatro das despus de fracturarse el fmur en un accidente automovilstico. El examen del cerebro en la autopsia muestra la presencia de petequias diseminadas en la sustancia blanca del cerebro. Qu otro hallazgo sera ms probable que se presente en la autopsia? A. Sndrome de distrs respiratorio en adultos B. Lesin de contragolpe C.* Embolia grasa D. Septicemia E. Hematoma subdural

146

La autopsia de una mujer de 24 aos muestra la presencia de pleuritis, engrosamiento membranoso de las paredes capilares glomerulares, anillos concntricos de colgeno alrededor de las arteriolas esplnicas y excrecencias en la parte inferior de la vlvula mitral. Cul de los siguientes hallazgos sera ms probable en el anlisis de la sangre de esta mujer? A.* Anticuerpos antinucleares B. Aumento de la concentracin de C3 C. Linfocitosis D. Gammapata monoclonal E. Cultivo positivo de bacterias

Un paciente con hepatitis B muere luego de 9 das del inicio de los sntomas. Cul de los siguientes hallazgos es ms probable en el examen microscpico de su hgado? A. Vacuolizacin grasa difusa con necrosis mnima B. Fibrosis difusa con presencia de ndulos C. Necrosis zonal perifrica limitada D.* Necrosis hepatocelular diseminada E. Infiltracin neutroflica diseminada en lbulos

Apndice B

147

Mecanismos de patologa
Plantilla para tems de mecanismos Enunciado del tem (vieta del paciente): (Describe un paciente con su problema) Pregunta introductoria: Cul de los siguientes mecanismos sera la causa ms probable de los hallazgos del paciente? Opciones: (Lista de mecanismos)

Introducir una vieta que describe un paciente con un problema. Cul de las siguientes opciones sera la causa ms probable de la enfermedad del miocardio? A. Toxicidad por alcohol B. Amiloidosis cardiaca C. Fibrosis endomiocrdica D. Hemocromatosis E. Miocardiopata hipertrfica F. Endocarditis de Lffler G. Miocarditis postviral H. Sarcoidosis I. Tripanosomiasis de Sudamrica (enfermedad de Chagas) J. Insuficiencia de vitamina B1 (tiamina) Introducir una vieta que describe un paciente con ictericia. Cul de las siguientes opciones sera la causa ms probable de la ictericia? A. Enfermedad heptica alcohlica F. Atresia del conducto intraheptico B. Reaccin a frmacos G. Ictericia obstructiva C. Sndrome de Dubin-Johnson H. Cirrosis biliar primaria D. Sndrome de Gilbert I. Hepatitis viral E. Ictericia hemoltica

148

Introducir una vieta que describe un paciente con anomala hemosttica. Cul de las siguientes opciones sera la causa ms probable de la anomala hemosttica? A. Coagulacin intravascular diseminada E. Hemofilia B aguda F. Prpura trombocitopnica idioptica B. Insuficiencia de factor V (proacelerina) G. Tromboembolismo pulmonar C. Insuficiencia de factor VII (proconvertina) H. Enfermedad de von Willebrand D. Hemofilia A Introducir una vieta que describe un paciente con un problema. Cul de las siguientes opciones sera la lesin gastrointestinal ms probable? A. Esofagitis por Candida F. Estenosis pilrica B. Diverticulitis G. Enteritis regional C. Hernia de hiato H. Absceso subfrnico D. lcera pptica I. Colitis ulcerosa E. Enterocolitis pseudomembranosa

Apndice B

149

Diagnstico de patologas
Plantilla para tems de diagnstico Enunciado del tem (vieta del paciente): (Describe un paciente con su problema) Pregunta introductoria: Cul de las siguientes opciones sera el diagnstico ms probable? Opciones: (Lista de diagnsticos)

Introducir una vieta que describe un paciente con anomala histolgica. Cul de las siguientes opciones sera el trastorno tiroideo ms probable? A. Tiroiditis autoinmune crnica (enfermedad de Hashimoto) B. Enfermedad de Graves C. Mixedema despus de terapia con I131 D. Tiroiditis subaguda E. Carcinoma tiroideo bien diferenciado Introducir una vieta que describe un paciente con un problema. Cul de las siguientes opciones sera el diagnstico ms probable? A. Leucemia linfoblstica aguda B. Leucemia mielgena aguda C. SIDA D. Leucemia linfoctica crnica E. Sarcoma de Ewing F. Linfoma folicular G. Enfermedad de Hodgkin H. Prpura trombocitopnica idioptica I. Mieloma mltiple J. Policitemia vera K. Histiocitosis X L. Linfoma de clula T

150

Introducir una vieta que describe un paciente con cardiopata. Cul de las siguientes opciones sera el diagnstico ms probable? A. Miocarditis viral aguda F. Miocardiopata hipertensiva B. Enfermedad de Chagas G. Miocardiopata hipertrfica C. Miocardiopata dilatada H. Miocardiopata isqumica D. Miocarditis de clulas gigantes I. Enfermedad de Pompe E. Hemocromatosis J. Miocardiopata restrictiva

Introducir una vieta que describe un paciente con un problema. Cul de los siguientes tipos de clulas sera ms probable que est involucrado en la lesin? A. Basfilos E. Clulas mesoteliales B. Clulas endoteliales F. Monocitos C. Eosinfilos G. Neutrfilos D. Linfocitos H. Clulas plasmticas

Introducir una vieta que describe un paciente con enfermedad renal. Cul de las siguientes opciones sera el diagnstico ms probable? A. Glomerulonefritis postestreptoccica E. Nefritis por lupus aguda F. Glomerulonefritis B. Esclerosis segmentaria focal membranoproliferativa C. Sndrome de Goodpasture G. Glomerulonefritis membranosa D. Nefropata IgA H. Nefropata de cambios mnimos Un hombre de 21 aos tiene prdida de peso y diarrea sanguinolenta intermitente severa. El enema de bario y la colonoscopia revelan mltiples lceras y cambios inflamatorios que se extienden desde el recto hasta el colon transverso medio. Las muestras de biopsia que se extrajeron de varios sitios revelan inflamacin crnica y aguda restringida a la mucosa. Cul de las siguientes opciones sera el diagnstico ms probable? A. Gastroenteritis asociada al SIDA E. Colitis asociada a Escherichia coli B. Amebiasis F. Colitis isqumica C.* Enfermedad de Crohn G. Gastroenteritis por Salmonella D. Colitis asociada a Clostridium difficile H. Colitis ulcerosa

Apndice B

151

Una mujer de 38 aos padece de insuficiencia cardiaca congestiva, contracciones ventriculares prematuras y episodios repetidos de taquicardia ventricular. Su presin arterial es normal. Su corazn est marcadamente agrandado. No se observan soplos; la angiografa coronaria es normal. Cul de las siguientes opciones sera el diagnstico ms probable? A. Fiebre reumtica aguda B. Fibroelastosis congnita C. Pericarditis constrictiva D.* Infarto de miocardio E. Miocardiopata primaria

Un hombre de 74 aos padece dolor abdominal con gases excesivos en el cuadrante inferior izquierdo; adems, presenta leucocitosis y fiebre. No ha tenido diarrea ni estreimiento. Cul de las siguientes opciones sera el diagnstico ms probable? A. Carcinoma del colon sigmoideo B.* Diverticulitis C. Poliposis adenomatosa familiar D. Colitis ulcerosa E. Adenoma velloso del recto superior

152

Fisiopatologa
Plantilla para tems de fisiopatologa: hallazgos complementarios Enunciado del tem (vieta del paciente): (Describe un paciente con su problema) Pregunta introductoria: Cul de los siguientes hallazgos complementarios sera el ms probable? Opciones: (Lista de hallazgos)

Los anlisis de laboratorio de un hombre de 35 aos de edad edematoso muestran una concentracin srica normal del complemento y un aumento en la concentracin srica del colesterol. El anlisis de orina revela proteinuria (4+), 0-5 eritrocitos/hpf y varios cilindros hialinos. Cul de los siguientes hallazgos sera el ms probable en una biopsia renal? A. Glomerulonefritis postestreptoccica (proliferativa) aguda B. Glomerulonefritis membranoproliferativa C.* Glomerulonefritis membranosa D. Enfermedad de cambios mnimos E. Glomerulonefritis rpidamente progresiva

Un adolescente de 16 aos se somete a una evaluacin de ictericia. Los estudios de laboratorio revelan actividades normales en las enzimas hepticas, prueba de antiglobulina directa negativa, aumento en la concentracin de hemoglobina corpuscular media y aumento de la fragilidad osmtica de los eritrocitos. Cul de los siguientes tipos de eritrocitos es ms probable que est presente en el frotis de sangre perifrica? A. Ovalocito D. Clula blanco B. Esquistocito E. Clula en forma de lgrima C.* Esferocito

Apndice B

153

Una mujer asintomtica de 50 aos tiene hipertensin arterial. Presenta aumento de la excrecin urinaria de catecolaminas. Una tomografa computarizada revela una masa suprarrenal. Cul de los siguientes hallazgos sera ms probable en el examen microscpico de la masa resecada? A. Neoplasia benigna de la corteza suprarrenal D. Neoplasia maligna de la mdula suprarrenal B.* Neoplasia benigna de la mdula suprarrenal E. Hiperplasia difusa de la corteza suprarrenal C. Neoplasia maligna de la corteza F. Hipoplasia difusa de la mdula suprarrenal suprarrenal

Introducir una vieta que describe un paciente con un problema. Cul de los siguientes sera el hallazgo pulmonar ms probable en la autopsia? A. Suave con consistencia esponjosa, mltiples ampollas en la superficie B. Muy pesado; supura lquido sanguinolento y espumoso libremente desde la superficie de corte C. Ndulos fibrticos difusos; aspecto espiralado que reemplaza el parnquima pulmonar normal D. Consistencia parecida a la del hgado en el lbulo inferior izquierdo; microscpicamente contiene fibrinas y neutrfilos en los alvolos E. Aspecto normal; cogulo laminado, en espiral, con forma de Y que casi llena ambas arterias pulmonares

Introducir una vieta que describe un paciente con un problema. Cul de los siguientes conjuntos de hallazgos sera el ms probable? Volumen circulatorio efectivo Volumen del lquido extracelular Volumen de plasma Excrecin de Na+ en orina A. disminuido disminuido disminuido disminuida B. disminuido aumentado disminuido disminuida C. disminuido aumentado aumentado disminuida D. aumentado aumentado aumentado aumentada

Introducir una vieta que describe un paciente con un problema. Cul de las siguientes enzimas digestivas o cofactores sera ms probable que est involucrada? A. Amilasa F. Lipasa B. Quimotripsina G. Pepsina C. Colipasa H. Sucrasa D. Enteroquinasa I. Tripsina E. Lactasa

154

Introducir una vieta que describe un paciente con un problema. Cul de los siguientes hallazgos de laboratorio sera el ms probable? A. Hipercalcemia E. Hipernatremia B. Hipocalcemia F. Hiponatremia C. Hipermagnesemia G. Hipercaliemia D. Hipomagnesemia H. Hipocaliemia

Introducir una vieta que describe un paciente con un problema. Cul de las siguientes opciones sera el perfil de funcin tiroidea ms probable? Tiroxina (T4) Captacin de T3 Hormona estimulante T3 (triyodotironina) por resina de la tiroides A. B. normal C. normal normal D. normal normal E. F. normal G. normal normal normal normal

Introducir una vieta que describe un paciente con unos datos electrocardiogrficos. Cul de las siguientes arritmias cardiacas sera la ms probable? A. Fibrilacin auricular F. Contracciones auriculares prematuras B. Taquicardia paroxstica auricular G. Contracciones ventriculares prematuras C. Bloqueo cardiaco incompleto de primer grado H. Arritmia sinusal D. Bloqueo cardiaco incompleto de segundo grado I. Fibrilacin ventricular E. Bloqueo cardiaco completo de tercer grado J. Taquicardia paroxstica ventricular

Apndice B

155

Introducir una vieta que describe un paciente con anomalas cardiacas (Un beb de cuatro semanas tiene un soplo sistlico fuerte con estremecimiento catreo sistlico; por lo dems, el beb tiene un aspecto saludable. No se observa cianosis.) Cul de las siguientes cardiopatas congnitas sera ms probable? A. Comunicacin interauricular E. Conducto arterioso permeable B. Coartacin de la aorta F. Estenosis de la vlvula pulmonar C. Transposicin completa de las grandes arterias G. Tetraloga de Fallot D. Defecto del cojn endocrdico H. Comunicacin interventricular

156

Farmacologa
Plantilla para reacciones adversas en farmacologa: Adivine cul es mi medicamento Enunciado del tem (vieta del paciente): (Describe un paciente y la reaccin farmacolgica adversa) Pregunta introductoria: Cul de los siguientes medicamentos es ms probable que haya tomado el paciente? Opciones: (Lista de medicamentos)

Un paciente que se presenta en el departamento de urgencias no recuerda cul es el medicamento para el corazn que toma. Manifiesta que siente un zumbido en los odos. Su frecuencia cardiaca es superior a 80/min. El ECG muestra intervalos PR y QRS prolongados. Cul de los siguientes medicamentos es ms probable que haya tomado el paciente? A. Digoxina B. Lidocana C. Fenitona D. Propranolol E.* Quinidina

Introducir una vieta que describe un paciente que ha tenido una reaccin adversa a un medicamento. Cul de los siguientes medicamentos tiene ms probabilidades de haber causado la reaccin adversa? A. Paracetamol J. cido nalidxico B. Amiodarona K. Nitrofurantona C. Inhibidores de la ECA L. Penicilina D. Aspirina M. Prednisona E. Atenolol N. Procainamida F. Bleomicina O. Propranolol G. Citosina arabinosida P. Sulfasalazina H. Furosemida Q. Tetraciclina I. Metronidazol R. Verapamilo

Apndice B

157

El medicamento Y tiene un volumen de distribucin (Vd) de 75 L, tanto en hombres jvenes como adultos ancianos. En los adultos ms jvenes, tiene un ndice de depuracin de 15 L/h; el 50% de sta se realiza a travs del hgado y el resto a travs de los riones. Para los hombres ms jvenes, el rgimen de mantenimiento es de 100 mg cada 6 horas. Cul de los siguientes regmenes producir esencialmente la misma concentracin en estado estable en un hombre de ms edad, cuya eliminacin de creatinina se reduce a la mitad de la de un hombre ms joven, pero que no presenta trastornos de la funcin heptica? A. 75 mg cada 3 horas B.* 75 mg cada 6 horas C. 75 mg cada 9 horas D. 100 mg cada 3 horas E. 100 mg cada 6 horas F. 100 mg cada 12 horas Una mujer sexualmente activa de 24 aos presenta un aumento en sus secreciones vaginales. El examen plvico revela una secrecin verde y espumosa. El examen microscpico de una preparacin en fresco de la secrecin revela organismos unicelulares mviles de 10 a 30 de longitud. Cul de las siguientes caractersticas de este trastorno explica la razn por la cual esta infeccin se puede tratar eficazmente con metronidazol? A. Facultativamente aerobio B. Microaerfilo C. Estrictamente aerobio D.* Estrictamente anaerobio

Plantilla para tems de mecanismos o sitios de accin: farmacologa Enunciado del tem (vieta del paciente): (Describe un paciente que necesita farmacoterapia) Pregunta introductoria: El frmaco que tenga cul de los siguientes sitios/mecanismos de accin es ms probable que sea eficaz? Opciones: (Lista de mecanismos de accin) o (Lista de sitios de accin)

158

Farmacoterapia
Plantilla para tems de farmacoterapia Enunciado del tem (vieta del paciente): (Describe un paciente que necesita farmacoterapia) Pregunta introductoria: Cul de los siguientes medicamentos sera el ms apropiado para administrarle? Opciones: (Lista de medicamentos)

Un hombre afroamericano de 40 aos tiene un comienzo sbito de vmitos, mareos y dolor de cabeza severos. Su presin arterial es de 260/130 mm Hg; padece de encefalopata y retinopata de grado IV. Cul de los siguientes medicamentos sera el ms apropiado para administrarle? A. Alfa agonistas G. Simpaticolticos centrales B. Alfabloqueantes H. Vasodilatadores directos C. Inhibidores de la ECA I. Agentes inotrpicos negativos D. Beta agonistas J. Diurticos tiazidas E. Betabloqueantes K. Vasoconstrictores F. Glucsidos cardiacos

Introducir una vieta que describe un paciente que necesita farmacoterapia. Cul de las siguientes opciones sera la farmacoterapia inicial ms adecuada? A. Adenosina G. Naloxona B. Aspirina H. Prednisona C. Cafena I. Propranolol D. Epinefrina J. Quinidina E. Insulina K. Estreptoquinasa F. Lidocana

Apndice B

159

Introducir una vieta que describe un paciente que necesita farmacoterapia. Cul de los siguientes agentes teraputicos sera el ms apropiado? A. Carbamacepina F. Carbonato de litio B. Dextroanfetamina G. Fenobarbital C. Etosuximida H. Primidona D. Haloperidol I. Propranolol E. L-dopa/carbidopa J. Piridostigmina

Introducir una vieta que describe un paciente que necesita farmacoterapia. Cul de los siguientes agentes teraputicos sera el ms apropiado? A. Atropina F. Leche de magnesia B. Subsalicilato de bismuto G. Misoprostol C. Cimetidina H. Omeprazol D. Difenoxilato I. Ranitidina E. Caoln J. Sucralfato

Introducir una vieta que describe un paciente que necesita farmacoterapia. Cul de las siguientes opciones sera el diurtico ms adecuado? A. Inhibidor de la anhidrasa carbnica B. De asa o de alto techo C. Diurtico ahorrador de potasio no esteroideo D. Diurtico osmtico E. Diurtico ahorrador de potasio esteroideo F. Tiazida G. Xantina

160

Fisiologa
Un paciente anestesiado recibe ventilacin mecnica. Los valores iniciales de gases en sangre arterial son normales. Si la ventilacin disminuye, cul de las siguientes opciones describe mejor el pH y el PCO2 arterial? PCO2 arterial A. Disminucin B. Disminucin C.* Disminucin D. Aumento E. Aumento F. Aumento pH Disminucin Aumento Sin cambios Disminucin Aumento Sin cambios

Introducir una vieta que describe un paciente con un problema. Cul de las siguientes sustancias es ms probable que est involucrada? A. ADH (vasopresina) E. Bradiquinina B. Aldosterona F. Calcitonina C. Angiotensina G. Hormona paratiroidea D. Pptido natriurtico auricular H. Renina

Un hombre de 22 aos con antecedentes de poliuria y polidipsia desde hace tres semanas ha tenido nuseas, vmitos y disminucin de la respuesta durante las ltimas 12 horas. El anlisis de orina revela glucosa 4+ y cetonas 4+. Cul de los siguientes conjuntos de hallazgos sobre los gases en sangre arterial sera el ms probable? pH PO2 (mm Hg) PCO2 (mm Hg) HCO3 (mEq/L) A. 7,15 98 33 11 B.* 7,15 98 24 8 C. 7,30 56 80 38 D. 7,40 100 40 25 E. 7,50 100 33 25 F. 7,50 100 24 18 G. 7,50 56 33 25

Apndice B

161

Introducir una vieta que describe un paciente con un problema. Cul de las siguientes sustancias humorales sera ms probable que est involucrada? A. Acetilcolina G. Prostaciclina (PGI2) B. Adenosina H. Prostaglandina E2 C. Aldosterona I. Prostaglandina F2 D. Bradiquinina J. Serotonina E. Epinefrina K. Tromboxano A2 F. Norepinefrina

162

Conjuntos de tems integradores

Plantilla para tems con indicaciones integradoras: microbiologa/farmacologa Enunciado del tem (vieta del paciente) (Describe un paciente con un problema) Pregunta introductoria: Cul de los siguientes patgenos sera la causa ms probable de la infeccin? Opciones: (Lista de patgenos) Pregunta introductoria: Cul de los siguientes medicamentos sera el ms apropiado para administrarle? Opciones: (Lista de medicamentos)

Apndice B

163

Un hombre de 40 aos con SIDA tiene antecedentes de fiebre leve y letargia desde hace una semana. Su temperatura corporal es de 38C (100,4F), y la presin arterial es de 110/70 mm Hg. Se observa una resistencia leve a la flexin pasiva del cuello. Los anlisis de laboratorio del lquido cefalorraqudeo revelan: Presin de apertura 210 mm Hg Aspecto color paja Recuento de leucocitos 400/mm3 (100% linfocitos) Recuento de eritrocitos 50/mm3 Glucosa 30 mg/dL Protena 100 mg/dL Preparacin con tinta china levadura encapsulada 1. Cul de los siguientes patgenos sera la causa ms probable de la infeccin? A. Blastomyces B. Cryptococcus C. Histoplasma D. Toxoplasma gondii E. Treponema pallidum 2. Cul de los siguientes medicamentos sera el ms apropiado para administrarle? A. Anfotericina B. Ketoconazol C. Miconazol D. Nistatina E. Trimetoprima-sulfametoxazol

164

Ejemplo de un conjunto de tems Una mujer de 34 aos de edad sufre de diarrea acuosa desde hace cuatro das. Hace dos meses tuvo mononucleosis infecciosa. Es drogadicta, usa drogas por va intravenosa; y es seropositiva para el VIH. El examen fsico muestra deshidratacin y debilidad muscular evidente. 1. Cul de las siguientes anomalas en los anlisis de laboratorio es la ms probable? D.* Aumento de la concentracin srica de Na+ A. Disminucin de la concentracin srica de K+ 2+ B. Disminucin de la concentracin srica de Ca E. Aumento del pH srico C. Aumento de la concentracin srica de HCO3 2. Para evaluar la causa de la diarrea, cul de los siguientes estudios es el ms adecuado? A. Biopsia de colon para identificar la presencia de Giardia lamblia B. Cultivo de material de la cavidad oral para detectar Candida albicans C. Biopsia del duodeno para identificar la presencia de Entamoeba histolytica D. Aspiracin gstrica para identificar la presencia de Mycobacterium avium-intracellulare E.* Muestra de heces para identificar la presencia de Cryptosporidium 3. Los estudios posteriores que se realizaron para evaluar su infeccin por VIH muestran que la razn de linfocitos T cooperadores a linfocitos T supresores es de 0,3. Cul de las siguientes acciones del VIH es la que mejor explica esta razn? A. Induccin de la proliferacin de linfocitos T cooperadores B. Induccin de la proliferacin de linfocitos T supresores C.* Infeccin de las clulas con receptor CD4 D. Infeccin de macrfagos E. Estimulacin de la sntesis de leucotrienos

Apndice B

165

Plantilla para tems de patologa/farmacologa Enunciado del tem (vieta del paciente): (Describe un paciente con su problema) Pregunta introductoria: Cul de las siguientes opciones sera el diagnstico ms probable? Opciones: (Lista de diagnsticos) Segunda pregunta introductoria: Cul de los siguientes medicamentos sera el ms apropiado para administrarle? Segunda lista de opciones: (Lista de medicamentos)

Ejemplo de un conjunto de tems 1. Un hombre de 62 aos que padece alcoholismo ingresa al hospital para una reseccin transuretral de la prstata. A la maana siguiente, mientras es llevado al quirfano, tiene dos convulsiones generalizadas en un lapso de 5 minutos. El examen neurolgico no presenta anomalas focales. Cul de las siguientes opciones sera el diagnstico ms probable? A.* Abstinencia alcohlica C. Convulsin compleja parcial B. Sndrome de Korsakoff D. Encefalopata de Wernicke 2. El tratamiento ms apropiado es la administracin intravenosa de cul de los siguientes medicamentos? A. Diazepam D. Fenitona B. Haloperidol E. Valproato C. Fenobarbital

166

Ejemplo de un conjunto de tems

Un nio de dos aos de edad ha tenido infecciones recurrentes desde los 6 meses de edad. Las concentraciones del complemento srico, la funcin fagoctica y la actividad bactericida de los neutrfilos son normales. Una prueba cutnea con antgenos de Candida arroj como resultado una induracin de 2 cm a las 48 horas. Cul de las siguientes opciones proporciona la mejor explicacin del inicio de la enfermedad a la edad de 6 meses? A. Desarrollo de una infeccin viral B. Exposicin a una bacteria poco comn C. Prdida de la inmunidad pasiva que recibe de la madre D. Una anomala en la maduracin en el timo E. Transferencia transplacentaria de anticuerpos IgM

Cul de los siguientes anlisis de laboratorio es ms probable que sea anormal? A. Actividad de la mieloperoxidasa B. Proporcin de linfocitos T CD4/CD8 C. Concentracin de cloruro en el sudor D. Receptores Fc en macrfagos E. Concentracin de IgG srica

Plantilla para tems integradores de fisiologa/farmacologa Enunciado del tem (vieta del paciente): (Describe un paciente con su problema) Pregunta introductoria: La administracin de un medicamento dirigido a producir cul de los siguientes efectos sera ms adecuado? O BIEN Cul de los siguientes efectos es ms probable que produzca la administracin de {medicamento especfico}? Opciones: (Lista de efectos fisiolgicos)

Apndice B

167

Ejemplos de listas de opciones para la redaccin de tems para las ciencias clnicas
Componentes de la vieta del paciente Edad, sexo (por ejemplo, hombre de 45 aos de edad) Lugar de atencin mdica (por ejemplo, llega al departamento de urgencias) Motivo de presentacin (por ejemplo, debido a un dolor de cabeza) Duracin (por ejemplo, constante desde hace 2 das) Historia clnica del paciente (incluir antecedentes familiares?) Determinaciones del examen fsico +/- Resultados de los estudios de diagnstico +/- Tratamiento inicial, hallazgos posteriores, etc.

Masa abdominal A. B. C. D. E. F. G. H. I. Absceso abdominal Leucemia linfoctica aguda Carcinoma del colon Carcinoma de ovario Quiste del coldoco Estreimiento Tumor desmoide Vejiga distendida Hepatoma J. K. L. M. N. O. P. Q. R. Hernia Quiste hidatdico Enfermedad intestinal inflamatoria Neuroblastoma Pseudoquiste pancretico Rin poliqustico Fibroides uterinos Vlvulo Tumor de Wilms

Para cada paciente que tiene una masa abdominal, seleccione el diagnstico ms probable.

168

Dolor abdominal A. B. C. D. E. F. G. H. I. J. Aneurisma abdominal Apendicitis Obstruccin intestinal Colecistitis Cncer de colon Estreimiento Diverticulitis Ruptura de embarazo ectpico Endometriosis Hernia K. L. M. N. O. P. Q. R. S. T. Clculo renal Adenitis mesentrica Trombosis de la arteria mesentrica Ruptura de quiste ovrico Pancreatitis Enfermedad inflamatoria plvica lcera pptica lcera pptica perforada Pielonefritis Torsin

Para cada paciente que padece dolor abdominal, seleccione el diagnstico ms probable.

Estado mental alterado A. Absceso cerebral B. Oclusin cerebrovascular/ accidente isqumico transitorio C. Deshidratacin D. Sobredosis de medicamentos/ Toxicidad medicamentosa E. Encefalopata heptica F. Hipercalcemia G. Hiperglucemia H. Hipercaliemia I. J. K. L. M. N. O. P. Q. R. Encefalopata hipertensiva Hipoglucemia Hiponatremia Hipotiroidismo Demencia por infartos mltiples Demencia degenerativa primaria, tipo Alzheimer Convulsin Sepsis Uremia Encefalopata de Wernicke

Para cada paciente con estado mental alterado, seleccione el diagnstico ms probable.

Apndice B

169

Anemia A. B. C. D. Leucemia mieloblstica aguda Anemia aplsica Leucemia linfoctica crnica Anemia hemoltica inmune inducida por medicamentos E. Malaria falciparum F. Deficiencia de folato G. Deficiencia de glucosa 6 fosfato deshidrogenasa H. I. J. K. L. M. N. O. P. Esferocitosis hereditaria Anemia ferropnica Metstasis maligna a mdula sea Anemia hemoltica microangioptica Mieloma mltiple Mielofibrosis Anemia falciforme Talasemia menor Deficiencia de vitamina B12 (cianocobalamina)

Para cada paciente con anemia, seleccione el diagnstico ms probable.

Dolor de espalda A. B. C. D. Espondilitis anquilosante Infeccin en disco intervertebral Mieloma mltiple Dolor miofascial E. F. G. H. Osteoporosis Estenosis de la columna vertebral Espondillisis Tuberculosis de columna vertebral

Para cada paciente que padece dolor de espalda, seleccione el diagnstico ms probable.

170

Comportamiento anormal A. B. C. D. E. F. G. H. I. Acidosis Psicosis aguda Hematoma subdural agudo Intoxicacin por alcohol Sobredosis/ingesta de cocana Delirium tremens Depresin Hipoglucemia Hipoxia J. K. L. M. N. O. P. Q. Ingestin de LSD Simulacin Meningitis Sndrome de Reye Psicosis esteroidea Hemorragia subaracnoidea Convulsin del lbulo temporal Sobredosis de antidepresivos tricclicos

Para cada paciente con comportamiento anormal, seleccione el diagnstico ms probable.

Dolencia/bulto mamario A. B. C. D. E. F. G. Quiste mamario Necrosis grasa de la mama Fibroadenoma de la mama Galactocele Ginecomastia Carcinoma inflamatorio de la mama Papiloma intraductal H. I. J. K. L. M. Lipoma de la mama Mastodinia Carcinoma metastsico de la mama Enfermedad de Paget de la mama Mastitis puerperal Sndrome de Tietze

Para cada paciente con un trastorno relacionado con las mamas, seleccione el diagnstico ms probable.

Apndice B

171

Dolor en el pecho A. B. C. D. E. F. G. Angina de pecho Fractura de compresin de la columna Aneurisma artico disecante Espasmo esofgico Esofagitis Herpes zoster Hiperventilacin H. I. J. K. L. M. Infarto de miocardio Dolor del msculo pectoral Pericarditis Neumona Neumotrax Embolia pulmonar

Para cada paciente con dolor de pecho, seleccione el diagnstico ms probable.

Diarrea A. B. C. D. E. F. G. H. I. Amebiasis Gastroenteritis bacteriana Sndrome carcinoide Carcinoma del colon Pancreatitis crnica Enfermedad de Crohn Infeccin por cryptosporidium Diverticulitis Sndrome de vaciamiento rpido J. K. L. M. N. O. P. Q. R. Linfoma gstrico Colitis isqumica Abuso de laxantes Colitis pseudomembranosa Sndrome del intestino corto Espre Adenoma velloso Gastroenteritis viral Sndrome de Zollinger-Ellison

Para cada paciente con diarrea, seleccione el diagnstico ms probable.

172

Cansancio A. B. C. D. E. F. G. Leucemia aguda H. Anemia: enfermedad crnica I. Insuficiencia cardiaca congestiva J. Depresin K. Infeccin por el virus EB L. Deficiencia de folato M. Deficiencia de glucosa 6 fosfato deshidrogenasa Esferocitosis hereditaria Hipotiroidismo Deficiencia de hierro Enfermedad de Lyme Tuberculosis miliar Deficiencia de vitamina B12

Para cada paciente que padece cansancio, seleccione el diagnstico ms probable.

Fiebre/Adultos A. B. C. D. E. F. G. H. I. Apendicitis Intoxicacin por aspirina Celulitis Enfermedad de Crohn Gastritis Enfermedad de Hodgkin Mononucleosis infecciosa Meningitis Sobredosis de narcticos J. K. L. M. N. O. P. Q. R. Pancreatitis Neumona Prostatitis Embolia pulmonar Pielonefritis Sinusitis Tuberculosis Infeccin del tracto urinario Infeccin respiratoria viral

Para cada paciente con fiebre, seleccione el diagnstico ms probable.

Apndice B

173

Fiebre en nios A. B. C. D. E. F. G. H. I. Leucemia linfoctica aguda Meningitis bacteriana Varicela Fiebre medicamentosa Gastroenteritis Artritis reumatoide juvenil Enfermedad de Kawasaki Mastoiditis Sarampin J. K. L. M. N. O. P. Q. R. Osteomielitis Otitis media Neumona neumoccica Pielonefritis Fiebre reumtica Rosola Rubola Sinusitis Infeccin estreptoccica

Para cada paciente con fiebre, seleccione el diagnstico ms probable.

Hemorragias gastrointestinales A. B. C. D. E. F. G. H. I. Amebiasis Angiodisplasia del colon Colitis por Clostridium difficile Carcinoma del colon Carcinoma del esfago Carcinoma de estmago Coagulopata Diverticulitis Epistaxis J. K. L. M. N. O. P. Q. R. Vrices esofgicas Hemorroides Enfermedad intestinal inflamatoria Colitis isqumica Desgarro de Mallory-Weiss lcera pptica Esofagitis por reflujo Infeccin por Salmonella Infeccin por Shigella

Para cada paciente con hemorragia gastrointestinal, seleccione el diagnstico ms probable.

174

Dolor de cabeza A. B. C. D. E. F. G. H. Meningitis asptica Meningitis bacteriana Absceso cerebral Metstasis cerebral Oclusin cerebrovascular Cefalea en racimos Cefalea inducida por drogas Disfuncin de la articulacin temporomandibular I. J. K. L. M. N. O. P. Q. Crisis hipertensiva Migraa Tumor cerebral primario Pseudotumor cerebral Endocarditis bacteriana subaguda Hemorragia subaracnoidea Arteritis temporal Cefalea por tensin Neuralgia del trigmino

Para cada paciente con dolor de cabeza, seleccione el diagnstico ms probable.

Dolor en la pierna A. B. C. D. E. F. G. H. I. Insuficiencia arterial aguda Enfermedad de Buerger Causalgia Celulitis Insuficiencia arterial crnica Estasis venosa crnica Dermatomiositis Neuropata diabtica Congelacin localizada J. K. L. M. N. O. P. Q. R. Sndrome de Leriche Linfedema Obstruccin de la vena cava inferior Osteomielitis lcera por presin Enfermedad de Raynaud Tromboflebitis Vena varicosa Insuficiencia venosa

Para cada paciente con dolor en la pierna, seleccione el diagnstico ms probable.

Apndice B

175

Debilidad en las extremidades A. B. C. D. E. F. G. H. I. Plexopata braquial Subluxacin C1-2 Subluxacin C7-T1 Encefalopata Absceso epidural Fractura de la columna cervical Sndrome de Guillain-Barr Neuropata por plomo Migraa J. K. L. M. N. O. P. Q. R. Esclerosis mltiple Miastenia grave Osteoartritis Poliomielitis Estado postictal Isquemia de disco cervical con ruptura Accidente cerebrovascular transitorio Hematoma subdural Metstasis vertebral

Para cada paciente con debilidad en una extremidad, seleccione el diagnstico ms probable.

Nuseas/Vmitos A. B. C. D. E. F. G. H. I. Bulimia nerviosa Intoxicacin por digoxina Obstruccin de la salida gstrica Gastroenteritis Gastroparesis Hepatitis Enfermedad intestinal inflamatoria Hemorragia intracraneal Laberintitis J. K. L. M. N. O. P. Q. R. Meningitis Migraa Infarto de miocardio Carcinoma pancretico Pancreatitis Embarazo Vlvulo sigmoideo Obstruccin del intestino delgado Ataque de sncope vasovagal

Para cada paciente con nuseas y vmitos, seleccione el diagnstico ms probable.

176

Trastornos respiratorios A. B. C. D. E. F. G. H. I. Reaccin alrgica Neumona por aspiracin Bronquiectasia Carcinoma de pulmn Enfermedad pulmonar obstructiva crnica Insuficiencia cardiaca congestiva Fibrosis qustica Efectos secundarios de medicamentos Asma inducido por el ejercicio J. K. L. M. N. O. P. Q. R. Presencia de cuerpo extrao Reflujo gastroesofgico Estenosis mitral Asma ocupacional Pleuresa Embolia pulmonar Virus respiratorio sincitial Sarcoidosis Tuberculosis

Para cada paciente con sntomas de problemas respiratorios, seleccione el diagnstico ms probable.

Shock A. B. C. D. E. F. G. H. I. Crisis addisoniana Anafilaxia Fibrilacin auricular Bloqueo aurculoventricular lcera pptica con hemorragia Sndrome carcinoide Taponamiento cardiaco Miocardiopata Deshidratacin J. K. L. M. N. O. P. Q. Fractura de pelvis Infarto de miocardio mbolo pulmonar Ruptura de aneurisma artico abdominal Ruptura de absceso del apndice Ruptura de embarazo ectpico Ruptura de quiste ovrico Obstruccin del intestino delgado

Para cada paciente con shock, seleccione el diagnstico ms probable.

Apndice B

177

Disnea A. B. C. D. E. F. G. H. I. Absceso de pulmn Sndrome de distrs respiratorio en adultos Anemia Asma Enfermedad pulmonar obstructiva crnica Insuficiencia cardiaca congestiva Ruptura diafragmtica Presencia de cuerpo extrao Hiperventilacin J. K. L. M. N. O. P. Q. R. Espasmo larngeo Mesotelioma Miastenia grave Tumor de Pancoast Derrame pleural Neumotrax Hipertensin pulmonar primaria Embolia pulmonar Fibrosis pulmonar

Para cada paciente con disnea, seleccione el diagnstico ms probable.

Problemas urinarios A. B. C. D. E. F. G. H. Cistitis aguda Retencin urinaria aguda Carcinoma de vejiga Carcinoma de prstata Diabetes inspida Diabetes mellitus Hipercalcemia Cistitis intersticial I. J. K. L. M. N. O. Vejiga neurognica Polidipsia psicognica Hiperplasia prosttica Prostatitis Pielonefritis Carcinoma de clulas renales Estenosis uretral

Para cada paciente con problemas urinarios, seleccione el diagnstico ms probable.

178

Sntomas urinarios A. B. C. D. E. F. G. H. I. Retencin urinaria aguda Fstula vesical Carcinoma de vejiga Cistitis Efectos de medicamentos Endometriosis Gonorrea Hemoglobinuria de March Menstruacin J. K. L. M. N. O. P. Q. Mioglobinuria Nefrolitiasis Neumaturia Prostatismo Carcinoma renal Incontinencia por estrs Sfilis Trauma

Para cada paciente con sntomas de problemas urinarios, seleccione el diagnstico ms probable.

Hemorragia/dolor vaginal A. B. C. D. E. F. G. H. I. Abruptio placentae Vaginitis bacteriana Hemorragia de disrupcin Vaginitis por Candida Carcinoma cervical Condiloma acuminado Embarazo ectpico Cncer de endometrio Presencia de cuerpo extrao J. K. L. M. N. O. P. Q. Cervicitis gonoccica Embarazo molar Menstruacin normal Placenta previa Amenaza de aborto espontneo Tricomoniasis Fstula vesicovaginal Carcinoma de vulva

Para cada paciente con dolor o hemorragia vaginal, seleccione el diagnstico ms probable.

Apndice B

179

Prurito/secreciones vaginales A. B. C. D. E. F. G. H. Vaginitis alrgica Vaginitis atrfica Vaginitis bacteriana Hemorragia de disrupcin Vaginitis por Candida Carcinoma cervical Condiloma acuminado Cncer de endometrio I. J. K. L. M. N. O. P. Presencia de cuerpo extrao Cervicitis gonoccica Menstruacin normal Placenta previa Amenaza de aborto espontneo Tricomoniasis Fstula vesicovaginal Carcinoma de vulva

Para cada paciente con prurito o secreciones vaginales, seleccione el diagnstico ms probable.

Sibilancias A. B. C. D. E. F. G. H. I. Angioedema Asma Bronquiectasia Sndrome carcinoide Enfermedad pulmonar obstructiva crnica Insuficiencia cardiaca congestiva Fibrosis qustica Plipo endobronquial Epiglotitis J. K. L. M. N. O. P. Q. R. Reflujo esofgico Presencia de cuerpo extrao Bocio Discinesia larngea Neumona Edema pulmonar Embolia pulmonar Infeccin por virus respiratorio sincitial Crup viral

Para cada paciente con sibilancias, seleccione el diagnstico ms probable.

180

También podría gustarte